Download as pdf or txt
Download as pdf or txt
You are on page 1of 46

2

BLOCK 1: ELEMENTARY GROUP THEORY


TABLE OF CONTENTS
Page No.
UNIT-1: Fundamentals of Groups
BLOCK 1.1A ( BLOCK I: Elementary Group Theory)
UNIT-2: Group Homomorphism
Fundamentals of Groups 1-15
Unit-1
UNIT-3: Isomorphism and Automorphism Theorems
Group Homomorphism 16-25
Unit-2
UNIT-4: Permutation groups.
Isomorphism and Automorphism Theorems 26-43
Unit-3
Permutation groups 44-57
Unit-4

BLOCK 1.1 B ( BLOCK II: Advance Group Theory)


Class equation of finite groups 58-68
Unit-5
Applications of Class equations 69-79
Unit-6

Unit-7 Sylow’s Theorems 80-92


Applications of Sylow’s Theorem 93-103
Unit-8

BLOCK 1.1C (BLOCK III: Elementary Ring Theory)

Fundamentals of Rings 104-113


Unit-9
Ring Homomorphism and Isomorphism 114-124
Unit-10
Special Types of Ideals 125-135
Unit-11
The Field of Quotients 136-145
Unit-12

BLOCK 1.1D ( BLOCK IV: Advance Ring Theory)


Euclidean rings 146-155
Unit-13
Classical Theorems on Euclidean rings 156-163
Unit-14
Polynomial rings 164-175
Unit-15

Unit-16 Polynomial over the rational field 176-185

iv

3 1. UNIT : FUNDAMENTALS OF GROUPS 4

1 UNIT : Fundamentals of Groups is unique. Similarly, ∗ will be a binary operation of S if and only if a ∗ b ∈ S for

all a, b ∈ S and a ∗ b is unique. This is said to be closure property of the binary

operation and the set S is said to be closed with respect to the binary operation.
OBJECTIVES

After studying this unit you will be able to: Example 1.1. The addition (+) and multiplication (×) are binary operationson

• define the binary operation and algebraic structures with examples; the set N of natural numbers, for, the sum or product of two natural numbers

• explain the concepts of group and their elementary properties; are also natural numbers. Therefore, N is closed with respect to addition and

• define the abelin group and their properties with an example; multiplication i.e., a + b ∈ N for all a, b ∈ N and a × b ∈ N for all a, b ∈ N,

• understanding fundamental properties of subgroups; respectively.

• distinguish between an order of a group and order of an element with exam- Note 1.1. The subtraction is not a binary operation on N, for 3 − 8 = −5 ∈
/ N
ples;
whereas 3 ∈ N, 8 ∈ N. But subtraction is a binary operation on integer I (i.e.,
• explain the concepts of cyclic group, normal subgroups, quotient group and
their propeties. a number with no decimal or fractional part from the set of positive and negative

• expalin the fundamnetal of permutation groups. numbers, including zero).

INTRODUCTION Definition 1.2. A non-empty set G together with one or more binary operations

is called an algebraic structure. Suppose ∗ (or ◦) is a binary operation on given


The core of a first course in abstract algebra deals with groups, rings, and fields.
The structure of a group is one of the simplest and important mathematical struc- set S. Then (S, ∗) is an algebraic structure.
tures. They have found wide application in physical sciences and biological sci-
ences, especially in the study of crystal structure, configuration molecules and
Example 1.2. (N, +), (I, +), (I, −), (R, +, ·) are all algebraic structures, where
structures of human genes. In this block, we shall study the basic definitions and
terminologies along with their properties. R is the set of real numbers.

PRELIMINARIES
Definition 1.3. A system (G, ∗), where G is a non-empty set and ∗ is a binary

Definition 1.1. Let S be a nonempty set. Then S × S ={(a, b) : a ∈ S, b ∈ S}. If operation on G is called a group, if it satisfies the following axioms:

the function f : S × S → S, then f is said to be binary operation on S. Thus (i) a ∗ b ∈ G for all a, b ∈ G (Closure Law).

a binary operation on S is a function that assign each ordered pairs of elements (ii) a ∗ (b ∗ c) = (a ∗ b) ∗ c for all a, b, c ∈ G (Associative Law).

of S. (iii) For each a ∈ G, there exist an elements e ∈ G such that a ∗ e = a = e ∗ a


(Existence of identity elements of G).
The symbols +, ·, o, ∗, etc., are used to denote binary operations on a set. Thus
(iv) For each a ∈ G, there exist an elements a−1 ∈ G such that a ∗ a−1 = e =
+ will be a binary operation on S if and only if a + b ∈ S for all a, b ∈ S and a + b a−1 ∗ a (Existence of inverese elements of G).
1. UNIT : FUNDAMENTALS OF GROUPS 5 1. UNIT : FUNDAMENTALS OF GROUPS 6

Example 1.3. The set of integers I, rationals Q, real numbers R are groups under Example 1.4. The set of all m × n matrices with their elements as integers

addition, where as natural numbers N is not a group under addition. Further, Q (rational, real or complex numbers) is an infinite abelian group under addition of

and R are groups under multiplication, where as I is not a group under multipli- matrices. Further, the set of all n × n non-singular matrices having their elements

cation. as rational (real or complex numbers) is an infinite non-abelian group under matrix
We now given some elementary properties of groups. multiplication.

Note 1.2. In view of associativity, given three elements a, b, c in a group G, their Definition 1.5. The order of an elelment a of a group G is the least positive
product abc is defined as abc = a(bc) = (ab)c. More generally, for a1 , a2 , . . . , an ∈ integer m such that am = e, where e is an identity of G. We write o(a) = m.
G, by indiction, the product a1 a2 · · · an is well defined. In particular, for any If there does not exist any integer m such that am = e, then an element a is of
a ∈ G, n an integer, we get infinite order.
(i) an = aa . . . a(n factors), for n > 0;
Example 1.5. In the multiplicative group G = {1, −1, i, −i} (i.e., forth root
(ii) a0 = e.
unity), the o(1) = 1(∵ 11 = 1) , o(−1) = 2(∵ (−1)2 = 1), o(i) = 4(∵ (i2 )2 = 4)
(iii) an = (a−1 )m , if n = −m, m > 0.
and o(−i) = 4(∵ (−i)4 = 4).
Property 1.1. Let G be a group. Then
Note 1.3. If the group operation is addition (+), then am means ma.
(i) Identity element of G is unique.
The following laws of exponents hold in a group G.
(ii) Every a ∈ G has unique inverse in G.
(iii) For every a ∈ G, (a−1 )−1 = a. Property 1.3. Let G be a group. Then

(iv) For every a, b ∈ G, (a · b) −1


=b −1 −1
·a . (i) an am = an+m , for a ∈ G and m, n integers.

(ii) (an )m = anm , for a ∈ G and m, n integers.


Property 1.2. Let G be a group and a, b, c ∈ G. Then
(iii) (ab)n = an bn , for a, b ∈ G and n integers, provided ab = ba.
(i) ab = ac implies b = c (Left cancellation law).
(ii) ba = ca implies b = c (Right cancellation law). Note 1.4. We say that a, b ∈ G commute if ab = ba. The integral powers of an

element in G always commute with each other. For an am = an+m = am+n = am an ,


Definition 1.4. A group (G, ∗) (or, simply G) is said to be an abelian group
for any integers m, n.
(or, a commutative group) if a ∗ b = b ∗ a for all a, b ∈ G. A group is said to be
In view of above properties, we have
non-abelian if it is not abelian. Further, a group G is said to be finite or infinite

according as the number of elements in G, called an order of G, and denoted by Property 1.4. Let G be a group with a, b ∈ G. Then G is abelian, provided;

o(G), finite or infinite, respectively. (i) if every element of G has its own inverse, or equivalently a2 = e.

1. UNIT : FUNDAMENTALS OF GROUPS 7 1. UNIT : FUNDAMENTALS OF GROUPS 8

(ii) if (ab)2 = a2 b2 . (i) HK is a subgroup of G if and only if HK = KH.


(ii) (HK)−1 = H −1 K −1 .
Note 1.5. Let G be a finite group containing even number of elements. Then

there exists an element a ∈ G, a ̸= e such that a2 = e(i.e., a = a−1 ). Further, odd Note 1.7.
(i) Every sugbroup of a cyclic group is cyclic.
number of such elements exist. (This fact follow as a consequence of Lagrange’s
(ii) Every proper subgroup of an infinite cyclic group is infinite.
theorem and Sylow theorems, which will be discussed later)
Definition 1.8. Let H be any subgroup of a group G and a be any element of G.
Definition 1.6. A group G is said to be cyclic group generated by a ∈ G if
The the set, Ha = {ha : h ∈ H} is called a right coset of H in G generated by
every element of G is an integral power a. A cyclic group generated by a ∈ G is
a and the set aH = {ah : h ∈ H} is called a left coset of H in G generated by a
denoted by ⟨a⟩. Therefore G = ⟨a⟩ = {an /n ∈ Z}.
with respect to multiplicative binary operation.
Example 1.6. Similarly, H +a = {h+a : h ∈ H} is a right coset and a+H = {a+h : h ∈ H}
1. The additive group of integers (Z, +) is a cyclic group generated by 1. is a left coset of H with respect to additive binary operation.
2. The group G = {2n /n ∈ Z} is a cyclic group with 2 as a generator, we know
 −n Note 1.8.
1 1
that 2n = , where n ∈ Z. Therefore is also a generator. (i) The cosets are also known as residue classes modulo the subroup.
2 2
(ii) If e is an identity element of G, then He = H = eH. Hence H itself is a
Note 1.6. Every cyclic group is an abelian group. But every abelian group is not
right as well as left coset. Since ea ∈ Ha, we have a ∈ Ha and therefore
cyclic group. Ha ̸= ∅. Therefore no coset can be empty.
(iii) If a, b are any two elements of a group G and H is a subgroup of G, then
Definition 1.7. Let G be a group and H be a non-empty subset of G. Then H is
1. Ha = H ⇔ a ∈ H.
said to be a subgroup of G if and only if H is agroup under the group operation 2. Ha = Hb ⇔ ab−1 ∈ H and aH = Ha ⇔ a−1 b ∈ H (for multiplicative
G. In any group G with an identity e, the set {e} and G are subgroups of G, called binary operation).
3. H + a = H + b ⇔ a − b and a + H = b + H ⇔ b − a ∈ H (for additive
trivial subgroups. The non-trivial subgroups are called proper subgroups. binary operation).
The following properties of subgroup of a group holds. (iv) Any two left (or right) cosets of a subgroup have the same number of ele-
ments.
Property 1.5. A non-empty subst H is a subgroup of G if and only if
Definition 1.9. The number of distinct left cosets of a subgroup H of a group G
(i) a, b ∈ H ⇒ ab−1 ∈ H (i.e., a, b ∈ H ⇒ ab ∈ H and b−1 = b ).
is equal to the number of distinct right cosets of H in G. The number of distinct
(ii) H −1 = H and H 2 = H.
cosets (left or right) of a subgroup H in a group G is called an index of a

Property 1.6. Let H and K be any two subgroups of a group G. Then subgroup H of a group G and is denoted by [G : H] or IG (H) or simply i(H).
1. UNIT : FUNDAMENTALS OF GROUPS 9 1. UNIT : FUNDAMENTALS OF GROUPS 10

Theorem 1.1. (Lagrange) Let H be a subgroup of a finite group G. Then the (i) We have g ∈ G ⇒ g −1 ∈ G. Therefore, N is a normal subgroup of G if and
only if g −1 n(g −1 )−1 , i.e., g −1 ng ∈ N ∀ g ∈ G and n ∈ N .
order of H divides the order of G.
(ii) Note that, the identity subgroup {e} and G itself are normal subgroup of G.
Proof. Since G is a finite group, the number of left cosets of H in G is finite.
Lemma 1.1. The normal subgroup N of a group G if and only if gN g −1 = N for
Denote the distinct left cosets of H by a1 (H) = H, a2 (H) = H, . . . , ak (H) = H.
every g ∈ G, where gN g −1 = {gng −1 : g ∈ G, n ∈ N }.
Since the group G is the union of all left cosets of H in G. We have G = a1 (H) ∪

a2 (H)∪. . . , ∪ak (H) and the cosets ai (H) are mutually disjoint. Also, since any two Proof. Let gN g −1 = N ∀ g ∈ G, where gN g −1 ⊂ N for all g ∈ G. Then by
left cosets of H in G have the same cardinality and H itself is a left coset, it follows the definition of normal subgroup, we have N is a normal subgroup of a group G.
that every left coset contains o(H) number of elements. Thus o(G) = k.o(H) and Conversely, let N be a normal subgroup of a group G. Then
sothe order of H divides the order of G.
gN g −1 ⊂ N f orall g ∈ G −→ (1)
Corollary 1.1. Let G be a finite group and let a ∈ G. The o(a) is a divisor of

o(G). Consequently ao(a) = e, we have ao(G) = e.


and also g ∈ G ⇒ g −1 ∈ G.

Proof. The order of the cyclic subgroup ⟨a⟩ generated by a is equal to o(a) and Therefore, we have

so by Lagrange’s theorem, o(a) divides o(G). Since ao(G) = e.


g −1 N (g −1 )−1 ⊂ N f orall g ∈ G
Corollary 1.2. A group G of prime order is cyclic and any non-trivial element
g −1 N g ⊂ N f orall g ∈ G
of G is a generator for G.
g(g −1 N g)g −1 ⊂ gN g −1 f orall g ∈ G
Proof. Let G be a group of order p, where p is a prime. If a ∈ G, a ̸= e, then o(a)
N ⊂ gN g −1 f orall g ∈ G −→ (2).
divides p by above corollory. Since p is prime and a ̸= e and o(G) = p. Hence

G = ⟨a⟩, i.e., G is cyclic. From equations (1) and (2), we have gN g −1 = N f orall g ∈ G.

Definition 1.10. A subgroup N of G is said to be a normal subgroup of G if Lemma 1.2. The subgroup N is a normal subgroup of a group G if and only if
for every g ∈ G and n ∈ N , gng −1 ∈ N . every left coset of N in G is a right coset of N in G.
From this definition, we can immediately conclude that the subgroup N is a
normal subgroup of a group G if and only if gN g −1 = N ∀ g ∈ G. The normal
subgroup N of a group G is denoted as N △ G.

Note 1.9.

1. UNIT : FUNDAMENTALS OF GROUPS 11 1. UNIT : FUNDAMENTALS OF GROUPS 12

Proof. Let N be a normal subgroup of G. Then Therefore the product of two right cosets is again a right coset. Conversely, sup-

pose N aN b = N ab ∀ a, b ∈ G,
gN g −1 = N f orall g ∈ G

(gN g −1 )g = N g f orall g ∈ G (N aN b)(ab)−1 = (N ab)(ab)−1

gN = N g f orall g ∈ G. (N aN a)b−1 a−1 = N

Therefore, each left coset gN is the right coset N g in G. Conversely, suppose Therefore (n1 an2 b)b−1 a−1 ∈ N .

that left coset gN is the right coset N g in G. Let g be any element of G then Since, N aN b = {(n1 a)(n2 b) : n1 a ∈ N a, n2 b ∈ N b}. Hence

gN = N g ′ for some g ′ ∈ G. Since e ∈ N and ge = g ∈ gN . Therefore g ∈ N g 1


n1 an2 a−1 ∈ N
[∵ N g 1 = gN ], but g ∈ N g 1 ⇒ N g = N g 1 . Hence N g = gN [∴ N g 1 = gN ]. Thus
an2 a−1 ∈ n−1
1 N = N

gN = N g ∀ g ∈ G an2 a−1 ∈ N [∵ n11 ∈ N ].


−1 −1
gN g = N gg ∀ g∈G
Therefore N is a normal subgroup of a group G.
gN g −1 = N ∀ g ∈ G.

Note 1.10. Let G be a group and N be a normal subgroup of G. We know that


∴ N is a normal subgroup of a group G if and only if gN = N g ∀ g ∈ G.
N a = aN f orall a ∈ G, i.e., the right and left cosets of N determined by any

Lemma 1.3. A subgroup N of G is a normal subgroup of G if and only if the a ∈ G are equal. So there is no need to distinguish between left and right cosets of

product of two right cosets of N in G. a normal subgroup N . We shall say that N a is a coset of N . Let G/N denote the

set of all cosets of N in G.


Proof. Let N be a normal subgroup of G, then N a and N b are two right cosets of

N in G. We have Theorem 1.2. If G is a group and N is a normal subgroup of a group G, then the

set G/N = {N a/a ∈ G} of cosets is a group under the multiplication of cosets.


(N a)(N b) = N (aN )b
Proof. Let G be a group and N be a normal subgroup of a group G. If G/N =
= N (N a)b [∵ N a = aN ∀ a ∈ G]
{N a/a ∈ G}, then N aN b = N ab.
= N N ab Here, the closure property is obvious as a, b ∈ G implies ab ∈ G and hence
(N a)(N b) = N ab [∵ N is a subgroup of G ⇒ N N = N ] N ab ∈ G/N .
1. UNIT : FUNDAMENTALS OF GROUPS 13 1. UNIT : FUNDAMENTALS OF GROUPS 14

(N a)(N bN c) = N a(N bc) = N a(bc) = N (ab)c = N abN c = (N aN b)N c. So, factor group along with basic properties and examples. Also, the significance of
Lagrange’s theorem and its converese are revealed.
associative property holds.

For the identity element e ∈ G, we have N e or N in G/N satisfying N aN e = KEYWORDS

N ae = N a. Similarly, N eN a = N a. Algebraic structures, Group, Abelian group, Cyclic group, Subgroup, Normal sub-
group, Quotient group.
Therefore, there exists N a−1 in G/N , such that
ASSESSMENT / TERMINAL QUESTIONS

N aN a −1
= N N aa −1
[∵ aN = N a] 1. Show that intersection of two subgroups of G is a subgroup of a group G.

= N e, the identity of G/N. 2. Show that a subgroup of a cyclic group is cyclic.

3. Let H and K are two subgroup of a group G. Then show that HK is a


subgroup of G if and only if HK = KH.
Therefore N a−1 is the inverse of N a. Hence G/N is a group.
4. Let H and K are two finite subgroup of a group G. Then show that
Definition 1.11. If G is a group and N is a normal subgroup of a group G, then
o(H)o(K)
the set G/N of all cosets of N in G is a subgroup with respect to multiplication of o(HK) = .
o(H ∩ K)
cosets. It is called the Quotient group or Factor group of G by N .
5. Explain the converse of Lagrange’s theorem with suitable example.
The identity element of the quotient group G/N is N .
6. Show that that a subgroup of index 2 in a group G is a normal subgroup of
Note 1.11. a group G.

o(G) 7. Show that intersection of two normal subgroups of G is a subgroup of G.


(i) o(G/N ) = iN (G) = [N : G] = .
o(N )
8. Show that every subgroup of an abelian group is normal. Justify the converse
(ii) Every quotient group of an abelian group is abelian (but the converese is with suitable example.
not true). [Hint: Consider Quaternion group (i.e., the quaternions are a number sys-
(iii) Every quotient group of a cyclic group is cyclic (but the converese is not tem in the form a + bi + cj + dk. The quaternions 1, i, j, k form a non-abelian
true). group of order eight called quaternion group and is denoted by Q8 ). It is
also known as a Hamiltonian quaternions.]

SUMMARY
REFERENCES
Group theory is the study of groups. Groups are sets equipped with an binary
1. I. N. Herstein: Topics in Algebra, 2nd Edition, John Wiley and Sons, 2008.
composition (like addition, multiplication) that satisfies certain basic axoims. As
the building blocks of abstract algebra, groups are so general and fundamental 2. Surjeet Singh and Qazi Zameeruddin: Modern Algebra, 8th Edition, Vikas
that they arise in nearly every branch of mathematics and the sciences. This Publishing House, 2021.
unit occupy a very essential and fundamental aspect of groups, subgroups, cyclic
group, nornmal subgroup (very special class of subgroups), quotient group or 3. N. Jacobson: Basic Algebra-I, 2nd Edition, Dover Publications, 2009.

1. UNIT : FUNDAMENTALS OF GROUPS 15 16

4. M. Artin : Algebra, 2nd Edition, Prentice Hall of India, 2015. 2 UNIT : Group Homomorphism
5. Darek F. Holt, Bettina Eick and Eamonaa A. O’brien: Handbook of com-
putational group theory, 1st Edition, Chapman & Hall/CRC Press, 2005.

6. J. B. Fraleigh : A first course in abstract algebra, 7th Edition, Addison- OBJECTIVES


Wesley Longman, 2002.
After studying this unit you will be able to:
7. D. S. Dummitv and R.M. Foote: Abstract Algebra, 2nd Edition, John Wiley
& Sons, 2003.
• identify whether a function between groups is a homomorphism or not;
8. J. A. Gallian: Contemporary Abstract algebra, 9th Edition, Narosa Pub-
lishing House. 2008. • explain the concepts of kernel and image of any homomorphism;

• illustrate with examples of kernel and image of any homomorphism;

• describe the different types of homomorphism;

• explain the different types of homomorphism with suitable examples.

• state, prove and apply the Fundamental theorem of homomorphism for

groups;

• extend the concepts of Fundamental theorem of homomorphism for groups

in other algebraic structures.

INTRODUCTION

The group homomorphism deals with functions from one group to another group.

The various properties of these functions between the groups, which preserve the

algebraic structures of their domain groups. These fuctions are called group ho-

momorphisms. The term homomorphism was initiated by Klein in 1893.

Definition 2.1. Homomorphism : Let (G, o) and (G, ∗) be two groups. A


mapping ϕ : G → G is said to be homomorphism if ϕ(aob) = ϕ(a)∗ϕ(b), ∀ a, b ∈
G or simply, ϕ(ab) = ϕ(a)ϕ(b), ∀ a, b ∈ G
2. UNIT : GROUP HOMOMORPHISM 17 2. UNIT : GROUP HOMOMORPHISM 18

Types of homomorphism: A homomorphism ϕ : G → G is said to be a/an 4. Let G = (R+ , ·) and (R, +). Define ϕ : G → G by ϕ(x) = log10 x, ∀ x ∈ G.

Then ϕ(xy) = log10 xy = log10 x+log10 y = ϕ(x)+ϕ(y), ∀ x, y ∈ G. Therefore


(i) Monomorphism if it is one - one function.
ϕ is a homomorphism. The kernel of homomorphism ϕ is Kφ = {x ∈ G |
(ii) Epimorphism if it is onto function. ϕ(x) = 0} = {x ∈ G | log10 1 = 0} = {1}.

(iii) Isomorphism if ϕ is both one-one and onto function. Two groups G and 5. Let G = (Z, +) and G = (Z, (+)n ). Define ϕ : G → G by ϕ(a) = r, where r
G are said to be “isomorphism”, if there exists an isomorphism ϕ : G → G, is the remainder obtained when a is divided by n.
and is denoted by G ≡ G.
Let a and b be any two integers such that a ≡ r1 (modn) and b ≡ r2 (modn).

(iv) Endomorphism if G = G. Then ϕ(a) = r1 and ϕ(b) = r2 . Therefore, ϕ(a) + ϕ(b) = r3 −→ (1)

(v) Automorphism if ϕ is both one-one and onto with G = G. Since a + b ≡ r1 + r2 (modn) ≡ r3 (modn), we have

ϕ(a + b) = r3 −→ (2)
Definition 2.2. Let ϕ be a homomorphism of G into G. Then the kernel of ϕ is
defined by Kerϕ = Kφ = {x ∈ G : ϕ(x) = e, the identity of G}. From equations (1) and (2), we have ϕ(a + b) = ϕ(a) + ϕ(b). Therefore ϕ is

a homomorphism. The kernel of homomorphism ϕ is Kφ = {a ∈ G : ϕ(a) =


Example 2.1.
0} = {a ∈ G : a = nk, K ∈ Z} = nZ.
1. Let G be a group and G = G. Define ϕ : G → G by ϕ(x) = e for all x ∈ G.
Note 2.1. In the above examples, the homomorprism, in (2), (3), (4) are all
Then ϕ(xy) = e = e.e = ϕ(x)ϕ(y) ∀ x ∈ G. Therefore, ϕ is a homomorphism.
isomorphism, the homomorphism, in (1) and (5) are not isomorphism.
The kernel of homomorphism ϕ is Kφ = {x ∈ G | ϕ(x) = e} = G.
Definition 2.3. If ϕ is a homomorphism of G into G, then the image of ϕ is
2. Let G be a group and G = G. Define ϕ : G → G by ϕ(x) = x ∀ x ∈ G. denoted by Im(ϕ) is defined by
Then ϕ(xy) = xy = ϕ(x)ϕ(y), ∀ x, y ∈ G. Therefore, ϕ is a homomorphism.

The kernel of homomorphism ϕ is Kφ = {x ∈ G : ϕ(x) = e} = {x ∈ G : x = Im(ϕ) = {y ∈ G/y = ϕ(x), x ∈ G}

e} = {e}. = {ϕ(x)/x ∈ G}

3. Let G = (R, +) and G = (R−{0}, ·). Define ϕ : G → G by ϕ(x) = 2x ∀x ∈ G, Property 2.1. Let ϕ be a homomorphism of G into G. Then
then ϕ(x + y) = 2x+y = 2x 2y = ϕ(x)ϕ(y), ∀ x, y ∈ G. Therefore, ϕ is a
(i) ϕ(e) = e, the identity element of G.
homomorphism. The kernel of homomorphism ϕ is Kφ = {x ∈ G : ϕ(x) =

1} = {x ∈ G : 2x = 1} = {0}. (ii) ϕ(x−1 ) = [ϕ(x)]−1 , f or all x ∈ G.

2. UNIT : GROUP HOMOMORPHISM 19 2. UNIT : GROUP HOMOMORPHISM 20

Proof. To prove the normality of K, we have to show that gkg −1 = e, whenever


ϕ(k) = e and k ∈ K. We consider,
(i) For any a ∈ G, we have ϕ(a)e = ϕ(a) = ϕ(ae) = ϕ(a)ϕ(e). Therefore, by
the cancellation law in G, we have ϕ(e) = e.
ϕ(gkg −1 ) = ϕ(g)ϕ(k)ϕ(g −1 )
(ii) For x ∈ G, ϕ(x)[ϕ(x)]−1 = e = ϕ(e) = ϕ(xx) = ϕ(x)ϕ(x). Again, by the = ϕ(g)e[ϕ(g)]−1 , by the properties of image of homomorphism.
cancellation law in G, we have ϕ(x−1 ) = [ϕ(x)]−1 .
= ϕ(g)[ϕ(G)]−1
−1
ϕ(gkg ) = e.

Property 2.2. Let ϕ be a homomorphism of a group G into G. Then Im(ϕ) is a


This shows that gkg −1 ∈ K and hence K is a normal subgroup of G.
subgroup of G.

Proof. Let x, y ∈ G and ϕ(x), ϕ(y) ∈ G. Then Im(ϕ) = {ϕ(x) : x ∈ G} = {x ∈ Note 2.2. We call ϕ−1 is the inverse map to ϕ.

G : x = ϕ(x), x ∈ G}. Similarly, Im(ϕ) = {ϕ(y)/y ∈ G} = {y/y = ϕ(y), y ∈ G}. Lemma 2.3. If ϕ is a homomorphism of G onto G with kernel K then the set of
Therefore, ϕ(xy −1 ) = e, where e is an identity of G. Therefore ϕ(x).ϕ(y −1 ) = e all inverse images g ∈ G under ϕ in G is given by Kx, where x is any particular
implies ϕ(y) = [ϕ(x)]−1 . Therefore an inverse exist. Since ϕ(x), ϕ(y) ∈ G. Hence inverse image of g in G.
ϕ(xy −1 ) = ϕ(x).ϕ(y) ∈ G. Thus Im(ϕ) is a subgroup of G.
Proof. Let g ∈ G and ϕ(g) = g ∈ G with e, e be an identities of G and G,
Lemma 2.1. Suppose G is a group and N is a normal subgroup of G. Define the
respectively. If {ϕ−1 (g)} = {x ∈ G : ϕ(x) = g}, then to prove that {ϕ−1 (g)} = Kx
mapping ϕ from G to G/N by ϕ(x) = N x ∀ x ∈ G. then ϕ is a homomorphism
whenever x ∈ ϕ−1 (g). The result is obvious, when g = e, because {ϕ−1 (e)} =
of G onto G/N . In otherwords, let G be a group and N , a normal subgroup of G.
Ke = K.
Then G/N is a homomorphic image of G.
Suppose g ̸= e and x ∈ ϕ−1 (g). We now verify that ϕ−1 (g) = Kx.
Proof. Consider the mapping ϕ : G → G/N , defined by ϕ(x) = N x ∀ x ∈ G. Let For, let y ∈ Kx so that y = kx for k ∈ K implies ϕ(k) = e. Then ϕ(y) =
N x ∈ G/N . Then x ∈ G, we have ϕ(x) = N x. Therefore ϕ is onto G/N . ϕ(kx) = ϕ(k)ϕ(x) = eϕ(x) = eg = g. Therefore, y ∈ ϕ−1 (g). Hence y ∈ Kx
Let x, y ∈ G. Then ϕ(xy) = N xy = N xN y = ϕ(x)ϕ(y). Therefore ϕ is implies y ∈ ϕ−1 (g). Thus Kx ⊆ ϕ−1 (g).
homomorphism. On the otherhand, if z ∈ ϕ−1 (g), then ϕ(z) = g. Therefore ϕ(z) = g = ϕ(x) or

Lemma 2.2. If ϕ : G → G is a homomorphism with kernel K, then K is a normal ϕ(z) = ϕ(x) implies ϕ(z)[ϕ(x)]−1 = e (i.e., ϕ(z)[ϕ(x−1 )] = e). Thus ϕ(zx−1 ) = e.

subgroup of a group G. Therefore zx−1 ∈ K or z ∈ Kx. Also z ∈ ϕ−1 (g) implies z ∈ Kx. Thus ϕ−1 (g) ⊆
Kx. Hence {ϕ−1 (g)} = Kx, where x ∈ ϕ−1 (g). Thus the result follows.
Proof. If x, y ∈ K, then ϕ(x) = e, where e is an identity element of G. Therefore
ϕ(xy) = ϕ(x)ϕ(y) = e.e = e (i.e., xy ∈ K). Also, if x ∈ K, then ϕ(x) = e, and so Corollary 2.1. A homomorphism of ϕ : G → G with kernel K is an isomorphism
ϕ(x−1 ) = [ϕ(x)]−1 = (e)−1 = e. Hence x−1 ∈ K. Therefore K is a subgroup of G. if and only if kerϕ = {e}. In otherwords, the necessary and sufficient condition
2. UNIT : GROUP HOMOMORPHISM 21 2. UNIT : GROUP HOMOMORPHISM 22

for a homomorphism ϕ of a group G into a group G with kernel K to be an Therefore


isomorphism on G into G is that K = {e}.
ϕ(a) = ϕ(kb)
Proof. Let ϕ : G → G be a homomorphism with kernel K, and e, e be the identities
= ϕ(k)ϕ(b)
of G, G, respectively.
= eϕ(b) {∵ k ∈ K, the kernel of ϕ}
Suppose ϕ is an isomorphism of G into G. Then ϕ is one-one. Let x ∈ K. Then
ϕ(x) = e =⇒ ϕ(x) = ϕ(e)[∵ ϕ(e) = e] =⇒ x = e. Thus x ∈ K =⇒ x = e. = ϕ(b)

In otherwords, an identity e is the only element of G which belongs to K. ϕ(a) = ϕ(b)


Therefore K = {e}. ψ(Ka) = ψ(Kb)
Conversely, suppose that K = {e}. Then to prove that ϕ is an isomorphism of
G into G, i.e., to prove that ϕ is one-one. Therefore ψ is well defined.

If x, y ∈ G, then ϕ(x) = ϕ(y). Therefore ϕ(x)[ϕ(y)]−1 = e =⇒ ϕ(x)ϕ(y −1 ) = Next, let Ka and Kb be any two elements of G/K. Then we have

e =⇒ ϕ(xy −1 ) = e. Thus xy −1 ∈ K. Also, xy −1 = e because k = {e} =⇒


ψ(KaKb) = ψ(Kab)
xy −1 y = ey =⇒ x = y. Therefore ϕ is one-one. Hence ϕ is an isomorphism of G
into G. = ϕ(ab)

= ϕ(a)ϕ(b)
Prove our next results, we make use of the following note.
ψ(KaKb) = ψ(Ka)ψ(Kb)
Note 2.3. If a, b are any two element of G and H be any subgroup of G, then
a ∈ Hb =⇒ Ha = Hb and a ∈ bH =⇒ aH = bH. Therefore ψ is a homomorphism.
Let g be any arbitrary element of G, since ϕ is onto, there exists an element
Fundamental Tuheorem of Homomorphism (FTH) for groups:
g ∈ G such that ϕ(g) = g. But then ψ(Kg) = ϕ(g) = g, where Kg ∈ G/K.
Theorem 2.1. Let ϕ be a homomorphism of G onto G with kernel K. Then G/K Therefore ψ is onto. Finally, the kernel of ψ is given by
is isomorphic with G ( i. e., G/K ≈ G).
Kψ = {Kg ∈ G/K : ψ(Kg) = e, the identity element of G}
Proof. Define ψ : G/K → G by ψ(Kg) = ϕ(g), f or all Kg ∈ G/K.
= {Kg ∈ G/K : ϕ(g) = e}
First, let Ka = Kb for some Ka and Kb in G/K. Then a = kb for some k ∈ K.
= {Kg ∈ G/K : g ∈ K}

Kψ = {K}.

Thus the kernel Kψ contains identity alone. Therefore ψ is one-one. Hence,

2. UNIT : GROUP HOMOMORPHISM 23 2. UNIT : GROUP HOMOMORPHISM 24

G/K ≈ G. 1. If ϕ : G → G is a homomorphism, then show that ϕ(xn ) = [ϕ(x)]n , where n

is an integer.
Note 2.4. In this above theorem, we may take ϕ(G), the homomorphic image of
G, for G. That is every homomorphic image of a group G is isomorphic to some Z Z
2. Find all the homomorphisms from to .
4Z 6Z
quotient group of G. In otherwords, if ϕ : G → G is a homomorphism, then
G/Kerϕ ≈ Im(ϕ). 3. Let ϕ : G → G be a homomorphism. Let x ∈ G be such that o(x) = n

and o(ϕ(x)) = m. Show taht o(ϕ(x))|o(x) and ϕ is one to one if and only if
Corollary 2.2. If ϕ : G → G is an epimorphism, then G/Kerϕ ≈ G.
m = n.
Proof. The proof of the above corollory is on the same lines of FTH.
4. Let ϕ : G → G be a homomorphism. Then show that ϕ(G) = {ϕ(x) : x ∈ G}

is a subgroup of G. Further, show that if H is a normal subgroup of G, then


SUMMARY ϕ(H) is a normal subgroup of ϕ(G). [Hint: ϕ(G) = Im(ϕ)].

A group homomorphism is a map between two groups such that the group oper- 5. Let f, g be homomorphism from G → G. Then show that H = {x ∈ G :
ation is preserved: for all, where the product on the left-hand side is in and on f (x) = g(x)} is a subgroup of G.
the right-hand side in. Thus, homomorphisms map the unique identity element
6. Show that homomorphic image of an ableian group is abelian. Justify the
of one group to the unique identity element of the other group. Similarly, homo-
converse with suitable example.
morphisms map the inverse of an element x in one group to the inverse of the

element ϕ(x). This is why homomorphisms are called structure-preserving maps. 7. Show that homomorphic image of a cyclic group is cyclic. Justify the con-

Particularly, the fundamental theorem on homomorphisms, relates the structure verse with suitable example.

of two objects between which a homomorphism is given, and of the kernel and
8. Show that homomorphic image of a finite group is finite. Justify the converse
image of the homomorphism.
with suitable example.

REFERENCES
KEYWORDS

1. I. N. Herstein: Topics in Algebra, 2nd Edition, John Wiley and Sons, 2008.
Homomorphism, Types of homomorphisms, Kernel, Image, Fundamental theorem

of homomorphisms. 2. Surjeet Singh and Qazi Zameeruddin: Modern Algebra, 8th Edition, Vikas

Publishing House, 2021.


ASSESSMENT /TERMINAL QUESTIONS
3. N. Jacobson: Basic Algebra-I, 2nd Edition, Dover Publications, 2009.
2. UNIT : GROUP HOMOMORPHISM 25 26

4. M. Artin : Algebra, 2nd Edition, Prentice Hall of India, 2015. 3 UNIT : Isomorphism and Automorphism The-
5. Darek F. Holt, Bettina Eick and Eamonaa A. O’brien: Handbook of com- orems
putational group theory, 1st Edition, Chapman & Hall/CRC Press, 2005.

6. J. B. Fraleigh : A first course in abstract algebra, 7th Edition, Addison- OBJECTIVES

Wesley Longman, 2002. After studying this unit you will be able to:

7. D. S. Dummitv and R.M. Foote: Abstract Algebra, 2nd Edition, John Wiley • define an isomorphism group with examples;

& Sons, 2003. • explain automorphism group and their elementary properties;

8. J. A. Gallian: Contemporary Abstract algebra, 9th Edition, Narosa Pub- • define the group of automorphism and inner automorphism along with prop-
lishing House. 2008. erties ;

• test the given homomorphism is an isomorphism or not;

• verify the given homomorphism is an automorphism/inner automorphism or


not.

• explain the concepts of Klein 4- group and their propeties;

• determine the elements of group of automorphism and inner automorphisms


of Klein 4- group.

INTRODUCTION

An isomorphism is a special type of homomorphism. The Greek roots “homo”


and morph” together mean “same shape.” There are two situations where homo-
morphisms arise: when one group is a subgroup of another; when one group is a
quotient of another. The corresponding homomorphisms are called embeddings
and quotient maps. The critical difference between an isomorphism and an auto-
morphism is just the range: it’s equal to the domain in the case of automorphisms,
and not in the case of isomorphisms.

3. UNIT : ISOMORPHISM AND AUTOMORPHISM THEOREMS27 3. UNIT : ISOMORPHISM AND AUTOMORPHISM THEOREMS28

Isomorphism Theorems: Finally, the kernel of ψ is given by

Theorem 3.1. [First isomorphism theorem] Let ϕ be a homomorphism on


Kψ ={g ∈ G : ψ(g) = N }
G onto G with kernel K. Let N be a normal subgroup of G and N = {x ∈ G :
={g ∈ G : N ϕ(g) = N }
ϕ(x) ∈ N }, Then
={g ∈ G : ϕ(g) ∈ N }
(i) G/N ≈ G/N ,
=N.
G/K
(ii) G/N ≈ . (Freshman’s theorem)
G/K
Therefore, by the fundamental theorem of homomorphism for groups, it
Proof. follows that G/N ≈ G/N .

(i) We know that N is a normal subgroup of G. Define ψ : G → G/N by G/K


(ii) Define θ : G → , by θ(g) = (N/K)Kg, f orall g ∈ G.
N/K
ψ(g) = N ϕ(g), f orall g ∈ G. If a = b, where a, b ∈ G, then ϕ(a) = ϕ(b).
If a, b ∈ G and a = b, then Ka = Kb. Therefore (N/K)Ka = (N/K)Kb =⇒
Therefore N ϕ(a) = N ϕ(b). θ(a) = θ(b).
∴ ψ(a) = ψ(b) Therefor, θ is well defined.
Therefore ψ is well defined. Let a and b be any two elements of G. Then
Let a and b be any two elements of G. Then
θ(ab) =(N/K)Kab
ψ(ab) =N ϕ(ab) =(N/K)KaKb
=N ϕ(a)ϕ(b) since ϕ is a homomorphism =(N/K)Ka(N/K)Kb
=N ϕ(a)N ϕ(b) since N is normal in G =θ(a)θ(b)
=ψ(a)ψ(b).
Therefore θ is a homomorphism.
Therefore ψ is a homomorphism. G/K
Next, let (N/K)Kg be any element of . Then g ∈ G is such that
N/K
Next, let N g be an arbitrary element of G/N . Then, g ∈ G, since ϕ is θ(g) = (N/K)Kg.
onto, there exists an element g ∈ G such that ϕ(g) = g. But then we have,
Therefore θ is onto.
N g = N ϕ(g) = ψ(g).

Therefore ψ is onto.
3. UNIT : ISOMORPHISM AND AUTOMORPHISM THEOREMS29 3. UNIT : ISOMORPHISM AND AUTOMORPHISM THEOREMS30

Finally, the kernel of θ is given by Consider

Kθ ={g ∈ G : θ(g) = N/K} n1 m1 n2 m2 =n1 (m1 n2 )m2

={g ∈ G : (N/K)Kg = (N/K)} =n1 n2 m1 m2 , [∴ m1 n2 = n2 m1 ]

={g ∈ G : Kg ∈ N/K} ϕ(n1 m1 n2 m2 ) =ϕ(n1 n2 m1 m2 )

={g ∈ G : g ∈ N } =(N ∩ M )n1 n2

=N =(n ∩ M )n1 (N ∩ M )n2 , [∴ N ∩ M isnormal]

=ϕ(n1 m1 )ϕ(n2 m2 )
Therefore, by the fundamental theorem of homomorphism for groups, if
G/K
follows that G/N ≈ . Hence the theorem. Therefore ϕ is a homomorphism.
N/K
Let (N ∩ M )n be any element of N/N ∩ M . Then n ∈ M .
Therefore ne ∈ N M, [∴ e ∈ M ].
Theorem 3.2. [Second Isomorphism Theorem] If N and M are normal Therefore ϕ(ne) = (N ∩ M )n
subgroups of a group G. Then Therefore ϕ is onto.
Finally, the kernel of ϕ is given by
N M/M ≈ N/N ∩ M.

Kφ ={nm ∈ N M : ϕ(nm) = N ∩ M }
Proof. First, we observe that N M and N ∩ M are normal subgroup of G.
={nm ∈ N M : (N ∩ M )n = N ∩ M }
Define ϕ : N M → N/N ∩ M by ϕ(nm) = (N ∩ M )n for all n ∈ N .
={nm ∈ N M : n ∈ N ∩ M }
Let n1 m1 and n2 m2 be any two elements of N M with n1 m1 = n2 m2 .
Then n1 = n2 m2 m−1 ∈ n2 M , since m2 m−1 ∈ M = M n2 , since M is normal in ={nm ∈ N M : n ∈ M }
1 1

G, n1 = mn2 for some m ∈ M . =M


Therefore n1 n−1 −1
2 = m ∈ M . But n1 n2 ∈ N , since N ⊆ G.

Therefore n1 n−1 Therefore, by the fundamental theorem of homomorphism for groups, it follows
2 ∈ N ∩ M.

⇒ (N ∩ M )n1 n−1 that, N M/M ≈ N/N ∩ M . Hence the theorem.


2 = (N ∩ M )
o(N ) o(N M )
⇒ (N ∩ M )n1 = (N ∩ M )n2 . Note 3.1. o(N M/M ) ≈ o(N/N ∩ M ) or = .
o(N ∩ M ) o(M )
Therefore ϕ(n1 m1 ) = ϕ(n2 m2 ). Definition 3.1. An isomorphism of a group G onto itself is called an Automor-
Therefore ϕ is well defined. phism of G. i.e., T : G → G (onto and one-one) is an automorphism of G if
Next, let n1 m1 and n2 m2 be any two elements of N M . T (ab) = T (a)T (b) [or (ab)T = (aT )(bT )].

3. UNIT : ISOMORPHISM AND AUTOMORPHISM THEOREMS31 3. UNIT : ISOMORPHISM AND AUTOMORPHISM THEOREMS32

Example 3.1. Proof. Let T be an automorphism and N be a normal subgroup of a group G. If


T (N ) is a subgroup of G. Then x = T (y), where y ∈ G because T is function of
1. Let G be any group. Then the identity transformation I : G → G, defined
G onto G. Also k = T (n), where n ∈ N . We have,
by I(x) = x f orall x ∈ G is an automorphism of G.

2. Let G be any group. Then the additive group Z of integer T : Z → Z, defined xkx−1 =T (y)T (n)[T (y)]−1
by T (x) = −x f orall x ∈ Z, is an automorphism. =T (y)T (n)[T (y −1 )]

Lemma 3.1. A homomorphism T : G → G defined by T (x) = x−1 is an auto- =T (yny −1 ), Since N is normal in G

morphism if and only if G is an abelian.


Therefore yny −1 ∈ N .
Proof. Let G be any abelian group. Define T : G → G by T (x) = x−1 f orallx ∈ G. Consequently, T (yny −1 ) ∈ T (N ). Thus xkx−1 ∈ T (N ). Therefore T (N ) is a
Then T (x) = T (y) ⇒ x−1 = y −1 ⇒ x = y. Therefore T is one-one function . normal in G.
For any x ∈ G, there exist x−1 ∈ G such that T (x−1 ) = (x−1 )−1 = x. Therefore
Lemma 3.4. If G is a group, then the set A(G) of all automorphism of G is a
T is onto function.
group.
For any x, y ∈ G, we have T (xy) = (xy)−1 = y −1 x−1 = T (y)T (x) = T (x)T (y),
since G is an abelian. Therefore T is an isomorphism of G onto G and hence is Proof. Let A(G) be the collection of all automorphism of a group G. Then A(G) =
an automorphism. {T : T is an automorphism of G}.
Conversely, suppose T is an isomorphism. Then T (xy) = (xy)−1 = y −1 x−1 = For T1 , T2 ∈ A(G), we have (T1 ◦ T2 ) is a bijective map.
T (y)T (x) = T (yx). That is, T (xy) = T (yx). Hence xy = yx.
(i) To verify that it is a homomorphism.
Therefore G is an abelian group.
Consider for any x, y ∈ G,
Lemma 3.2. Let G be a group, H be a subgroup of G and T be an automorphism
of G. Let T (H) = {T (h) : h ∈ H}. Then prove that T (H) is a subgroup of G. (T1 ◦ T2 )(xy) =T1 [T2 (xy)]

Proof. Let G be a group, H be a subgroup of G and T be an automorphism of =T1 [T2 (x)T2 (y)]

G. For a, b ∈ T (H). we have h1 , h2 ∈ H such that a = T (h1 ); b = T (h2 ). But =[T1 (T2 (x))][T1 (T2 (y))]
T (h1 h−1 −1
2 ) ∈ T (H) ⇒ T (h1 )T (h2 ) ∈ T (H) ⇒ T (h1 )[T (h2 )]
−1
∈ T (H). That is, =[(T1 ◦ T2 )(x)][(T1 ◦ T2 )(y)]
ab−1 ∈ T (H). Therefore T (H) is a subgroup of G.
Therefore (T1 ◦ T2 ) is a homomorphism and hence (T1 ◦ T2 ) is an automor-
Lemma 3.3. Let G be a group, T be an automorphism of G and N be a normal
phism.
subgroup of G. Then prove that T (N ) is an normal subgroup of G.
Therefore T1 ◦ T2 ∈ A(G), so closure property holds in A(G).
3. UNIT : ISOMORPHISM AND AUTOMORPHISM THEOREMS33 3. UNIT : ISOMORPHISM AND AUTOMORPHISM THEOREMS34

(ii) For T1 , T2 , T3 ∈ A(G), we have [T1 ◦ T2 ] ◦ T3 = T1 ◦ [T2 ◦ T3 ]. Since, the Finally, If x ∈ G, then gxg −1 ∈ G and Tg (gxg −1 ) = g −1 (gxg −1 )g = x.
resultant composition for mapping is associative. Hence the composition in Therefore Tg is onto.
A(G) is also associative. Hence, Tg is an automorphism of G.

(iii) For any T ∈ A(G), there exists the identity map I ∈ A(G) such that Definition 3.3. If G is a group, the mapping Tg : G → G defined by Tg (x) =

T ◦ I = T = I ◦ T . Therefore I is the identity of A(G) g −1 xg f orall x ∈ G is an automorphism of G known as inner automorphism.

Note 3.2.
(iv) For any T ∈ A(G), there exists the inverse map T −1 ∈ A(G) such that
1
T ◦T = I = T −1
◦ T . Therefore T −1
is the inverse of A(G). Hence A(G) is 1. The group consisting of inner automorphism is called group of inner auto-

a group. morphism. It is denoted by Inn(G) (or., I(G)) i.e., Inn(G) = {Tg : g ∈ G}.

2. Inn(G) ⊂ Aut(G) (or I(G) ⊂ A(G)).

3. An automorphism which is not inner is called an outer automorphism. Also,


Group of automorphism or automorphism group: A(G)
the quotient group is called the group of all outer automorphism of G.
I(G)
Definition 3.2. Let G be a group. Then the set A(G) of all automorphism of G
4. A subgroup N of a group G is a normal subgroup if and only if gN g −1 =
forms a group under the composition of mapping with identity map I as the identity
N f orall g ∈ G. But Tg (N ) = gN g −1 . Therefore N is normal if and only if
and inverse of T as T −1 . This group is called the group of Automorphisms.
Tg (N ) = N f orall g ∈ G.
Inner automorphism: In other words, a subgroup N of a group G is a normal subgroup if and only
if it is invariant under all inner automorphism of G. Hence normal subgroups are
Theorem 3.3. Let G be a group, g ∈ G and Tg : G → G be defined by Tg (x) =
−1 some times called invariant subgroups.
g xg f orall x ∈ G. Then Tg is an automorphism of G.
Theorem 3.4. If G is a group, then the set Inn(G) of all inner automorphism
Proof. First, let x, y ∈ G. Then
of G is aslo a group.

Tg (xy) =g −1 xyg Proof. Let G be a group with Inn(G) = {Tg : g ∈ G}.


For Tg , Th ∈ Inn(G), we have
=(g −1 xg)(g −1 yg)

=Tg (x)Tg (y)


(Tg .Th )(x) =T g(Th (x))

=Tg (hxh−1 )g −1
Therefore Tg is a homomorphism.
−1
Next, let x, y ∈ G. Then Tg (x) = Tg (y) ⇒ g xg = g yg −1 =(gh)x(gh)−1

By cancellation laws in G, we get x = y. Therefore Tg is one-one. =Tgh (x)

3. UNIT : ISOMORPHISM AND AUTOMORPHISM THEOREMS35 3. UNIT : ISOMORPHISM AND AUTOMORPHISM THEOREMS36

Therefore Tg Th = Tgh ∈ Inn(G). Hence the closure property holds in Inn(G). G


Lemma 3.5. Inn(G) ≈ , where Inn(G) is the group of inner automor-
Z(G)
For Tg , Th and Tr ∈ Inn(G), we have, phisms of G, and Z = Z(G) is the centre of G.

(Tg Th )Tr =(Tgh )Tr Proof. Define ϕ : G → Inn(G), by ϕ(g) = Tg f orall g ∈ G. If a, b ∈ G and
a = b, then a−1 = b−1 and a−1 xa = b−1 xb f orall x ∈ G. Therefore Ta (x) =
=T(gh)r
Tb (x) f orall x ∈ G =⇒ Ta = Tb =⇒ ϕ(a) = ϕ(b).
=Tg(hr)
Therefore ϕ is well defined.
=Tg(hr)
Next, let a, b ∈ G, then ϕ(ab) = Tab = Ta Tb = ϕ(a)ϕ(b).
=Tg (Thr )
Therefore ϕ is a homomorphism.
=Tg (Th Tr ). Let Tg be any element of Inn(G). Then g ∈ G and ϕ(g) = Tg .
Therefore ϕ is onto.
So, associative property holds in Inn(G).
The kernel of ϕ is given by,
For Tg ∈ Inn(G), there exists the identity map Te (or I) in Inn(G) such that
Tg Te = Tg = Te Tg . Therefore Te is the identity of Inn(G).
For Tg ∈ Inn(G), there exists the inverse map Tg−1 in Inn(G), such that Kφ ={g ∈ G : ϕ(g) = Te , the identity element in Inn(G)}
Tg Tg−1 = Tgg−1 = Te . Therefore (Tg )−1 = Tg−1 =⇒ Tg−1 is the inverse of Inn(G).
={g ∈ G : Tg = Te }
Hence, Inn(G) is a group.
={g ∈ G : Tg (x) = Te (x), ∀ x ∈ G}
Theorem 3.5. For any group G, Inn(G) is a normal subgroup of Aut(G).
={g ∈ G : g −1 xg = x ∀ x ∈ G}
Proof. Clearly, Inn(G) is a subgroup of Aut(G), we have Inn(G) ⊂ Aut(G). Let ={g ∈ G : xg = gx ∀ x ∈ G}
Tg be any element of Inn(G) and T be any element of Aut(G). Then for x ∈ G,
=z, the centre of G.

[T Tg T −1 ](x) =T [Tg (T −1 (x))]


Thus, ϕ is a homomorphism of G onto Inn(G) with Kernel Z.
=T [Tg (T −1 (x))g −1 ] Therefore, By the Fundamental theorem of homomorphism for groups, we have
=T (g)T (T −1 (x))T (g −1 ) G
≈ Inn(G).
Z(G)
=T (g)x(T (g))−1
Lemma 3.6. Let G be a group and ϕ an automorphism of G. If a ∈ G is of order
=TT (g) (x) f orall x and T (g) ∈ G
o(a) > 0, then o(ϕ(a)) = o(a).

Therefore T Tg T −1 = TT (g) ∈ Inn(G). Proof. Suppose ϕ : G → G is an automorphism. For a ∈ G, suppose o(a) = n, so


Therefore Inn(G) is a normal subgroup of Aut(G).
3. UNIT : ISOMORPHISM AND AUTOMORPHISM THEOREMS37 3. UNIT : ISOMORPHISM AND AUTOMORPHISM THEOREMS38

that n is the least positive integer satisfying an = e, the identity of G. Then, Put 1 = I, 3 = A, 5 = B and 7 = AB = BA = C, in the composition table
and rewrite as follows.
[ϕ(a)]n =ϕ(a)ϕ(a) · · · ϕ(a)
X8 I A B C
=ϕ(a a · · · a) I I A B C
A A I C B
=ϕ(an )
B B C I A
=ϕ(e). C C B A I

Therefore [ϕ(a)]n = e. If [ϕ(a)]m = e for some 0 < m < n then [ϕ(a)]m =


[ϕ(a)]n . That is, ϕ(am ) = ϕ(an ), am = an = e f or 0 < m < n, which is a Clearly, K4 = {I, A, B, C} = {I, A, B, AB}.

contradiction to the fact o(a) = n. Therefore [ϕ(a)]n = e and n is the least positive Therefore K4 = ⟨A, B⟩ : A2 = B 2 = I, AB = BA, (i.e., K4 is an abelian

integer with this property. Therefore [ϕ(a)] = n. That is, [ϕ(a)] = o(a). group). {I, A} × {I, B} (product of two cyclic groups need not be cyclic). So, K4
is product of two cyclic groups of order 2 each but if itself is not cyclic.
Remark 3.1.    
I A B C I A B C
T0 = T1 =
1. Every cyclic group is an abelian group, but every abelian group is not cyclic I A B C I B C A
group. The example of the converse part is also called Klein’s 4 - group.
   
I A B C I A B C
2. If ⟨I, A⟩ and ⟨I, B⟩ are cyclic group, then the product of two cyclic group T2 = T3 =
I C A B I A C B
need not be cyclic, i.e., ⟨I, A⟩ · ⟨I, B⟩ is not cyclic.
   
Example 3.2. Write (i) Aut(K4 ), and (ii) Inn(K4 ), where K4 is Klein four I A B C I A B C
T4 = T5 =
I C B A I B A C
group. Hence, illustrate that the automorphism group of an abelian group need not
be abelian.
T0 T1 T2 T3 T4 T5
Solution : The set K4 = {1, 3, 5, 7} forms a group under multiplication mod8,
I I I I I I I
i.e., (K4 , (.mod 8)) is a group called Klein four group. A A B C A C B

The composition table for K4 is given by B B C A C B A


C C A B B A C
X8 1 3 5 7
1 1 3 5 7
3 3 1 7 5 Clearly, all maps are bijective.
5 5 7 1 3
Test for homomorphism : T3 (AB) = T3 (C) = B, T3 (A).T3 (B) = A.C = B
7 7 5 3 1
(from the table). Therefore T3 (AB) = T3 (A).T3 (B). Similarly, we can verify for

3. UNIT : ISOMORPHISM AND AUTOMORPHISM THEOREMS39 3. UNIT : ISOMORPHISM AND AUTOMORPHISM THEOREMS40

r
other pairs of elements. Therefore T0 , T1 , T2 , · · · , T5 are automorphism. Hence, Therefore r = and hence d = 1.
d
Aut(K4 ) = {T0 , T1 , T2 , · · · , T5 }. Further, note that Aut(K4 ) ≈ S3 , where S3 Conversely, suppose 0 < s < r and (r, s) = 1, then we verify that the homo-
is non-abelian group. Therefore Aut(K4 ) is non-abelian, where K4 is abelian . morphism S : G → G defined by S(a) = as ( or S(ar ) = as ) or S(ai ) = ais is an
Therefore, automorphism group of an abelian group need not be abelian. automorphism. For ai , aj ∈ G we have,
We observe that Inner automorphism of an abelian group is an only identity Therefore S is homomorphism. For ai , aj ∈ G we have
map. For g ∈ G, we have
S(ai , aj ) =S(ai+j )
Tg (x) =gxg −1 =a(i+j)s
=xgg −1 =ais .ajs
=x =S(ai )S(aj )
=Te (x), ∀ x ∈ G
Therefore S is homomorphism. Let S(ai ) = S(aj ), then ais = ajs .
Therefore Tg = Te (or I) or Tg is the identity map. Since, K4 is an abelian ⇒ ais−js = 1ore
group. Inn(K4 ) = I. ⇒ a(i−j)s = 1ore, where 0 < i, j < r
r
Example 3.3. Let G be a finite group of order r and ∪r be the group of integers less ⇒ (i − j)s = 0 or ; (i − j)s = 0 or i = j
(i − j)s
than r and relatively prime to r, under modr [i.e., (∪r , ⊗r , 1)]. Then Aut(G) ≈ ∪r . Therefore S(ai ) = S(aj ) ⇒ ai = aj .

Solution : Let G be a cyclic group of order r with generator a. Therefore ∴ S is one-one.

G = ⟨a⟩ = {a1 , a2 , a3 , · · · , ar−1 , ar = e}. Now, to prove S is onto.


i i
If T : G → G is a homomorphism, then T (a ) = [T (a)] for 0 < i < r, it Choose an element of ai in G, let k be any integer such that

follows that all maps T are determined if we know T (a). sk ≡ i(modr) for 0 < k < r. i. e., sk = i + qr for an integer q.

We verify that T is an sutomorphism of G iff T (a) = at where 0 < t < r and Then, S(ak ) = ask = ai+qr = ai .(ar )q = ai [∵ ar = 1 or e]. i.e., For ai ∈ G

(t, r) = 1 (i.e., t is less than r and prime to r or t is relatively prime with r). there exixts ak ∈ G such that S(ak ) = ai .

For, suppose, if possible the map T is defined by T (a) = at is an automorphism Therefore S is onto.
d d Hence, S is an automorphism.
for (t, r) = d. Then and .
r t
We denote the automorphism T (a) = at by Tt (a) = at , where (r, t) = 1, for
r t t
0 < t < 1. So, Aut(G) = {Tt : (t, r) = 1, 0 < t < 1}.
[T (a)] d = (at ) d = (ar ) d = e [∵ ar = e or 1]
Define a map η : Ur → Aut(G) by η(t) = Tt .
r r
Therefore O(T (a)) = . Therefore O(a) = , since O(T (a)) = o(a)[∵
d d
T is homomorrphism ⇒ o(T (a)) = o(a)].
3. UNIT : ISOMORPHISM AND AUTOMORPHISM THEOREMS41 3. UNIT : ISOMORPHISM AND AUTOMORPHISM THEOREMS42

Consider
t1 , t2 , η(t1 , t2 ) = Tt1 t2 −→ (1) SUMMARY

again, consider A group isomorphism is a function between two groups that sets up a one-to-one
correspondence between the elements of the groups in a way that respects the
(Tt1 t2 )(a) =at1 t2 [∵ Tt (a) = at ] given group operations. If there exists an isomorphism between two groups, then
=(at2 )t1 the groups are called isomorphic. An automorphism is an isomorphism from a

=[Tt2 (a)]t1 [∵ at = Tt (a)] group to itself. Also, set of all automorphisms /inner automorphisms forms a

=Tt1 [Tt2 (a)] group.

(Tt1 t2 )(a) =(Tt1 Tt2 )(a)


KEYWORDS

But, Tt1 t2 = Tt1 Tt2 −→ (2) Isomorphism, Automorphisms, Group of automorphisms, Inner automorphisms,
By using (2) in (1) , we get Group of Inner automorphisms, Kelin 4-group.

η(t1 t2 ) = Tt1 t2 = Tt1 Tt2 = η(t1 )η(t2 ) [∵ Tt = η(t)].


ASSESSMENT /TERMINAL QUESTIONS
G ∼ G∼
Therefore η is a homomorphism. 1. For any group G, show that = G and = {e}.
{e} G
To prove that η is one-one and onto. 2. If N and M are two normal subgroup of a graoup G, then show that N M
Let η(t1 ) = η(t2 ). Tt1 = Tt2 . Therefore Tt1 (a) = Tt2 (a). and N ∩ M is a normal subgroup of a group G.
⇒ at1 = at2 ⇒ at1 −t2 = 1 or e
r 3. Show that any inifinte cyclic group is isomorphic to < Z, + > the group of
t1 − t2 or for some 0 < t1 , t2 < r implies t1 = t2 .
t1 − t2 integers.
Therefore η is one-one.
Now, for any Tt ∈ Aut(G) (i.e., 0 < t < r and (r, t)− = 1 there exists t ∈ ∪r 4. Find all automorphism of any cyclic group of order 8.

such that η(t) = Tt . Therefore η is onto. Hence η is isomorphism. Therefore


5. Let G be an infinite group. Then show that G has just one non-trivial
∪r ≈ Aut(G).
automorphism.

Note 3.3. Aut(G) is abelian but it need not be cyclic


6. Show that x → x−1 is an automorphism of a goup if and only if G is ableian.

Example 3.4. ∪φ(8) ≈ K4 where K4 is a klen’s four group. i. e., K4 is not cyclic.
7. Let G be a group of order 4, G = {e, a, b, ab}, a2 = b2 = e, ab = ba.
But Aut(G) ≈ ∪φ(8)
Determine Aut(G).

3. UNIT : ISOMORPHISM AND AUTOMORPHISM THEOREMS43 44

8. Prove that every finite group having more than two elements has a nontrivial 4 UNIT : Permutation groups
automorphism.

REFERENCES OBJECTIVES

1. I. N. Herstein: Topics in Algebra, 2nd Edition, John Wiley and Sons, 2008. After studying this unit you will be able to:

2. Surjeet Singh and Qazi Zameeruddin: Modern Algebra, 8th Edition, Vikas • explain the concepts of permutation group;
Publishing House, 2021.
• understand the concepts of symmetric group and alternating group with
3. N. Jacobson: Basic Algebra-I, 2nd Edition, Dover Publications, 2009. suitable examples;

4. M. Artin : Algebra, 2nd Edition, Prentice Hall of India, 2015. • illustrate the examples of symmetric group Sn with n = 3, 4, ...;

5. Darek F. Holt, Bettina Eick and Eamonaa A. O’brien: Handbook of com- • describe the concepts of non-ableian group through the symmetric group S3 ;
putational group theory, 1st Edition, Chapman & Hall/CRC Press, 2005.
• explain the concepts of group of automorphism and inner automorphism in
6. J. B. Fraleigh : A first course in abstract algebra, 7th Edition, Addison- symmetric group S3 .
Wesley Longman, 2002.
• state, prove and apply the Cayley thoerem’s for permutation group;
7. D. S. Dummitv and R.M. Foote: Abstract Algebra, 2nd Edition, John Wiley
• extend the concepts of group of permutation groups and compare with group
& Sons, 2003.
of symmetric group.
8. V. K. Khanna and S. K. Bhambri: A course in Abstract algebra, 4th Edition,
Vikas Publishing House Pvt Ltd., 2013.
INTRODUCTION

A permutation is a mathematical calculation of the number of ways a particular


set can be arranged, where the order of the arrangement matters. A permutation
group is a group G whose elements are permutations of a given set S and whose
group operation is the composition of permutations in S. The way in which the
elements of a permutation group permute the elements of the set is called its group
action. Group actions have applications in the study of symmetries, combinatorics
and many other branches of mathematics, physics and chemistry.
4. UNIT : PERMUTATION GROUPS 45 4. UNIT : PERMUTATION GROUPS 46

Definition 4.1. A permutation is a ono-one transformation of a finite set into Definition 4.5. Let S = {1, 2, 3}. Then the symmetric group S3 = {ρ0 , ρ1 , ρ2 , δ1 , δ2 , δ3 },
itself. The number of elements in the finite set is known as the degree of permu- where
 
tation. 1 2 3
ρ0 =   = I ⇒ even permutation,
Definition 4.2. The group Sn is called the symmetric group or the permuta- 1 2 3

tion group on n symbols. The group Sn is called the symmetric group because for
 
n = 3, 4, etc., elements of Sn can be interpreted as symmetric group of a triangle, 1 2 3
ρ1 =   = (1 3 2) = (1 3)(1 2) ⇒ even permutation,
a square, etc. 3 2 1

Note 4.1.  
1 2 3
ρ2 =   = (1 2 3) = (1 2) (1 3) ⇒ even permutation,
(i) If S contains n elements, then Sn contains n! elements.
2 3 1

(ii) The group Sn (n ≥ 3) is a non-abelian group.


 
1 2 3
Definition 4.3. A permutation which replaces n objects cyclically is called a δ1 =   = (2 3) ⇒ odd permutation,
1 3 2
cyclic permutation of degree n.

Example 4.1.
 
1 2 3
  δ2 =   = (1 3) ⇒ odd permutation, and
1 2 3 4 5 3 2 1
(1) f =   is a cyclic permutation and this can be written as
2 3 4 5 1
f = (12345).  
1 2 3
δ3 =   = (1 2) ⇒ odd odd permutation.
(2) The number of objects permuted by a cycle is called its length. A cycle of 2 1 3
length two is called a transposition.
Therefore A3 = {ρ0 , ρ1 , ρ2 } is an alternating group.
Definition 4.4. A permutation in Sn is said to be even permutation if it can Now, we show that the group S3 is non-abelian.
be expressed as a product of an even number of transposition (cycles of length two) Consider,   
 
and it is said to be odd permutation if it is expressed as a product of odd number 1 2 3 1 2 3 1 2 3
ρ2 δ 1 =   =  = δ2 , and
of transpositions. 2 3 1 1 3 2 3 2 1

Note 4.2. In any permutation group, even permutation × even permutation =     


1 2 3 1 2 3 1 2 3
even permutation = odd permutation × odd permutation. Also, even permutation δ1 ρ2 =   =  = δ3
1 3 2 2 3 1 2 1 3
× odd permutation = odd permutation × even permutation = odd permutation.

4. UNIT : PERMUTATION GROUPS 47 4. UNIT : PERMUTATION GROUPS 48

   
Therefore ρ2 δ1 ̸= δ1 ρ2 . ρ0 ρ 1 ρ 2 δ 1 δ 2 δ 3 ρ0 ρ1 ρ2 δ1 δ2 δ3
T ρ1 = , T ρ1 =
ρ0 ρ1 ρ2 δ2 δ3 δ1 ρ0 ρ1 ρ2 δ3 δ1 δ2
Hence S3 is a non-abelian group of order 6.
   
ρ0 ρ1 ρ2 δ1 δ2 δ3 ρ0 ρ1 ρ2 δ1 δ2 δ3
Note 4.3. T δ1 = , T δ2 =
ρ0 ρ2 ρ1 δ1 δ3 δ2 ρ0 ρ2 ρ1 δ3 δ2 δ1

(i) The symmetric group S3 is a cyclic group. Also, every proper subgroup of
Similarly,
 we have 
S3 is cyclic. ρ0 ρ1 ρ2 δ1 δ2 δ3
T δ3 =  .
(ii) Every permutation is the product of its cycle. Also, every permutation is ρ0 ρ2 ρ1 δ2 δ1 δ3

product of 2-cycles (We shall refer to 2-cycles as transpositions). Therefore, InnS3 = {Tρ0 , Tρ1 , Tρ2 , Tδ1 , Tδ2 , Tδ3 }. We claim that only the inner

(iii) The symmetric Sn has a normal subgroup of index 2, the alternating group automorphisms the only automorphism of S3 .

An , consisting of all even permutations. Suppose J is the other automorphism of S3 . Then J(ρ0 ) = ρ0 and

(iv) The composition table (Cayley table) in S3 as follows. J(ρ2 ) = J(ρ1 ρ1 ) = J(ρ1 )J(ρ1 ) −→ (1)

· ρ0 ρ1 ρ2 δ1 δ2 δ3 Hence, J(ρ2 ) is known, when J(ρ1 ) is known. We can now verify that J(ρ1 ) =
ρ0 ρ0 ρ1 ρ2 δ1 δ2 δ3 ρ1 or ρ2 . Because if J(ρ1 ) = δ1 , then J(ρ2 ) = δ1 δ1 = ρ0 , where ρ2 ̸= ρ0 . This
ρ1 ρ1 ρ2 ρ0 δ3 δ1 δ2 contradiction the fact that J is one-one and J(ρ0 ) = ρ0 . Hence, J(ρ1 ) ̸= δ1 .
ρ2 ρ2 ρ0 ρ1 δ2 δ3 δ1
Similarly, J(ρ1 ) ̸= δ2 and δ3 . Thus
δ1 δ1 δ2 δ3 ρ0 ρ1 ρ2
δ2 δ2 δ3 δ1 ρ2 ρ0 ρ1 J(ρ1 ) = ρ1 or ρ2 −→ (2)
δ3 δ3 δ1 δ2 ρ1 ρ2 ρ0
Further, we verify that J(δ1 ) = δ1 or δ2 or δ3 . For, if J(δ1 ) = ρ1 , then J(ρ0 ) =
J(δ1 δ1 ) = J(δ1 )J(δ1 ) = ρ1 ρ1 = ρ2 . This again contradicts the fact that J(ρ0 ) = ρ0
Excercise 4.1. For G = S3 , determine Inn(G) and verify that Inn(G) = Aut(G).
only. Therefore J((δ1 ) ̸= ρ1 .
Solution : Let S = {1, 2, 3} and S3 = {ρ0 , ρ1 , ρ2 , δ1 , δ2 , δ3 }. By composition table Similarly we can prove, J((δ1 ) ̸= ρ1 , ρ2 . Hence,
for S3 , wehave 
ρ0 ρ1 ρ2 δ1 δ2 δ3 J((δ1 ) = δ1 orδ2 or δ3 −→ (3).
T ρ0 =  
−1 −1 −1 −1 −1 −1
ρ0 ρ0 ρ0 ρ0 ρ1 ρ0 ρ0 ρ2 ρ0 ρ0 δ1 ρ0 ρ0 δ2 ρ0 ρ0 δ3 ρ0
 
ρ0 ρ1 ρ2 δ 1 δ 2 δ 3 If we choose J(ρ1 ) = ρ1 and J((δ1 ) = δ1 . Then
T ρ0 =  
ρ0 ρ1 ρ2 δ 1 δ 2 δ 3
4. UNIT : PERMUTATION GROUPS 49 4. UNIT : PERMUTATION GROUPS 50

 
ρ0 ρ1 ρ2 δ1 δ2 δ3 Now, if y ∈ G, then there exists yg −1 ∈ G such that τg (yg −1 ) = yg −1 g = ye =
J = .
ρ0 ρ1 ρ2 δ1 δ2 δ3 y. It follows that τg is onto. i.e., τg maps S onto itself.
Thus we have proved that for every g ∈ G, τg ∈ A(S). Define : ψ : G → A(S)
Since J(δ2 ) = J(ρ1 δ1 ) = J(ρ1 )J(δ1 ) = ρ1 δ1 = δ2 , and
by ψ(g) = τg f orall g ∈ G. Consider
J(δ3 ) = J(δ1 ρ1 ) = J(ρ1 )J(δ1 ) = ρ1 δ1 = δ3 .
From (2) and (3) it follows that J maps ρ’s onto ρ’s and δ’s onto δ’s but not τgh (x) = x(gh) [by def inition of Igh ]
ρ’s onto δ’s or δ’s onto ρ’s.
= (xg)h [by associativity in G]
Therefore J is one among Tρ0 , Tρ1 , Tρ2 , Tδ1 , Tδ2 , Tδ3 . i.e., J ∈ Inn(S3 ) for any
= τh (xg) [by def inition of τh ]
automorphisms J. So, Aut(S3 ) ⊂ Inn(S3 ) −→ (4)
= τh (τg (x)) [by def inition of τg ]
But, Inn(S3 ) ⊂ Aut(S3 ) −→ (5).
From (4) and (5) , we conclude that Inn(S3 ) = Aut(S3 ). = (τh τg )(x) ∀ x ∈ G

= (τg )τh (x) ∀ x ∈ G [by associativity in G]


Definition 4.6. If S is the finite set {1, 2, · · · , n}, then the group of all permu-
tations of S is the symmetric group on n letters (i.e., Sn ). An A(S) is called a
Therefore τgh = τg τh .
group of permutation of G.
The above relation tells us that ψ(gh) = τgh = τg τh

Theorem 4.1. (Cayley’s Theorem) Every group is isomorphic to a subgroup Therefore ψ(gh) = ψ(g)ψ(h). i. e., ψ is a homomorphism. We shall now find

of A(S) for some appropriate S or Every group is isomorphic to a group of per- the kernel K of ψ.

mutations A(G) or Every group is isomorphic to a sub-group of a transformation By definition of K if g0 ∈ K, ψ(g0 ) = τg0 is the identity map on S, so that for

group. x ∈ G in particular for e ∈ G, we have τg0 (e) = e −→ (1)


On the other hand, τg0 (e) = eg0 = g0 −→ (2)
Proof. Let G be any group. Then A(S) is the group of all permutations of G
Comparing equations (1) and (2), we get g0 = e.
i.e., S = {x/x ∈ G}. Define for g ∈ G, τg : G → G (i.e., τg : S → S) by
Hence K = {e}.
τg (x) = xg f orall x ∈ G. For any x, y ∈ G, we have
(We know that a homomorphism ϕ : G onto G with Kernel K is an isomorphism
of G onto G if and only if, K = {e}).
τg (x) = τg (y)
Thus, we have prove that ψ is an isomorphism of G into A(S).
xg = yg [by cancellation law in G]

x = y. Note 4.4. The above theorem, put S = G, then A(G) is also called a group of
permutations.
Therefore τg is one - one.
Corollary 4.1. Any finite group is isomorphic to subgroup of a symmetric group.

4. UNIT : PERMUTATION GROUPS 51 4. UNIT : PERMUTATION GROUPS 52

Proof. If G is finite and its contain n-elements then A(S) = Sn (i.e,. Symmetric Now, define θ : G → A(S) by θ(g) = τg , ∀ g ∈ G clearly, θ is well-defined. Let
group). Hence by above theorem (Cayley theorem), we get G is isomorphic to g, h ∈ G. Then θ(gh) = τg τh = θ(g)θ(h).
subgroup of the symmetric group or G is isomorphic to a permutation group. Therefore θ is a homomorphism.
The Kernel of θ is given by,
Theorem 4.2. A : If G is a group, H a subgroup of G and S is the set of all
right cosets of H in G, then there is a homomorphism θ of G into A(S) and the
Kθ = {g ∈ G/θ(g) = τe , the identity in A(S)
kernel of θ is the largest normal subgroup of G which is contained in H.
= {g ∈ G/τg = τe }
Proof. We have S = {Hx : x ∈ G}. For each g ∈ G. Define τ : S → S by
= {g ∈ G/τg (Hx) = τe (Hx), f orall x ∈ G}
τg (Hx) = Hxg, ∀ x ∈ G. Suppose that Hx = Hy. Then xy −1 ∈ H.
= {g ∈ G/Hxg = Hx, f orall x ∈ G}
⇒ xgg −1 y −1 ∈ H
⇒ (xg)(yg −1 ) ∈ H Let g be any element of Kθ . Then Hxg = Hx, f orall x ∈ G
Therefore Hxg = Hyg. i.e., τg (Hx) = τg (Hy) Therefore Hg = Heg = He = H.
Therefore τg is well-defined. ∴ Hg = H, f orall g ∈ H.
Next, τg (Hx) = τg (Hy). Therefore Kθ ⊂ H.
⇒ Hxg = Hyg. Finally, let N be any normal subgroup of G and N ⊂ H, let n ∈ N and g ∈ G
⇒ Hx = Hy. be arbitrary. Then gng −1 ∈ H.
Therefore τg is one-one. Therefore Hgng −1 = H =⇒ Hgn = Hg f orall g ∈ G. i.e., n ∈ Kθ .
Let Hx be any element of S. Then there exists Hxg −1 is such that Therefore N ⊂ Kθ . Thus Kθ is the largest normal subgroup of G which is
τg (Hxg −1 ) = Hxg −1 g = Hx. contained in H.
Therefore τg is onto.
Let τg , τh be any two elements of A(S). Then Note 4.5.

(i) If H = {e}, then the above theorem gives the result of Cayley’s theorem.
τg τh (Hx) = τg [τh (Hx)] [by the def inition of τg ]

= τg [Hxh] (ii) A group is called Simple group if it has no non-trivial (proper) normal sub-
groups.
= Hx(hg)

= Hx(gh) [associativityiG] (iii) The number of distinct right(left) cosets of a subgroup H of a group is called

= τgh (Hx) the index of the subgroup H in G and is denoted by [G : H] or IG (H) or


o(G)
i(H) = or o(H) | o(G).
o(H)
Therefore τg τh = τgh .
4. UNIT : PERMUTATION GROUPS 53 4. UNIT : PERMUTATION GROUPS 54

(iv) Let G be a finite group and H be a subgroup of G. Then o(H) | o(G), which ∴ o(G) > i(H)!
is known as Lagrange’s theorem. Therefore o(G) ∤ i(H)!

Theorem 4.3. B If G is a finite group, and H ̸= G is a subgroup of G such By the Theorem B, G cannot be simple.
that o(G) ∤ i(H)!, then H must contain a non-trivial normal subgroup of G. In Therefore G has a non-trivial normal subgroup which contained in H. Since
particular, G cannot be simple. o(H) = 9 and is divisible by the order of that normal subgroup, the normal

Proof. Let S = {Hg/g ∈ G}. Then, there is a homomorphism θ : G → A(S) and subgroup is of order 3 or 9.

the Kθ is the largest normal subgroup of G which is contained in H. (i. e., by


2. Let G be a group of order 99 and H be a subgroup of G of order 11. Then
theorem A)
i(H) = 9
First, let o(G) > i(H)!, then, the homomorphism θ cannot be an isomorphism.
∴ i(H)! = 9! (∵ 99 = 11 × 3 × 3)
For, if it were then θ(G) would be a subgroup of A(S) and by Lagrange’s theorem,
we have o(θ(G))/o(A(S)) i.e., o(θ(G))/i(H)!. Therefore o(G) ∤ i(H)!

If θ is an isomorphism, then o(θ(G)) = o(G). Thus, o(G)/i(H)!. This is By, the Theorem B, G is not simple.
impossible, since o(G) > i(H)!. Therefore G has a non trivial normal subgroup N (say) such that N ≤ H.
Now, Since θ is not an isomorphism, we have Kθ ̸= {e}. Thus, H contains a Since o(N )/o(H) = H.
non trivial normal subgroup Kθ of H.
Therefore N = H i.e., H itself is a normal subgroup of G.
Therefore G is not simple.
Next, let o(G) < i(H)! and o(G) ∤ i(H)! by Lagrange’s theorem, the order of Excercise 4.2. Prove that any non-abelian group of order 6 is isomorphic to

every subgroup of A(S) divides o(A(S)) = i(H)!. Since, o(G) ∤ i(H)!. Therefore symmetric group S3 .

A(S) does not contain a subgroup of order o(G).


Solution. Let G be a non-abelian group of order 6. Then there exists an element
Again, if θ is an isomorphism, then θ(G) would be a subgroup of A(S) and
a ̸= e in G satisfying a2 = e. Thus, H = {e, a} is a subgroup of G. clearly,
o(θ(G)) = o(G). So, θ is not an isomorphism. Therefore Kθ ̸= {e}. Hence, G o(G) 6
i(G) = = = 3.
contains the non-trivial normal subgroup Kθ of G. o(H) 2
We claim that H is not a normal subgroup of G. For, suppose, if possible,
Therefore G is not simple. Hence the proof is complete.
H ◁ G. Then for any g ∈ G
Example 4.2. We have, gag −1 ∈ H. So, gag −1 = e or gag −1 = a. Since gag −1 = e gives
a = e. We must have, gag −1 = e i.e., ga = agor f orall g ∈ G.
Applications of Theorem B
For b ∈ G, b ∈
/ H. Consider N (b) = {x ∈ G/xb = bx} which is a subgroup of
1. Let G be a group of order 36 and H is a subgroup of G of order 9. Then G. Clearly N (b) ⊃ H.
i(H) = 4 ⇒ i(H)! = 4! = 24.

4. UNIT : PERMUTATION GROUPS 55 4. UNIT : PERMUTATION GROUPS 56

However N (b) ̸= H, Since b ∈ N (H) and b ∈


/ H. Thus H is a proper subgroup ric group Sn ; n ≥ 2 is a normal subgroup of Sn . Further show that index of
of N (b). So, by Lagranges theorem, we have o(H)/o(N (b)) and o(N (b))/o(G). An in Sn is 2.
Therefore, o(N (b))/o(G) ⇒ o(N (b))/6 ⇒ o(N (b)) = 3 or 6, if o(N (b)) = 3, then
2. Show that every permutation on a finite set is a product of disjoint cycles.
o(H)/o(N (b)) ⇒ 2/3, which is not true.
Sn
Therefore o(N (b)) = 6 and hence N (b) = G. 3. Show that ≈ {+1, −1}.
An
Therefore b commutes with all elements of G, Since b is arbitrary element of
4. Show by an example of permutation group that converse of Lagrenge’s the-
G, it follows that every element of G commutes with every other element of G,
orem may not hold.
making G into an abelian group, contrary to the assumption. Thus H ⋪ G.
Clearly, H has no non-trivial normal subgroups of G is isomorphic to a sub- 5. Show that an order of any permutation f in Sn is equal to the least common

group T or under the isomorphism, θ : G → A(S), defined by θ(g) = τg , where multiple of the orders of the disjoint cycles.

S = {Hx/x ∈ G} and τg (Hx) = Hxg.


6. Show that an odd permutation is of even order.
Since, o(A(S)) = [i(H)]! = 3! = 6, it follows that T is a subgroup of order 6 in
A(S) which is also of order 6. Hence, T = A(S) = S3 . Therefore G ≈ S3 . 7. Using Cayley’s theorem, find the permutation group isomorphic with the
dihedral group of order 8, where the dihedral group D8 is a subgroup of

SUMMARY symmetric group S4 . i.e., D8 = {a, a2 , a3 , a4 = e, ab, a2 b, a3 b, b : a4 = e =


b2 , b−1 ab = a−1 , ...}.
A permutation group is a finite group G whose elements are permutations of a
given set and whose group operation is composition of permutations in G. The 8. Show by an example of permutation group that we can find three groups

action of a group on a set captures the algebraic structure of this transformation, E ⊆ F ⊆ G such that E is normal in F , F is normal in G, where E is not

for all the elements of the group. So, the action of a group on a set details precisely normal in G.

how the set transforms under the symmetry described by the group. REFERENCES

1. I. N. Herstein: Topics in Algebra, 2nd Edition, John Wiley and Sons, 2008.
KEYWORDS

2. Surjeet Singh and Qazi Zameeruddin: Modern Algebra, 8th Edition, Vikas
Permutation group, Symmetric group, Alternating group, Cayley’s theorem for
Publishing House, 2021.
permutation group, Dihedral group.

3. N. Jacobson: Basic Algebra-I, 2nd Edition, Dover Publications, 2009.


ASSESSMENT /TERMINAL QUESTIONS
4. M. Artin : Algebra, 2nd Edition, Prentice Hall of India, 2015.
1. Show that the alternating group An of all even permutations of the symmet-
4. UNIT : PERMUTATION GROUPS 57 4. UNIT : PERMUTATION GROUPS 58

5. Darek F. Holt, Bettina Eick and Eamonaa A. O’brien: Handbook of com- BLOCK 2: ADVANCED GROUP THEORY
putational group theory, 1st Edition, Chapman & Hall/CRC Press, 2005.

6. J. B. Fraleigh : A first course in abstract algebra, 7th Edition, Addison-


Wesley Longman, 2002. UNIT-5: Class Equation of Finite Groups

7. D. S. Dummitv and R.M. Foote: Abstract Algebra, 2nd Edition, John Wiley UNIT-6: Applications of Class Equation

& Sons, 2003.


UNIT-7: Sylow’s Theorems

8. V. K. Khanna and S. K. Bhambri: A course in Abstract algebra, 4th Edition,


UNIT-8: Applications of Sylow’s Theorem
Vikas Publishing House Pvt Ltd., 2013.

59 5. UNIT : CLASS EQUATION OF FINITE GROUPS 60

5 UNIT : Class Equation of Finite Groups (i) N (H) is a subgroup of G

(ii) H is normal in N (H)

OBJECTIVES (iii) If H is a normal subgroup of the subgroup K in G, then K ⊆ N (H) ( i.e.,


After studying this unit you will be able to: N (H) is the largest subgroup of G in which H is normal).
• understanding the concepts of conjugate elements;
(iv) H is normal in G if and only if N (H) = G.
• explain the concepts of equivalence relations;
Solution. Let H be a subgroup of group G and N (H) = {g ∈ G : gHg −1 =
• define the concepts of normalizer/centralizer of a group and their properties; H}. Then

• understanding fundamental properties of conjugacy classes and self-conjugacy (i) Let g1 .g2 ∈ N (H) so that g1 Hg1−1 = H and g2 Hg2−1 = H.
class; ∴ g2 Hg2−1 = g1 Hg1−1
i.e., g1−1 g2 Hg2−1 g1 = H
• derive the class equation of finite group; i.e., g1−1 g2 H(g1−1 g2 )−1 = H
∴ g1−1 g2 ∈ N (H) and hence N (H) is a subgroup of G.
• expalin the fundamnetal properties of class equation of finite groups.
(ii) For g ∈ N (H) and h ∈ H, we have
• by using generator-relater form, verify the class equation of symmetric group
gHg −1 = H, where gHg −1 = {ghg −1 : h ∈ H}
S3 , dihedral group D8 and hamiltonian group Q8 .
∴ ghg −1 ∈ gHg −1 =⇒ ghg −1 ∈ H for any g ∈ N (H) and h ∈ H
=⇒ H ◁ N (H)
INTRODUCTION
(iii) Let k ∈ K. Since H is a normal subgroup of K, we have
The study of conjugacy classes of non-abelian groups is fundamental for the study kHk −1 = H =⇒ k ∈ N (H) (by(i))
of their structure. The term class equation (or class formula, or orbit decompo- Thus k ∈ K =⇒ k ∈ N (H).
sition formula) refers to a basic type of counting argument that comes about by Hence K ⊆ N (H).
decomposing a finite group set as a union of its orbits. This has a number of fun-
damental applications in group theory. In this unit, we deal with the conjugates (iv) Let H be normal in G.
of an element and find a useful relation, known as the conjugacy class equation, If g ∈ G, then gHg −1 = H [∵ H is normal in G].
between the order of a finite group G and order of its center. =⇒ g ∈ N (H). Thus g ∈ G =⇒ g ∈ N (H) (by(iii))
∴ G ⊆ N (H). But N (H) ⊆ G. Therefore, G = N (H).
Normalizer of a subgroup of a group: Conversely, let N (H) = G. Then g ∈ G =⇒ g ∈ N (H) =⇒ gHg −1 = H.
Thus we have gHg −1 = H forall g ∈ G.
Therefore, H is a normal subgroup of G.
Definition 5.1. The normalizer of H in a group G is the set of all those ele-
ments of G which commute with H. Symbolically N (H) = {g ∈ G : gHg −1 = H}. Definition 5.2. If a, b ∈ G, then b is said to be a conjugate of a in G if there
The normalizer of an element N (x) (or centralizer of a group Z = Z(G)) exists an element c ∈ G such that b = c−1 ac. When b is conjugate to a, we denote
of a group G is the set of all elements of G which commute with every element of
this relation by b ∼ a. Thus b ∼ a if b = c−1 ac. This relation is also called a
G. N (x) = {x ∈ G : gxg −1 = x ∀g ∈ G} (or, Z(G) = {x ∈ G : xg = gx ∀g ∈ G}).
Conjugacy.
Excercise 5.1. If H is a subgroup of group G and N (H) = {g ∈ G : gHg −1 = H},
Note: If G is abelian, then for any c ∈ G, we have c−1 ac = c−1 ca = a. In other-
then show that words, the conjugate of ′ a′ is ′ a′ itself (also known as self conjugate element).
5. UNIT : CLASS EQUATION OF FINITE GROUPS 61 5. UNIT : CLASS EQUATION OF FINITE GROUPS 62

Lemma 5.1. Conjugacy is an equivalence relation on a group G. Lemma 5.2. N (a) is a subgroup of G.

Proof. (i) Since e−1 ae = a, e ∈ G, ′ a′ is a conjugate of ′ a′ in G. i.e., a ∼ a. Proof. For x, y ∈ N (a), we have ax = xa and ya = ay. Therefore (xy)a = x(ya) =
∴ The relation is reflexive. x(ay) = (xa)y = (ax)y = a(xy) =⇒ xy ∈ N (a). Further, from ax = xa, we have

−1
x−1 a = ax−1 . Therefore x−1 ∈ N (a) and hence N (a) is a subgroup of G.
(ii) Let a ∼ b. Then there exists an element c ∈ G such that b = c ac
theref orecbc−1 = a Note 5.2.
theref orea = (c−1 )−1 bc−1 for c−1 ∈ G
(i) N (a) is not necessarily a normal subgroup of G.
theref oreb ∼ a
(ii) Since ex = xe ∀x ∈ G, N (e) = G.
∴ The relation is symmetric.
(iii) If G is an abelian group, N (a) = G ∀a ∈ G.
(iii) Let a ∼ b and b ∼ c. Then there exist x, y ∈ G such that b = x−1 ax and (iv) ∩N (a) = Z(G), the centre of G.
c = y −1 by.
o(G)
But then we have, Theorem 5.1. If G is a finite group and a ∈ G, then Ca = . [In other-
o(N (a))
−1 −1 −1 −1
c = y by = y x axy = (xy) a(xy), xy ∈ G. words number of elements conjugate to a in G is the index of the normalizer of a

∴ The relation is transitive. in G].

Hence the conjugacy is an equivalence relation.


Proof. Suppose x and y belong to some right coset of Na in G then y ∈ (N (a))x.

Definition 5.3. The equivalence classes under the relation b ∼ a if and only if Therefore y = nx, where n ∈ N (a). Since n ∈ N (a), we have a = n−1 an or

b = c−1 ac, c ∈ G, are called a Conjugate classes. na = an. Now y −1 = x−1 n−1 . Therefore

Definition 5.4. Let ∼ be the conjugacy relation in a group G,defined by a ∼ b iff


y −1 ay = x−1 n−1 anx [y −1 = x−1 n−1 , y = nx]
a = x−1 bx ∀x ∈ G. Then the equivalence class of ′ a′ , denoted by C(a) is called the
= x−1 n−1 nax [∵ n ∈ N (a)]
conjugacy class of an element ′ a′ in G.
= x−1 ax.
i.e., C(a) = b ∈ G : b ∼ a = b ∈ G : b = x−1 ax = x−1 ax : x ∈ G.

Note 5.1. Therefore x and y yield the same conjugate of a.

(i) o(C(a)) = Ca Next, let x and y ∈ distinct right coset of Na . We claim x−1 ax ̸= y −1 ay.
For if x−1 ax = y −1 ay, we have (yx−1 )a = a(yx−1 ), (yx−1 ) ∈ N (a). Therefore
(ii) o(G) = o(∪a∈G C(a)), where two conjugacy class C(a) and C(b) are either
yx−1 = n for some n ∈ N (a) =⇒ y = nx ∈ (N (a))x. In other words, xy belongs
P
disjoint or identical (i.e., o(G) = a∈G Ca ). Here, the summation runs
through a set a ∈ G, using only one a from each conjugate class.
to same right coset (N (a))x of N (a). This contradicts the fact that they are in
(iii) If G is a group and a ∈ G, then N (a) = x ∈ G/xa = ax is called the nor- different cosets. Thus, there is a one-one correspondence between the conjugate
malizer of a in G.

5. UNIT : CLASS EQUATION OF FINITE GROUPS 63 5. UNIT : CLASS EQUATION OF FINITE GROUPS 64

of a and the right coset of N (a). So if G is a finite group, then Ca means the Clearly, a ∈ Z if and only if N (a) = G. If G is finite, a ∈ Z if and only if
number of distinct elements in C(a), the number of distinct right coset of N (a) in o(N (a)) = o(G). Equation (1) in view of above lemma, we have
o(G)
G. Therefore index of Na in G. Hence o(N (a))
.
X o(G) X o(G)
o(G) = +
P o(G) o(G) o(N (a))
Corollary 5.1. If G is a finite, then o(G) = a , where summation runs a∈Z(G) a∈GorN
/ (a)̸=G
o(N (a)) X X o(G)
over one element a in each conjugate class. = 1+
o(N (a))
N (a)=G N (a)̸=G
P
Proof. Since o(G) = a Ca , by above theorem we have result. X o(G)
=o(∩a∈G N (a)) +
o(N (a))
N (a)̸=G
Note 5.3. The equation in this corolary is often called the class equation.
X o(G)
o(G) =o(Z(G)) + .
Lemma 5.3 (A). a ∈ Z(G) if and only if N (a) = G. If G is finite, then o(N (a))
N (a)̸=G

a ∈ Z(G) if and only if o(N (a)) = o(G).

Proof. First if a ∈ Z(G) then ax = xa ∀x ∈ G [ By definition of Z(G) ].


Note 5.4. If G is abelian, we observe that N (a) = G. Hence {a ∈ G : N (a) ̸=
⇔ x ∈ N (a) ∀x ∈ G [ By definition of N (a) ].
G} = ϕ, the empty set. Therefore
Conversely, if N (a) = G then a = xax−1 ∀x ∈ G ⇔ ax = xa ⇔ a ∈ Z(G). Finally
if G is finite, o(N (a)) = o(G) X o(G)
= 0.
o(N (a))
Derivation of a class equation: N (a)̸=G

Theorem 5.2. For a finite group G, By class equation of finite group G, o(G) = o(Z(G)), where, Z(G) is a subgroup
of G. G = Z(G). Thus G is abelian if and only if G = Z(G).
X o(G)
o(G) = o(Z(G)) + Excercise 5.2. Verify the class equation for
o(N (a))
N (a)̸=G

(i) Symmetric group S3 , and


Proof. The conjugacy relation ∼ is defined in G by a ∼ b if a = x−1 bx ∀ x ∈ G
becomes an equivalence relations. Hence the equivalence class C(a) = {x−1 a : (ii) Dihedral group D8 .
x ∈ G}. Which are either identical or disjoint partition of G. i.e., G = ∪a∈G Ca . Solution.
Therefore
(i) Let ρ0 = 1, ρ1 = a, δ1 = b. Then S3 = {ρ0 , ρ1 , ρ2 , δ1 , δ2 , δ3 } become S3 =
{1, a, a2 , b, ab, ba}, where a3 = b2 = 1, a2 b = ba and ba2 = ab etc. Thus
X o(G)
S3 = ⟨a, b⟩ : a3 = b2 = 1, a2 b = ba, ba2 = ab, ... etc., is the generator
X
o(G) = Ca =
a∈G
a∈G
o(N (a)) relation form of symmetric group S3 . By class equation of finite group,
X o(G) X o(G) wehave
= + −→ (1)
o(N (a)) o(N (a))
N (a)∈G N (a)̸=G
5. UNIT : CLASS EQUATION OF FINITE GROUPS 65 5. UNIT : CLASS EQUATION OF FINITE GROUPS 66

dihedral group D8 . By class equation of finite group, wehave

−1 −1 2 −1 2 −1 −1 −1
C[1] ={1 11, a 1a, (a ) 1a , b 1b, (ab )1ab, (ba) 1ba} C[1] ={1} for 1 ∈ Z(D8 )
={1, 1, 1, 1, 1, 1} = {1}. C[a] ={a, a2 } = C[a3 ] for a, ae ∈
/ Z(D8 )
C[a2 ] ={a2 } for a2 ∈ Z(D8 )
Therfore, C[1] = 1. i.e., 1 is self-conjugate and 1 ∈ Z(S3 ). 2 2 2
C[b] ={b, a b} = C[a b] for b, a b ∈/ Z(D8 )
C[ab] ={ab, ba} = C[ba] for ab, ba ∈ / Z(D8 ).
C[a] = {1−1 a1, a−1 aa, (a2 )−1 aa2 , b−1 ab, (ab−1 )a(ab), (ba)−1 a(ba)}
∴ C[a] = {a, a2 }, not a self-conjugate. Therefore, (D8 ) = {1, a2 } are self-conjugacy element (i.e., 1, a2 ∈ Z(D8 ))
∴ C[a2 ] = {a, a2 }, a ∈
/ Z(S3 ). and a2 ∈ / Z(S3 ). and C[a], C[b] and C[ab] are distinct conjugacy classes. Therefore, wehave
Similarly, C[b] = {1−1 b1, a−1 ba, (a2 )−1 ba2 , b−1 bb, (ab−1 )b(ab), (ba)−1 b(ba)}
= {b, ba, ab} = C[ba]. N (a) ={1, a, a2 , a2 a3 }
N (b) ={1, a2 , b, ab}, and
Therefore Z(S3 ) = {1} and C[a], C[b] are the distinct conjugacy classes. So N (ab) ={1, a2 , ab, ba}.
we have to calculate N (a) and N (b).
X o(D8 )
i.e., N (a) = {1, a, a2 }, N (b) = {1, b} (it has to be 2- element). Consider, o(Z(D8 )) + = o(G)
o(N (x))
N (x)̸=G
X o(S3 )
Consider, o(Z(S3 )) + = o(G) (Here x = a, b, ab. i.e., x represents distinct conjugacy classes.) o(Z) +
o(N (x))
N (x)̸=G P o(D8 ) 8 8 8
x=a,b,ab ⇒ 2 + + + = 8 = o(D8 ).
o(N (x)) 4 4 4
(Here x = a, b. i.e., x represents distinct conjugacy classes.) Excercise 5.3. Let G be a group and e ̸= a ∈ G such that o(G) = finite. Suppose
X o(S3 ) G has only two conjugate classes. Then show that G is a finite group of order 2.
⇒ o[{1}] +
x=a,b
o(N (x)) Solution: Let e ̸= b ∈ G. Since G has only 2 conjugate classes, namely e and
C(a), b ∈ C(a) ∴ b = g −1 ag for some g ∈ G. Therefore o(b) = o(a) for some
o(S3 ) o(S3 ) g ∈ G. Suppose o(G) = mn, m > 1, n > 1. Then o(am ) = m. Since order of all
⇒1+ + non identity elements in G is same, o(am ) = mn.
o(N (a)) o(N (b))

6 6 ∴ n = mn =⇒ m = 1; a contradiction
⇒1+ + = 6. ∴ o(G) = p = prime.
3 2
∴ o(b) = pf oralle ̸= b ∈ G.
(ii) Let ρ0 = 1, ρ1 = a, δ1 = b. Then D8 = {ρ0 , ρ1 , ρ2 , ρ3 , δ1 , δ2, β1 , β2} becomes
D8 = {1, a, a2 , a3 , b, ab, a2 b, a3 b}, where a4 = b2 = 1, a2 b = ba. Thus, D8 =
Suppose p ̸= 2. Then a2 ̸= e =⇒ a2 ∈ C(a). Therefore, a2 = g −1 ag for some g ∈
⟨a, b⟩, a4 = b2 = 1, a3 = b = ba, ... etc., is the generator relation form of
G =⇒ (a2 )2 = (g −1 ag)2 =⇒ = g −1 a2 g =⇒ = g −1 (g −1 ag)g =⇒ = g −2 ag 2 =⇒
2
a2 = g −2 ag 2 .
In this way, we have a2 = g −p ag p . Since o(g) = o(a) = p =⇒ a2 = eae = a =⇒
p p

a2 −1 = e =⇒ o(a) = p|2p − 1.
p

By Fermet’s Theorem, p|2p − 2 =⇒ p(2p − 1) − (2p − 2) = 1, which is a

5. UNIT : CLASS EQUATION OF FINITE GROUPS 67 5. UNIT : CLASS EQUATION OF FINITE GROUPS 68

contradiction. Therefore p = 2 =⇒ o(G) = 2. So o(b) = 2 for all e ̸= b ∈ G =⇒ 1. Show that the centralizer Z(G) is a subgroup of a group G.
G is abelian. Hence, every conjugate class in G is of length one. Since G has only
G
2 classes, order of G is 2. 2. If G is a group, Z(G) its center, and if is cyclic, prove that G must
Note: There exist an infinite groups in which no non-trivial element has finite Z(G)
be abelian.
order and group has only 2 conjugate classes. Therefore, it is necessary to assume
that there exist e ̸= a ∈ G such that o(a) = finite. 3. List all the conjugate classes in S4 , find the Ca ’s and verify the class equation.

Excercise 5.4. Let H ̸= G be a subgraph of a finite group G. Show that G cannot 4. Find the number of conjugates of (1,2) (3,4) in Sn for n ≥ 4.

be expressed as union of conjugates of H. 5. Find the form of all elements commutating with (1,2) (3,4) in Sn for n ≥ 4.
o(G) 6. If N is a normal subgroup of G and a ∈ G, show that every conjugate of a
Solution. The number of conjugates of H in G is given by . So
o(N (H)) in G is also in N .
7. Let x, y ∈ Sn . Then show that x and y are conjugate in Sn if and only if
o(G)
o(∪x−1 Hx) ≤ (o(H) − 1) + 1 they have the same cycle structure.
o(N (H))
o(G) 8. Verify the class equation for the Hamiltonian quternion Q8 , where Q8 =
≤ (o(H) − 1) + 1 as H ≤ N (H) {±1, ±i, ±j, ±k : i2 = j 2 = k 2 = −1, ij = −ji, jk = −jk = i · · · }. [Hint.
o(H)
o(G) C[1] = {1} and C[−1] = {−1}. Therefore 1, −1 ∈ Z(Q8 ), C[−1] = {−1}.
= o(G) − +1 Also, C[i] = {i, −i} = C[−i], C[j] = {j, −j} = C[−j] and C[k] = {k, −k} =
o(H)
C[−k]. Further, N (i) = {±1, ±i}, N (j) = {±1, ±j} and N (k) = {±1, ±k}.
o(G) 8 8 8
≤ o(G) − 2 + 1 as ≥2 Finally, wehave o(Q8 ) = 2 + + + = 8].
o(H) 4 4 4
= o(G) − 1 < o(G)
REFERENCES
Thus, the group G cannot be written as union of conjugates of H.
1. I. N. Herstein: Topics in Algebra, 2nd Edition, John Wiley and Sons, 2008.
2. Surjeet Singh and Qazi Zameeruddin: Modern Algebra, 8th Edition, Vikas
Publishing House, 2021.

SUMMARY 3. N. Jacobson: Basic Algebra-I, 2nd Edition, Dover Publications, 2009.

Finite groups are the modulo multiplication groups, point groups, cyclic groups, 4. M. Artin : Algebra, 2nd Edition, Prentice Hall of India, 2015.
dihedral groups, symmetric groups, alternating groups, and so on. The equivalence 5. Darek F. Holt, Bettina Eick and Eamonaa A. O’brien: Handbook of com-
classes determined by this relation are referred to as the conjugacy classes of G. putational group theory, 1st Edition, Chapman & Hall/CRC Press, 2005.
A group G is the disjoint union of its conjugacy classes. Moreover the conjugacy
class of the identity element of G contains no other element of G. 6. J. B. Fraleigh : A first course in abstract algebra, 7th Edition, Addison-
Wesley Longman, 2002.
KEYWORDS
7. D. S. Dummitv and R.M. Foote: Abstract Algebra, 2nd Edition, John Wiley
Conjugate elements, Normalizer, Centralizer, Equivalence relation, Conjugacy & Sons, 2003.
class, Self-conjugacy class, Class equation of finite group.
8. V. K. Khanna and S. K. Bhambri: A course in Abstract algebra, 4th Edition,
Vikas Publishing House Pvt Ltd., 2013.
ASSESSMENT / TERMINAL QUESTIONS
69 6. UNIT : APPLICATIONS OF CLASS EQUATION 70

6 UNIT : Applications of Class Equation (ii) Every group of prime order, cyclic, abelian p group.

(iii) The group (Z4 , 04 ) (Integer modulo 4) of order 4, cyclic and abelian 2 group

OBJECTIVES (4 = 22 ).

After studying this unit you will be able to: (iv) The Klein-4 group K4 is a cyclic, abelian 2 group.

• understanding the concepts of prime number in class equation; (v) The dihedral group D8 of order 8 is an acyclic and non-abelian two group
because (8 = 23 ).
• define the concepts of prime group (p-group) with examples;
(vi) The hamiltonian quaternion Q8 is acyclic and non-abelian.
• understanding fundamental properties of p-group;
(vii) The symmetric group S3 is the smallest non-abelian group not a p-group
• explain the concepts of p-groups by applying class equation of finite groups;
(because it contains elements of order 2 and 3).
• identify the group of order is abelian or cyclic by applying class equation of
We observe that a group of order 22 is abelian but a group of 23 need not be
finite groups;
abelian. However both D8 and Q8 , though non-abelian have a non-trivial centre.
• solve problems by applying class equation of finite groups. Note that S3 has a trivial centre.

Theorem 6.1. If o(G) = pn , where p is a prime number, then Z(G) ̸= {e} (or.
INTRODUCTION Every p group has a non-trivial centre).

The conjugacy class of a group forms an equivalence relation which permits the Proof. From the class equation of finite group, we have
partitioning of a finite group G in the conjugacy classes of every elements of the
finite group. One of the applications of the class equation is to verify whether the
X O(G)
o(G) = o(Z(G)) + −→ (1).
O(N (a))
given group with prime power order is abelian or not / cyclic or not. N (a)̸=G

The structure of p-groups : Clearly, N (a) is a proper subgroup [N (a) ̸= G] of G under the condition
N (a) ̸= G. So, by Lagrange’s theorem, we have o(N (a)) | o(G). Therefore
Definition 6.1. A finite group G is said to be p group for some prime p if every
o(N (a)) = pna (say) where na < n.
element of G has order a power of p. In such a case, G itself is a group of prime pn
Therefore from (1) we have, pn = o(Z(G)) +
P
na <n
power order. pna
X pn
Note 6.1. ⇒ pn − = o(Z(G)).
pna
na <n

(i) o(p) = 1 = p0 .

6. UNIT : APPLICATIONS OF CLASS EQUATION 71 6. UNIT : APPLICATIONS OF CLASS EQUATION 72

Since each term of the sum, in the left hand side is divisible by p, it follows Let now H be a non-trivial subgroup of G i.e. H ̸= e, G. Since G is abelian,
pn
that, p | pn −
P
na <n na . Hence, p | o(Z(G)), where o(Z(G)) > 0 as e ∈ Z(G) H is normal in G. If p|o(H), then, by induction hypothesis as o(H) < o(G), H
p
i.e., o(Z(G)) is a positive integer divisible by a prime p. is abalian. There exist x ∈ H such that o(x) = p, x ∈ H =⇒ x ∈ G, result is
Therefore o(Z(G)) > 1, i.e., there must be an element other than e in Z(G). again true.
 
G
Thus Z(G) ̸= {e}. Let p ∤ o(H). Since o(G) = o(G/H).o(H) and p|o(G), we find p|o H
 
G
Corollary 6.1. If o(G) = p2 , where p is a prime number, then G is abelian (or, But p ∤ o(H), hence we have p|o(G/H). Also o H < o(G) as H ̸= e and G is
G G
Every group of order p2 is always an abelian group ). abelian means H
is abelian. So, by induction hypothesis H
has an element Hy of
order p, we have (Hy) = H =⇒ Hy = H =⇒ y = H =⇒ (y p )t = e, where
p p p
Proof. Since, the centre Z(G) of G is a subgroup of G, by Lagranges theorem
t = o(H). Also, this implies (y t )p = e =⇒ o(y t )|p =⇒ o(y t ) = 1 or p.
o(Z(G)) | o(G). But o(G) = p2 . Therefore o(Z(G)) | p2 .
If y t = e (i.e.o(y t ) = 1), then Hy t = He = H
Thus o(Z(G)) = 1 or p or p2 .
From the above theorem, it follows that o(Z(G)) > 1.
=⇒ (Hy)t = H
Therefore, o(Z(G)) = p or p2 .
=⇒ o(Hy)|t
Now, if o(Z(G)) = p, then there exists an element a ∈ G such that a ∈
/ Z(G),
=⇒ p|t = o(H), a contradiction.
then N (a) in G satisfies Z ⊂ N (a) ⊂ G, since a ∈ N (a) and a ∈
/ Z(G). Then we
have o(N (a)) > o(Z(G)) = p.
Therefore, o(y t ) = p, y t ∈ G. So result is true in this case.
Again, by Lagranges theorem o(N (a)) | [o(G) = p2 ].
By induction, result is true for all abelian graups.
Therefore, we must have, p2 = o(N (a)) = o(G).
Let now G be any group. We again use induction on o(G). The result is vacuously
Therefore, a ∈ Z(G), because a ∈ Z(G) if and only if N (a) = G. This is a
true for o(G) = 1. Assume result is true for all groups with order less then o(G).
contradiction.
If o(T ) < o(G) and p|o(T ) then by induction hypothesis, there exist x ∈ T such
Therefore o(Z(G)) ̸= p. So we must have o(Z(G)) = p2 = o(G) ⇒ Z(G) = G.
that o(x) = p. So, result is true in this case.
Therefore G is abelian.
Assume p ∤ o(T ) for all o(T ) < o(G), Consider class equation of G.
Theorem 6.2 (Cauchy’s theorem). Let G be a finite group and suppose p is a
X o(G)
prime such that p|o(G), then there exist x ∈ G such that o(x) = p. o(G) = o(Z(G)) + .
o(N (G))
a∈Z(G)
Proof. We first prove the result when G is abelian. We prove if by induction on
n = o(G). Result is vacuously true when n = 1. Assume it to be true for all groups Now a ∈ Z(G) =⇒ N (a) < o(G), we have
o(G) o(G) P o(G)
having order less than o(G). If G has no non-trivial subgroup, then G is cyclic p ∤ o(N (a)) =⇒ p| o(N (a))
(as o(G) = o(N (a))
.o(N (a)).Therefore p| o(N (a))
.
a∈Z(G)
/
P o(G)
group of prime order. Since p|o(G), o(G) = p, G = ⟨x⟩ such that o(x) = o(G) = p. Since p|o(G), we have p|o(G) − = o(Z(G)).
o(N (a))
a∈Z(G)
/
So result follows.
6. UNIT : APPLICATIONS OF CLASS EQUATION 73 6. UNIT : APPLICATIONS OF CLASS EQUATION 74

But p ∤ o(T ) forall o(T ) < o(G) and Z(G) = (G) =⇒ G is abelian. Solution: Let g = g1 , g2 , ..., gn be distinct elements of a group G such that
But result is true for abelian groups. Hence, by induction, result is true for all gi−1 xgi = y. Let S = {g = g1 , g2 , ...gn }.
groups. We show that S = N (x)g.
Suppose s ∈ S then s = gi for some i, 1 ≤ i ≤ n.
Theorem 6.3. Let G be a finite group. Then G is a p-group if and only if
If s = g1 = g, then s = g = eg ∈ N (x)g.
o(G) = pn .
If s ̸= g1 then s = gi , i ̸= 1 and g −1 xg = gi−1 xgi
Proof. Suppose G is a p-group. Let q be a prime dividing o(G). By Cauchy’s =⇒ gi g −1 x = xgi g −1
theorem there exists x ∈ G such that o(x) = q. But o(x) = pr as G is a p- =⇒ gi g −1 ∈ N (x)
group. Therefore q = pr =⇒ q = p. So, p is the only prime dividing o(G). =⇒ gi ∈ N (x)
Thus o(G) = pn . Conversely, let o(G) = pn . let x ∈ G. Then o(x)|o(G) = pn =⇒ s ∈ N (x)g or that S ⊆ N (x)g.
=⇒ o(x) = pr . Therefore, every element of G has order which is some power of Again z ∈ N (x)g =⇒ z = hg, h ∈ N (x)
p. So, G is a p-group. =⇒ z −1 xz = g −1 h−1 xhg

Note 6.2. =⇒ z −1 xz = g −1 xg as xh = hx
=⇒ z −1 xg = y
(i) Any finite p-group has non-trivial centre.
=⇒ z = g1 for some i

(ii) A p-group may or may not be abelian. =⇒ z ∈ S


=⇒ N (x)g ⊆ S
Excercise 6.1. Suppose a ∈ G has only two conjugates in G then show that N (a)
Hence S = N (x)g, and thus o(S) = o(N (x)g) = o(N (x)).
is normal subgroup of G.
Solution: Let a, g −1 ag be two conjugates of a in a group G. We show Note 6.3. As ggi−1 x = xggi−1 for all i = 1, ..., n ggi−1 ∈ N (x) for all i. Therefore

G = N (a) ∪ N (a)g. N (x)g = N (x)gi for all i.

Let x ∈ G. Consider x−1 ax = a or g −1 ag.


Excercise 6.3. Suppose X is a conjugate class of non- trivial element of a group
If x−1 ax = a, then xa = ax =⇒ x ∈ N (a).
G. Let T ∈ Aut(G). Show that T (X) = {T (x)|x ∈ X} is a conjugate class of
If x−1 ax = g −1 ag, then xg −1 a = axg −1 .
elements of G.
This implies that xg −1 ∈ N (a). Therefore x ∈ N (a)g.
Hence G = N (a) ∪ N (a)g and thus index of N (a) in G is 2, showing thereby that
N (a) is normal subgraph of a group G.

Excercise 6.2. Let G be a finite group and x, y be conjugate elements of G. Show


that the number of distinct elements g ∈ G such that g −1 xg = y is o(N (x)).

6. UNIT : APPLICATIONS OF CLASS EQUATION 75 6. UNIT : APPLICATIONS OF CLASS EQUATION 76

Solution. Let X = C(a), a ̸= x. We show that T (X) = C(T a). Solution: Since G is non-abalian, there exist a ∈ G, such that Z(G) ⊂ N (a) ⊂ G
(by a group of order p2 is ableian, where p is prime).
Let y ∈ T (X) =⇒ y = T x, x ∈ X = C(a) Now o(Z(G))|o(G) = p3 =⇒ o(Z(G)) = 1, p, p2 or p3
= T (g −1 ag), g ∈ G Similarly, o(N (a)) = 1, p, p2 or p3 .
= T (g)−1 T (a)T (g) ∈ C(T (a)) o(Z(G)) ̸= 1 =⇒ o(Z(G)) ̸= p3 as Z(G) ̸= G

∴ T (X) ⊆ C(T a) so o(Z(G)) = p or p2 .


Similarly, o(N (a)) = p or p2 and as Z(G) ⊂ N (a).
Again z ∈ C(T a) =⇒ z = h−1 T (a)h, h ∈ G
We find o(Z(G)) = p and o(N (A)) = p2 .
(T h1 )−1 T aT h
Now, let k be the total number of conjugate clesses. Since G = ∪a∈G C(a).
(as T is onto =⇒ h = T h1 , h1 ∈ G) P P P
o(G) = o(C(a)) = + +a ∈/ Z(G)o(C(a))
a∈G a∈Z(G)
T h−1
1 T aT h 3
P
i.e., p = o(Z(G)) + o(C(a))
a∈Z(G)
/
T (h−1 ah)
As number of conjugate classes when a ∈ Z(G) is o(Z(G)) = p
∈ T (C(a)) = T (X) [a ∈ Z(G) ⇔ C(a) = {a}, i.e., o(C(a)) = 1]
T (X) = C(T a). So remaining classes are k − p, each will have given by
o(G) p3
o(C(a)) = o(N (a))
= p2
=p
Hence T (X) is a conjugate class of a group G. The following theorem helps us to Hence p3 = p + (k − p)p =⇒ k = p2 + p − 1.
determine the order of conjugate class of an element.
Excercise 6.6. If index of Z(G) in a group G is n then show that any conjugate
Excercise 6.4. Show that an abelian group of order pq (i.e., o(G) = pq) is cyclic,
class has at most n-elements.
where p and q are distinct prime numbers.
o(G) o(G)
Solution. By Cauchy’s theorem, there exist a, b ∈ G such that o(a) = p, o(b) = q. Solution. We have n = and o(C(a)) = .
o(Z(G)) o(N (a))
Also, as (p, q) = 1, ab = ba, we have o(ab) = 0(a).o(b) = pq. i.e., G has an element Since Z(G) ⊆ N (a) always o(Z(G)) | o(N (a)).

ab of order equal to o(G). Hence G is cyclic. This implies o(N (a)) = k.o(Z(G)). Therefore

Note 6.4. In view of above problems, abelian groups of order 6, 10, 15 etc., are o(G)
o(C(a)) =
all cyclic. o(N (a))
n.o(Z(G))
G
 o(G) p2 =
Note 6.5. If o(Z(G)) = p, then o Z(G)
= o(Z(G))
= p
= p, a prime. This k.o(Z(G))
G n
implies Z(G)
is cyclic. Therefore G is abelian. = .
k
Excercise 6.5. Let G be a non-abelian group of order p3 . Determine o(Z(G))
Thus, maximum value of o(C(G)) is when k = 1, proving the result.
and k = number of conjugate classes of G.
6. UNIT : APPLICATIONS OF CLASS EQUATION 77 6. UNIT : APPLICATIONS OF CLASS EQUATION 78

Excercise 6.7. Let G be a group of order pn . Then show that o(Z(G)) > 1, where 3. For any primes p and q, show that any group of order pq is not simple. where
p is a prime, n is a positive integer and Z(G) is a centralizer of G. a simple group is a nontrivial group whose only normal subgroups are the
trivial group and the group itself.
Solution. If G = Z(G), o(Z(G)) = o(G) > 1. If G ̸= Z(G), then there exist
some a ∈ G such that a ∈
/ Z(G). Then N (a) ⊂ G [as a ∈
/ Z(G) =⇒ at ̸= ta for 4. Show that an abelian group of order 15 is cyclic.
some t ∈ G, that is t ∈
/ N (a), t ∈ G].
o(G) 5. Show that the symmetric group S3 is the smallest non-abelian group not a
Therefore o(N (a)) = pm , m < n. That is, = pn − m, n − m > 0. =⇒
n−m
Po(N (a)) p-group.
o(C(a)) = p = multiple of p Therefore a∈Z(G)
/ o(C(a))= multiple of p = kp,
(say). By class equation of G, we have pn = o(G) = o(Z(G)) + a∈Z(G) 6. If G is a finite non-abelian p-group, then show that p2 | o(Aut(G)).
P
/ o(C(a)).
This implies that o(Z(G)) = pn − kp = p(pn−1 − k) =⇒ p | o(Z(G)). Therefore
7. Let G be a finite group with the property that if H, K are two subgroups
o(Z(G)) > 1.
of G then either H ⊆ K or K ⊆ H. Show that G is cyclic p-group.
SUMMARY
8. Let G be a finite cyclic p-group. Show that if H, K be any two subgroups
The class equation implies a nontrivial finite p-group has a nontrivial center, or
of G then either H ⊆ K or K ⊆ H.
more generally that a nontrivial normal subgroup of a nontrivial finite p-group G
contains a nontrivial element of the center of G. (i.e., there are infinite groups REFERENCES

where every element has p-power order and the center is trivial, so the finiteness
1. I. N. Herstein: Topics in Algebra, 2nd Edition, John Wiley and Sons, 2008.
assumption on the group is important.) This has standard further consequences
for finite p-groups, although they are no longer direct consequences of the class 2. Surjeet Singh and Qazi Zameeruddin: Modern Algebra, 8th Edition, Vikas

equation. Publishing House, 2021.

3. N. Jacobson: Basic Algebra-I, 2nd Edition, Dover Publications, 2009.


KEYWORDS
4. M. Artin : Algebra, 2nd Edition, Prentice Hall of India, 2015.
Application of Class equation of finite group, prime group (or p- group), prime
5. Darek F. Holt, Bettina Eick and Eamonaa A. O’brien: Handbook of com-
power order, Cauchy’s theorem.
putational group theory, 1st Edition, Chapman & Hall/CRC Press, 2005.

ASSESSMENT / TERMINAL QUESTIONS


6. J. B. Fraleigh : A first course in abstract algebra, 7th Edition, Addison-
Wesley Longman, 2002.
1. By using p-group, verify wheather group of order 36 is abelian? Justify.
7. D. S. Dummitv and R.M. Foote: Abstract Algebra, 2nd Edition, John Wiley
2. Show that every group of order 49 is an abelian group.
& Sons, 2003.

6. UNIT : APPLICATIONS OF CLASS EQUATION 79 80

8. V. K. Khanna and S. K. Bhambri: A course in Abstract algebra, 4th Edition, 7 UNIT : Sylow’s Theorems
Vikas Publishing House Pvt Ltd., 2013.

OBJECTIVES

After studying this unit you will be able to:

• define the concepts of the Sylow’s theorem;

• understanding the cconjugacy relation of the Sylow subgroups;

• identifying the number of Sylow subgroups;

• explain the p-sylow subgroups with suitable examples;

• identify the group of order is abelian or cyclic by applying class equation of


finite groups;

• solve problems by applying class equation of finite groups.

INTRODUCTION

According to Lagrange’s theorem the order of a subgroup of a finite group divides


the order of the group. But if an integer k divides O(G), then G need not have
a subgroup of order k. As an example for this fact we ca see that A4 has no
4!
subgroup of order 6 though 6|o(A4 ) (Note : o(A4 ) = = 12).
2
However, we prove, in sylow theorem, that if a number, which is a power of a
prime, divides the order of the group G, then G has a subgroup of prime power
order.

Sylow’s First Theorem :


7. UNIT : SYLOW’S THEOREMS 81 7. UNIT : SYLOW’S THEOREMS 82

Theorem 7.1. If p is a prime number and pα | o(G). Then G has a subgroup of into disjoint equivalence classes.
order pα .
P
Therefore we have o(M) = M CM , where the summation runs over the
elements of M and only one M is taken from each equivalence class.
Proof. Since pα | o(G), we have o(G) = pα m for some integer m. Let pr | m and
We claim that there exist atleast one equivalence class C(M ) such that pr+1 ∤
pr+1 ∤ m. Let M denote the set of all subset of G containing pα element. Then,
(CM ).
consider
For, if pr+1 ∤ (CM ) for some M , then
 
pα m  
o(M) =  
pα m

X
pr+1 | (CM ) = O(M) =  
α
(pα m)! M p
=
(pα m − pα )!pα ! This contradicts (1),
(pα m)(pα m − 1) · · · (pα m − i) · · · (pα m − pα + 1)
= . So, let C(MD ) = {M1 , M2 , · · · , Mn } be such an equivalence class in M where
(pα )(pα − 1) · · · (pα − i) · · · (pα − pα + 1)
pr+1 | n. By our definition of equivalence in M , for each i, 1 ≤ i ≤ n.
Note that the power of p which divides (pα − i) is same as that of p which
We have Mi g = Mj for some g ∈ G and 1 ≤ j ≤ n.
divides (pα − i). Hence these powers of p gets cancelled except the power of p
Now, we define H, by H = {g ∈ G/M1 g = M1 }, since M1 e = M1 , where e is
which divides m in the numerator.
identity element in G, e ∈ H.
Because   Therefore H is non empty. Let a, b be any two elements of H. Then we have
pα m
pr |(pα m) and pr+1 ∤   −→ (1) M1 a = M1 and M1 b = M1 .
α
p
Therefore M1 ab = M1 b = M1 ⇒ ab ∈ H. Thus the closure law holds in H.
In M, we define M1 ∼ M2 if there exists an element g ∈ G such that M1 g = M2 .
Therefore H is subgroup of G. Next, suppose x and y lie in the same right coset
The relation M1 ∼ M2 is an equivalence relation. Since M1 e = M2 , where e is
of H. Then y ∈ Hx (∵ x ∈ Hx). Therefore y = hx, for some h ∈ H. But, then
identity element in G, we have M1 ∼ M2 , therefore the relation is reflexive.
M1 y = M1 hx = M1 x (∵ h ∈ H = M1 h = M1 ). i.e., x and y give the same element
Let M1 ≈ M2 . Then M1 g = M2 for some g ∈ G. (∵ M1 = M2 g −1 , where
in the equivalence class C(M ).
g −1 ∈ G). We have M2 ∼ M1 . Therefore relation is symmetric. Let M1 ∼ M2
Suppose x and y lie in the distinct right coset of H. then we claim that
and M2 ∼ M3 . Then M1 g = M2 and M2 g = M3 for some g1 , g2 ∈ G. Therefore
M1 x ̸= M1 y.
M3 = M2 g2 = M1 g1 g2 , where g1 , g2 ∈ G, we have M1 ∼ M3 .
For, if M1 x = M1 y, we have M1 xy −1 = M1
Therefore the relation is Transitive.
Therefore xy −1 ∈ H ∵ x ∈ Hy. i.e., x and y lie in the same right coset Hy of
Hence the relation ‘M1 ∼ M2 ’ is an equivalence realtion. We denote the
H, which is a contradiction.
equivalence class M ∈ M by C(M ). i.e., C(M ) = {M 1 ∈ M/M 1 ∼ M }
Thus there is a one-one correspondence between the elements in C(M1 ) and the
Let o(C(M )) = CM . This equivalence relation induces a decomposition of M
right cosets of H in G.

7. UNIT : SYLOW’S THEOREMS 83 7. UNIT : SYLOW’S THEOREMS 84

o(G) (ii) Let x ∼ y. Then y = axb for some a ∈ A, b ∈ B. Therefore a−1 y, b−1 = x
Therefore CM 1 = or CM 1 · o(H) = o(G).
o(H)
Therefore n · o(H) = o(G) −→ (2) [∵ CM 1 = n]. , where a−1 ∈ A, b−1 ∈ B. Therefore y ∼ x. Therefore the relation is

Finally, since pr+1 ∤ n and pα+r | pα m = n · o(H), it must be follow that Symmetric.

pα | O(H), and so o(H) ≥ pα . −→ (3) (iii) Let x ∼ y and y ∼ z. Then y = axb, z = cyd for some a, c ∈ A and
However, if m1 ∈ M1 then for all h ∈ H, m1 h ∈ M1 . Thus, M1 has atleast b, d ∈ B therefore z=cyd=caxbd, where ca ∈ A and ad ∈ B. Therefore
O(H) elements. But M1 was a subset of G containing pα elements. Thus pα ≥ x ∼ z. Therefore the relation is Transitive.
o(H). −→ (4)
Hence, (A, B) relation is an Equivalence relation.
From (3) and (4) it follows that O(H) = pα .
Thus, G has a subgroup H of order pα Note 7.2. The equivalence class of x ∈ G is the set AxB = {axb/a ∈ A, b ∈ B}.
Hence the theorem. We call the set AxB, a double coset of A, B ∈ G so, G = Ux∈G AxB (i.e., partition
G into mutually disjoint equivalence classes)
Corollary 7.1. If pm | o(G), pm+1 ∤ o(G), then the group G has a subgroup of
order pm . Example 7.2. Let G = S3 = ⟨a, b⟩ : a3 = a2 = 1, a2 b = ba. If A = {1, a, a2 },
B = {1, b} is a subgroup of G, then AxB = {axb/a ∈ A, b ∈ B} where x ∈ G. For
Note 7.1.
x = 1; AxB = {1, b, a, ab, a2 , a2 b} = AB = S3 .
(i) The above corollary is usually called the Sylow’s First Theorem. For x = a;

(ii) A subgroup of a group G of order pα , where pm | o(G) but pm+1 ∤ o(G) is


AxB = AaB ={1a1, 1ab, aa1, aab, a2 a1, aa ab}
called a p-sylow subgroup of G.
={a, ab, a2 , a2 b, a3 , a3 b}

Example 7.1. If o(G) = 54 = 33 · 2, then 33 | o(G) but 34 ∤ O(G). By the Sylows ={a, ab, a2 , ba, 1, b}
theorem G has a subgroup H of order 33 . H is called a 3-sylow subgroup of G. =S3 .

Definition 7.1. Let G be a group, A < B subgroup of G. If x, y ∈ G, we define


For x = b;
x ∼ y if y = axb for some a ∈ A, b ∈ B. or Let G be a finite group with two
subgroups A and B. Call two elements x, y of G to be (A ⊂ B) related, if y = axb
AxB = Aa2 B ={1a2 1, 1a2 b, aa2 1, aa2 b, a2 a2 1, a2 a2 b}
for some a ∈ A and b ∈ B.
={a2 , a2 b, a3 , a3 b, a4 , a4 b}
We verify that, this relation is an equivalence relation in G. ={a2 , ba, 1, b, a, ab}

(i) Since x = exe, where e ∈ A and e ∈ B we have x ∼ x. Therefore the relation =S3 .
is Reflexive.
7. UNIT : SYLOW’S THEOREMS 85 7. UNIT : SYLOW’S THEOREMS 86

For x = b, AxB = AbB = {b, 1, ab, a, ba, a2 } = S3 . From (1) and (2), we have
All the double cosets AxB are just S3 . Like wise BxA = S3 for all x ∈ S3
(∵ A∆S3 ). (what are BxB ? BxB = {x, xb, bx, bxb} =
̸ S3 ) o(AxB) =o(AxBx−1 )
2 2 2
BaB = {a, ab, ba, a }; Ba B = {a , ba, ab, a}, B1B = {1, b}; BbB = {1, b} = o(A)o(xBx−1 )
=
o(A ∩ xBx−1 )
B, BabB = {a, a2 , ab, ba}; BbaB = {a, a2 , ab, ba}. o(A)o(B)
= .
Therefore S3 = (BaB) ∪ (BbB). (i.e., union of disjoint double cosets) o(A ∩ xBx−1 )

Lemma 7.1. If A and B are finite subgroups of G, then o(H)o(K)


(By one of the previous theorem o(HK) = )
o(H ∩ K)
O(A)O(B) Hence, the lemma follows.
O(AxB) =
O(A ∩ xBx−1 )
Note 7.3. The above lemma is very significant and has a strong consequences
Proof. First define a function ϕ : AxB → AxBx−1 by ϕ(axb) = axbx−1 ∀ axb ∈ giving the Sylow second theorem.
AxB in ϕ(a1 xb1 ) = ϕ(a2 xb2 ).
Sylow’s Second Theorem:
⇒ a1 xb1 x−1 = a2 xb2 x−1 , ⇒ a1 xb1 = a2 xb2 . Therefore ϕ is one-one.
Next, for any elements a1 xb1 x−1 ∈ A1 xB1 x−1 , there exists axb ∈ AxB such Theorem 7.2. If G is a finite group, p a prime and pn | o(G) but pn+1 ∤ o(G),
that, ϕ(axb) = a1 xb1 x−1 . Therefore ϕ is onto. then any two subgroups of G of order pn are conjugate. In other words, any two
Therefore, ϕ is a bijective. p-sylow subgroups are conjugates to each other.
Since AxB and AxBx−1 are finite sets. Hence
Proof. Let A and B be any two p-sylow subgroup of a group G. Then o(A) =
−1 pn = o(B). We Want to show that A = gBg −1 for every x ∈ G.
o(AxB) = o(AxBx ) −→ (1)
Decompose G into double cosets of A and B. i.e., G = ∪x AxB, where, only

Since, B is a subgroup of a group G, so is xBx−1 . Hence one x is taken from each double coset. Then

X
o(AxB) = o(B) −→ (2) o(G) = o(AxB) −→ (1)
x

Since, A ̸= xbx−1 , for every x ∈ G, A ∩ xBx−1 is a proper subgroup of A.


o(A ∩ xbx−1 ) = pm , where m < n [∵ o(A ∩ xBx−1 )/o(A)]. By previous lemma, we

7. UNIT : SYLOW’S THEOREMS 87 7. UNIT : SYLOW’S THEOREMS 88

have Also, y −1 = x−1 n−1 . Therefore y −1 py = x−1 n−1 P nx = x−1 P x (∵ n−1 P n = P ).


Therefore, x and y yield the same conjugate of P .
o(A)o(B)
o(AxB) = Next, Let x and y belong to distinct right cosets of N (P ). Then we claim that
o(A ∩ xBx−1 )
n n
p p y −1 P y ̸= x−1 P x.
= m
p
For, if y −1 P y = x−1 P x, we have P yx−1 = yx−1 P . Therefore yx−1 ∈ N (P ).
=p2n−m .
Therefore y ∈ (N (P ))x.
Therefore y and x lie in the same right coset N (P )x of N (P ), which is a
Since 2n−m ≥ n+1. Therefore pn+1 | p2n−m . Therefore pn+1 | o(AxB)∀x ∈ G.
contradiction. Hence y −1 P y ̸= x−1 P x.
Therefore pn+1 | x o(AxB). Hence pn+1 | o(G), by(1). This is a contradiction.
P
In other words, x and y yield different conjugates of P . Thus there is a one-one
Therefore, we must have A = gBg −1 for some g ∈ G. In other words, A and
correspondence between the distinct conjugates of P and the distinct right coset
B are conjugates to each other.
of N (P ). Since all P -sylow subgroups of G are conjugates to P , we have
o(G)
Note 7.4. The above theorem (Sylow second theorem) can also state that ”any The number of p-sylow subgroups = .
o(N (P ))
two p-sylows subgroups of G (for the some prime p) are conjugate, that is, if A Hence the Lemma follows.
and B are two p-sylows subgroups of G, then A = xBx−1 for some x ∈ G”.

Corollary 7.2. A p-sylow subgroup of a group G is unique if and only if it is


normal.

Proof. Suppose A is the unique p-sylow subgroup of G. By above theorem, A =


xAx−1 or Ax = xa x ∈ G. Therefore A is normal subgroup of a group G.
Conversely, If A is normal subgroup of a group G, then A = xAx−1 . But xAx−1 =
B for a p-sylow subgroup B of a group G.
Therefore A = B, i.e., A is the unique p-sylow subgroup of G.
o(G)
Lemma 7.2. The number of p-sylow subgroups in G equals , where P is
o(N (P ))
any p-sylow subgroup of G. In Particular, this number is advisor of o(G).

Proof. We have N (P ) = {x ∈ G : xP = P x or P = x−1 P x}.


First, let x and y lie in the same right coset of N (P ). Then y = nx, where
n ∈ N (P ).
Therefore nP = P n or n−1 P n = P .
7. UNIT : SYLOW’S THEOREMS 89 7. UNIT : SYLOW’S THEOREMS 90

Sylow’s Third Theorem o(G) = o(N (P )) + P n+1 α.


o(G) pn+1 · α
Therefore =1+ . −→ (4)
Theorem 7.3. The number of p-sylow subgroup of G, for a given prime, is con- o(N (P )) O(N (P ))
Since, N (P ) is a subgroup of G, o(N (P ))|o(G). Therefore o(G) | o(N (P )) is
gruent to 1 modulo p.
an integer.
Proof. Let P be a p-sylow subgroup of G. We decompose G into double coset of Therefore P n+1 α | o(N (P )) is also an integer. Also, Since pn+1 ∤ o(G), P n+1
P
P and P . i.e., G = ∪x P xP . Then, o(G) = x o(P xP ), where only one x is taken can not divide o(N (P )).
from each double coset. Therefore But then P n+1 α | o(N (P )) must be divisible by p, so we can write (P n+1 · α |

X X o(N (P )) = kp , where k is an integer equation (4) becomes


O(G) = O(P xP ) + O(P xP ) −→ (1) O(G)
Therefore = 1 + kp .
x∈N (P ) x∈N
/ (P ) o(N (P ))
In other words, the number of p-sylow subgroups of G is congruent to 1 modulo
Now, if x ∈ N (P ), then xP = P x. p. Hence the theorem.
Therefore, P xP = P P x = P 2 x = P x.
Therefore, o(P xP ) = o(P x) = o(P ) = pn (where pn | o(G) but pn+1 | o(G)).
SUMMARY
Hence, in the first summation, we have o(P xP ) = o(P x). Therefore
In the field of finite group theory, the Sylow theorems are a collection of theorems
X X
O(P xP ) = O(P x) = O(N (P )). −→ (2) named after the Norwegian mathematician Peter Ludwig Sylow that give detailed
x∈N (P ) x∈N (P )
information about the number of subgroups of fixed order that a given finite group
(Which is merely the summation over distinct cosets of P in N (P ) and hence contains. In this unit, we discuss Sylow’s three theorems, the ideas developed in
equal to O(N (P ))). these theorems are very useful to known about the nature of a group by knowing
/ N (P ), then P ̸= xP x−1 .
Next, if x ∈ only its order.
Therefore o(P ∩ xpx−1 ) = pm , where m < n, therefore 2n − m ≥ n + 1
KEYWORDS
o(P )o(P ) P −1 .P −1
o(P xP ) = = = P 2n−m .
o(p ∩ xP x−1 ) pm p-sylow subgroups, Sylow’s first thoerem, Sylow’s second thoerem, Sylow’s third
Therefore, P n+1 | o(P xP ), for every x ∈
/ N (P ). Therefore thoerem.

X
P n+1 / o(P xP ). ASSESSMENT / TERMINAL QUESTIONS

1. State all sylow’s theorem and explain its significance.


o(P xP )) = P n+1 · α f or some α −→ (3)
P
Therefore
Substituting (2) and (3) in (1), we have 2. Define p-sylow subgroup of a group G with suitable example.

7. UNIT : SYLOW’S THEOREMS 91 7. UNIT : SYLOW’S THEOREMS 92

3. Discuss the nature of 3-sylow subgroup of a group of order 54. 7. V. K. Khanna and S. K. Bhambri: A course in Abstract algebra, 4th Edition,
Vikas Publishing House Pvt Ltd., 2013.
4. If H is a p-sylow subgroup of a group G and K = N (H), then show that
N (K) = K. 8. J. A. Gallian: Contemporary Abstract algebra, 9th Edition, Narosa Pub-

m m+1
lishing House. 2008.
5. If p | o(G), p ∤ o(G), then show that the group G has a subgroup of
order pm [Hint. Take α = m and r = 0 in the proof of the Sylow’s first
theorem. ]

6. Show that any two p-sylows subgroups of G (for the some prime p) are
conjugate, that is, if A and B are two p-sylows subgroups of G, then A =
xBx−1 for some x ∈ G [Hint. Solution is anologous to the Sylow’s second
theorem].

7. Show that the number of p-sylow subgroup np of G is of the form np = 1+pk


or np ≡ 1(modp) [Hint. Solution is anologous to the Sylow’s third theorem].

8. Explain the role of Lagrange’s theorem in Sylow’s theorem.

REFERENCES

1. I. N. Herstein: Topics in Algebra, 2nd Edition, John Wiley and Sons, 2008.

2. Surjeet Singh and Qazi Zameeruddin: Modern Algebra, 8th Edition, Vikas
Publishing House, 2021.

3. N. Jacobson: Basic Algebra-I, 2nd Edition, Dover Publications, 2009.

4. M. Artin : Algebra, 2nd Edition, Prentice Hall of India, 2015.

5. Darek F. Holt, Bettina Eick and Eamonaa A. O’brien: Handbook of com-


putational group theory, 1st Edition, Chapman & Hall/CRC Press, 2005.

6. J. B. Fraleigh : A first course in abstract algebra, 7th Edition, Addison-


Wesley Longman, 2002.
93 8. UNIT : APPLICATIONS OF SYLOW’S THEOREM 94

8 UNIT : Applications of Sylow’s Theorem Similarly, nq ≡ 1(modq) and nq /p we get, nq ≡ 1(modq) and nq = 1, p, nq = 1
and nq = q and p ≡ 1(modq), nq = 1 and q/p − 1. Therefore nq = 1 is possible (∵
p − 1 | q given that p < q. Therefore p − 1 ∤ q which is not possible).
OBJECTIVES
Hence np = 1 and nq = 1.

After studying this unit you will be able to: Suppose A and B are the p-sylow and q-sylow subgroups respectively. Since
an unique sylow subgroup is normal, we have A ◁ G and B ◁ G, where o(A) = p
• explain the p-sylow subgroups with suitable examples;
and o(B) = q.
• identify the group of order is abelian or not by applying class equation of We now verify that AB = BA. Let ab ∈ AB. Then ab = bb−1 ab = b(b−1 ab).
finite groups by using sylow’s thoerems; i.e., ab = ba1 ∈ BA where a1 = b−1 ab ∈ A (∵ A ◁ G). Therefore ab = ba1 ∈ BA.
Therefore ab ∈ AB ⇒ ba ∈ BA.
• identify the group of order is cyclic or not by applying class equation of finite
Therefore,
groups by using sylow’s thoerems;
AB ⊂ BA −→ (i)
• solve problems by applying sylow’s thoerems of finite groups.
Similarly, we can prove that
• understand the nature of group of order in terms of distinct primes.
BA ⊂ AB −→ (ii)

INTRODUCTION
. from (i) and (ii) we have AB = BA
Sylow theorems are very useful in studying finite nonabelian groups. Here we deal Now, To Show That G = AB = BA
o(A)o(B)
with the classical results of Sylow theorems and apply them to examples. Consider, o(AB) =
o(A ∩ B)

pq
APPLICATIONS OF SYLOW’S THEOREM o(AB) = −→ (1)
o(A ∩ B)

Theorem 8.1. Let G be a group of order pq, where p and q are primes with p < q But (A ∩ B) ⊂ A and (A ∩ B) ⊂ B
and q ≇ 1(modp). Then G is cyclic and abelian. So, Lagrange’s theorem

Proof. Let o(G) = pq. By Sylow first theorem, G has p-sylow subgroup of o(p) o(A ∩ B) o(A ∩ B)
and
and q-sylow subgroup of o(q). Also np ≡ 1(modp) and np | q, we get np ≡ 1(modp) o(A) o(B)
and np = 1, q. Therefore, np = 1 and np = q, q ≡ 1(modp) (i.e., 1 and q are 2 o(A ∩ B) o(A ∩ B)
⇒ and , where (p, q) = 1, by virtue of p < q and q ≇
possible value of np ). p q
1(modp). Therefore o(A∩B) = 1 and hence A∩B = {e}. By Sylow’s first theorem,
⇒ np = 1 is possible (∵ given q ≇ 1(modp)).

8. UNIT : APPLICATIONS OF SYLOW’S THEOREM 95 8. UNIT : APPLICATIONS OF SYLOW’S THEOREM 96

 
pq pq Also, np ≡ (modp), np | q and nq ≡ 1(modq), mq | p from, np ≡ (modp), np | q,
we get i.e., o(AB) = = (i.e., o(A) = p, o(B) = q) o(AB) = pq =
o(A ∩ b) 1
o(G), Similarly o(BA) = o(G). we get np ≡ 1(modp), np = 1, q.

Therefore G = AB = BA. Hence G is abelian. Similarlly, we have nq ≡ 1(modq), nq = 1, p.

Finally, to prove that G is cyclic. Since A and B are groups of prime orders Therefore nq = 1 and nq = 1, p [ ∵ p ≡ 1(modq)or q | p − 1 ⇒ p < q, which is

they must be cyclic. not possible].

Let A = ⟨a⟩, B = ⟨b⟩ so that ap = e and bq = e. Then If np = q, nq = 1. Since np = q > 1, we have np > 1 then p-sylow subgroup is
not unique and hence is not normal in G.
o(ab) = pq = o(G) −→ (1) Therefore G is non-abelian.

Note 8.2. In view of above theorem, the group of order 6 = 2.3, 10 = 2.5, 21 =
Therefore G = ⟨ab⟩ and therefore G is cyclic.
3.7, 55 = 5.7, · · · are non-abelian.
[(ab)pq = (ap )q )(bq )p ) = e.e, (ab)p q = e.
Therefore o(ab) = pqo⟨ab⟩ = o(pq) = o(G) [i.e., G = ⟨a, b⟩]. By above theorems, we arrive at

Example 8.1. Theorem 8.3. Let o(G) = p2 q, where p and q are primes with p < q and q ≇
1(modp), then G is abelian.
1. Every group of order 15 is cyclic.
Because o(G) = 15 = 3.5, where 3, 5 are primes with 3 < 5 and 5 ≇ 1(mod3). Note 8.3. In view of above theorem, the group of order 45 = 32 .5, 175 = 52 .7, .
. . are abelian.
2. (i) Every group of order 33 = 3.11 (3 < 11, 11 ≇ 1(mod3)) is cyclic and
abelian. Theorem 8.4. Let o(G) = p2 q 2 , where p and q are primes with p < q and if
p ∤ (q 2 − 1)(i.e.p ∤ q − 1 and p ∤ q + 1), then G is abelian.
(ii) Every group of order 35 = 5.7(5 < 7, 7 ≇ 1(mod5)) is cyclic and abelian
or there is no non-abelian group of order 33 or 35. Excercise 8.1. Show that every group of order 112 .132 is abelian.

Note 8.1. The result fails for group of order 21 therefore o(G) = 21 = 3.7 where Solution. o(G) = 112 .132 . Therefore G has 11 sylow subgroups of order 112
3 and 7 are primes with 3 < 7 but 7 ≡ 1(mod3). and 13 sylow subgroup of order 132 , where n11 ≡ 1(mod11), n11 ∤ 132 and n13 ≡
1(mod13), n13 | 112 .
Theorem 8.2. Let o(G) = pq, where p and q are primes with p < q and q ≡
Therefore n11 = 1 and n13 = 1.
1(modp). Then G is non-abelian.
Thus G has unique 11-sylow subgroup A and unique 13-sylow subgroup B.
Proof. Since o(G) = pq, it follows that G has p-sylow subgroup of order p and Clearly, these subgroups are normal subgroups. Then we can prove that G =
q-sylow subgroup of order q. AB = BA. Hence G becomes abelian.
8. UNIT : APPLICATIONS OF SYLOW’S THEOREM 97 8. UNIT : APPLICATIONS OF SYLOW’S THEOREM 98

Excercise 8.2. Let G be group of order 231. Show that 11-sylow subgroup of G Again, any two groups of order p have only identity in common and thus the
is contained in the centre of G. number of elements of order p in Sp is (1 + kp)(p − 1). Also any element of order
p in Sp is a cycle of length p and the number of cycles of length p in Sp is (p − 1)!
Solution. o(G) = 231 = 3 × 7 × 11
So (1 + kp)(p − 1) = (p − 1)!.
The number of sylow 11-subgroups of G is 1 + 11k and (1 + 11k)|21. Clearly
Therefore, (p − 1)! ≡ −1(modp).
these k = 0.
So, Sylow 11-subgroup H of G is normal in G. Excercise 8.4. Let p be a prime dividing o(G) and (ab)p = ap bp for all a, b ∈ G.
′ ′ ′
The number of Sylow 7-subgroups of G is 1 + 7k and (1 + 7k )|33. So, k = 0. Show that
Thus, Sylow 7-subgroups of G is normal in G. (i) Sylow p- subgroup P is normal in G.
o(H) = 11, o(K) = 7
(ii) there exist a normal subgroup N of G such that P ∩ N = {e} and G = P N .
G G G
Now o( K ) = 33 = 3 × 11 and 3 ∤ (11 − 1), thus K
is cyclic and so K
is abelian.
(iii) G has non-trivial centre.
But G′ is smallest subgroup of G such that G/G′ is abelian (G′ denotes the
commutator subgroup of G). Solution. Let pn |o(G) and pn+1 ∤ o(G). Let H = {x ∈ G \ xp = e}. Then H ̸= φ
n

∴ G′ ⊆ K ⇒ o(G′ ) = 1 or 7. as e pn
= e ⇒ e ∈ H.
If o(G′ ) = 1, then G′ = {e} ⇒ x−1 y −1 xy = e ⇒ xy = yx for all x, y ∈ G ⇒ G Let x, y ∈ H ⇒ (xy −1 )p = xp (y −1 )p = e.e = e. Therefore ⇒ xy −1 ∈ H.
n n n

is abelian ⇒ G = Z(G) ⇒ H ⊆ Z(G). Hence H ≤ G.


Let o(G′ ) = 7 ⇒ G′ = K. Clearly, H ∩K = {e} as o(H ∩K) divides o(H) = 11 Let q be a prime dividing o(H). Then x ∈ H such that o(x) = q. But
and o(K) = 7. x ∈ H ⇒ o(x) | pn ⇒ q | pn ⇒ q = p. Therefore, H is Sylow p-subgroup of G.
Let x ∈ H, y ∈ G. Then x−1 y −1 xy ∈ G′ = K. Also, x−1 y −1 xy = x−1 (y −1 xy) ∈ Then o(P ) = pn . Let x ∈ P ⇒ xp = e
n

H as H is normal in G. ⇒ x ∈ H ⇒ P ⊆ H ⇒ o(P ) | o(H) ⇒ pn | pm ⇒ n ≤ m ∴ m = n


∴ x−1 y −1 xy ∈ H ∩ K = {e} ⇒ xy = yx for all y ∈ G, x ∈ H So, o(H) = pn = o(P ) ⇒ H = P.
⇒ H ⊆ Z(G). Thus H is the only Sylow p-subgroup of G and so is normal in G.
This proves (i)
Excercise 8.3. (Wilson’s Theorem) Using Sylow’s theorems show that (p − 1)! ≡
Define θ : G → G such that θ(x) = xp . Then θ is a homomorphism.
n

−1(modp) for any prime p.


⇒ G ∼
= Im(G) is normal in G [θ(G) = Im(G)].
Kerθ
Solution. Consider Sp , then order of Sp is p(p − 1)(p − 2)...2.1. The number of Since x ∈ Kerθ ⇔ θ(x) = e. ⇔ xp = e ⇔ x ∈ H = P .
n

sylow p-subgroups of order p is Sp are of the form 1 + kp, where k is a non -ve Therefore P = Kerθ.
integer. Since each Sylow p-subgroup is of order p, we get (p − 1) elements of order Let N = Im(G) which is normal inG.
p. ( As θ(x) ∈ N, g ∈ G ⇒ g −1 θ(x)g = g −1 xp g = (g −1 xg)p = θ(g −1 xg) ∈ N ).
n n

8. UNIT : APPLICATIONS OF SYLOW’S THEOREM 99 8. UNIT : APPLICATIONS OF SYLOW’S THEOREM 100

pn pn p2n
Let x ∈ P ∩ N ⇒ x ∈ P, x ∈ N ⇒ x = e, x = θ(y) = y ⇒y =e⇒ of order 3. Thus we have more than 44 elements in G, a contradiction. So,
o(y) = pr , r ≤ n. As pn+1 ∤ o(G), we have y p = e, r ≤ n ⇒ y p = (y p )p either k = 0 or k ′ = 0.
r n r n−r
=e⇒
x = e ⇒ P ∩ N = {e}. That is either Sylow 3-subgroup or Sylow 5-subgoup is normal in G, which
Also, G ∼
=N ⇒
o(G)
= o(N ) ⇒ o(G) = o(P ).o(N ). But o(P N ) = o(P )o(N ) ⇒ proves (i).
P o(P )

o(G) = o(P N ) ⇒ P N = G. This proves (ii).


(ii) Let H be a Sylow 3-subgroup of order 3 and K be a Sylow 5-subgroup of
Let z ∈ Z(P ), z ̸= e. Let g ∈ G. Since G = P N, g = xy, x ∈ P, y ∈ N .
order 5.
Also P is normal in G, N is normal in G, P ∩ N = {e} ⇒ x′ y ′ = y ′ x′ for all
by (i), either H is normal in G or K is normal in G.
x′ ∈ P, y ′ ∈ N . Now zg = z(xy) = (zx)y = (xz)y as z ∈ Z(P ) = x(zy) = x(yz) as
In any case, HK ≤ G, o(HK) = 15 as o(H ∩ K) divides o(H) = 3 and
z ∈ P, y ∈ N = (xy)z = gz for all g ∈ G. Therefore z ∈ Z(G). Hence Z(G) ̸= {e},
o(K) = 5 =⇒ o(H ∩ K = 1). Since index of HK in G is 2, HK is normal
which proves (iii).
in G. This proves (ii).
Theorem 8.5. Let o(G) = 30. Show that
(iii) Suppose, H is normal in G, K is not normal in G. By 1 G has 6 Sylow
(i) either Sylow 3-subgroup or Sylow 5-subgroup is normal in G. 5-subgroups and so 24 elements of order 5. But o(?K)=15 =⇒ HK is
cyclic =⇒ HK has φ(15) = 8 elements of order 15. Thus G has 24+8=32
(ii) G has a normal subgroup of order 15.
elements, A contradiction.
(iii) both Sylow 3-subgroup and Sylow 5-subgroup are normal in G. ∴ K is normal in G.

Proof. (i) o(G) = 30 = 2 × 3 × 5. The number of Sylow 3-subgroup is 1 + 3k If H is not normal in G, they by 1, G has 10 Sylow 3-subgroups and so

and (1 + 3k)|10 =⇒ k = 0 or 3. 20 elements of order 3. From the above HK has 8 elements of order 15

If k = 0, then Sylow 3-subgroup is normal. and K has 4 elements of order 5. This gives 20+8+4= 32 elements in G, a

Let k ̸= 0, then k = 3. This gives 10 Sylow 3-subgroups Hi each of order 3 contradiction.

and so we have 20 elements of order 3. [Notice (f ori ̸= j) o(Hi ∩Hj )|O(Hi ) = Therefore H is normal in G. So both H and K are normal in G.

3 =⇒ o(Hi ∩ Hj ) = 1 only and so these 20 elements are different. Each This proves (iii).

Hi has one element e of order 1 and other two of order 3. a ∈ Hi =⇒


o(a)|o(Hi ) = 3 =⇒ o(a) = 1, 3 ]. Excercise 8.5. Find all the Sylow p-subgroups of S4 and show none of them is
The number of Sylow 5-subgroups is 1 + 5k ′ and (1 + 5k ′ )|6 which implies normal.
k ′ = 0 or 1. Solution: We have o(S4 ) = 24 = 23 × 3. Thus S4 has Sylow 2-subgroups and
If k ′ = 0. Then Sylow 5-subgroup is normal. Sylow 3-subgroups. Therefore the number of Sylow 2-subgroups is (1 + 2k), where
Let k ′ ̸= 0. Then k ′ = 1. This gives 6 Sylow 5 subgroups each of order 5 (1 + 2k)|3. That is, k = 0 or 1. Therefore, either there is a unique (thus normal)
and we get 24 elements of order 5. But we have already counted 20 elements Sylow 2-subgroup or 3 Sylow 2-subgroups.
8. UNIT : APPLICATIONS OF SYLOW’S THEOREM 101 8. UNIT : APPLICATIONS OF SYLOW’S THEOREM 102

Consider H = {e, a, a2 , a3 , b, ab, a2 b, a3 b|a4 = e = b2 , b−1 ab = a−1 }, where a =


(1234) and b = (13). Then H is a Sylow 2-subgroup of S4 . Again as gag −1 = KEYWORDS
/ H, where g = (132). As b, ab, a2 b, a3 b, a2 are all of
(g(1)g(2)g(3)) = (3124) ∈
p-sylow subgroups, Sylow’s first thoerem, Sylow’s second thoerem, Sylow’s third
order 2 and a3 = a−1 = (4321). Therefore H is not normal. We can write
thoerem.

H = {I, (1234), (13)(24), (1432), (14)(23), (24), (12)(34), (13)}.

The other two Sylow 2-subgroups would be (the conjugates) ASSESSMENT / TERMINAL QUESTIONS

(12)H(12)−1 = {I, (2134), (23)(14), (2431), (24)(13), (14), (12)(34), (23)} 1. Discuss the number and nature of the 3-Sylow subgroups and 5-Sylow sub-
(23)H(23)−1 = {I, (1324), (12)(34), (1423), (14)(23), (34), (13)(24), (12)} groups of a group of order 32 · 52 .
So S4 has 3 Sylow 2-subgroups which are not normal.
2. If G is a group of order 385, show that 11-Sylow subgroup is normal and its
Again number of Sylow 3-subgroups is (1 + 3k)|8. That is,k = 0 or 1. There exist
7-Sylow subgroups is in the center of G.
either a unique (thus normal) Sylow 3-subgroup or 4 Sylow 3-subgroups. It is easily
seen that {I, (123), (132)}, {I, (124), (142)}, {I, (134), (143)}and{I, (234), (243)} 3. Find the possible number of 11-Sylow subgroups, 7-Sylow subgroups and
are the four Sylow 3-subgroups (and so they are not normal). 5-Sylow subgroups in a group of order 52 · 7 · 11.

4. If p, q are primes, p < q and p ∤ (q − 1), show that every group of order pq
SUMMARY is cyclic and abelian.

The Sylow theorems are important tools for analysis of special subgroups of a 5. Find all 3-Sylow subgroups of the symmetric group S4 and verify that they
finite group G, known as Sylow subgroups. They are especially useful in the are all conjugate.
classification of finite simple groups. Also, provide us with a sort of partial converse
6. Show that group of order 52 · 72 is abelian.
to Lagrange’s theorem, by asserting the existence of certain subgroups (called p-
Sylow subgroups) of any group with a given order, and gives some information 7. Show that group of order 132 · 172 is abelian

about their properties. There is a complete classification of groups with all Sylow- 8. Show that the group of order 36 is not simple.
subgroups being cyclic. Further, a subgroup of a finite group is termed a normal
REFERENCES
Sylow subgroup if it satisfies the following equivalent conditions: It is a Sylow
subgroup, and is normal in the whole group. It is a Sylow subgroup, and is 1. I. N. Herstein: Topics in Algebra, 2nd Edition, John Wiley and Sons, 2008.
subnormal in the whole group.
2. Surjeet Singh and Qazi Zameeruddin: Modern Algebra, 8th Edition, Vikas
Publishing House, 2021.

8. UNIT : APPLICATIONS OF SYLOW’S THEOREM 103 8. UNIT : APPLICATIONS OF SYLOW’S THEOREM 104

3. N. Jacobson: Basic Algebra-I, 2nd Edition, Dover Publications, 2009. BLOCK 3: ELEMENTARY RING THEORY
4. M. Artin : Algebra, 2nd Edition, Prentice Hall of India, 2015.

5. Darek F. Holt, Bettina Eick and Eamonaa A. O’brien: Handbook of com-


UNIT-9: Fundamentals of Rings
putational group theory, 1st Edition, Chapman & Hall/CRC Press, 2005.
UNIT-10: Ring Homomorphism and Isomorphisms
6. J. B. Fraleigh : A first course in abstract algebra, 7th Edition, Addison-
Wesley Longman, 2002. UNIT-11: Special Types of Ideals

7. V. K. Khanna and S. K. Bhambri: A course in Abstract algebra, 4th Edition, UNIT-12: The field of Quotients

Vikas Publishing House Pvt Ltd., 2013.

8. J. A. Gallian: Contemporary Abstract algebra, 9th Edition, Narosa Pub-


lishing House. 2008.
105 9. UNIT : FUNDAMENTALS OF RINGS 106

9 UNIT : Fundamentals of Rings (i) (R, +) is an abelian group


⇒ For any a, b, c ∈ R, whenever a + b ∈ R (Closure law)
⇒ For any a, b, c ∈ R, whenever a + (b + c) = (a + b) + c (Associative law)
OBJECTIVES ⇒ For a ∈ R, there exist an element 0 in R such that a = a + 0 = 0 + a,
where ‘0’ is called the zero element of of R (Identity law)
After studying this unit you will be able to:
⇒ Q ∈ R, there exist−a ∈ R such that a + (−a) = 0 = (−a) + a, where
• define rings and known its elementary properties; ‘ − a′ is called the negative element of ‘a′ (Inverse law)

• explain the some special classes of rings along with examples; ⇒ For any a, b ∈ R, we have a + b = b + a (Commutative law)

• define the Integral domain, division ring and skew division rings(fields); (ii) (R, ·) is a semi-group
⇒ For any a, b, c ∈ R we have a · (b · c) = (a · b) · c (Associative)
• describe the importance of subring and ideals;
(iii) Distributive property in R
• explain some fundamental lemma’s of ideals;
⇒ For any a, b, c ∈ R we have
• solving some problems of ideals; a · (b + c) = a · b + a · c (Left)
• define quotient rings along with its elementary properties. (a + b) · c = a · c + b · c (Right)

INTRODUCTION Example 9.1. The number systems such as an integer Z, rationals Q, reals R
and complex C are rings under the usual addition and multiplication.
The theory of rings grew out of study of two particular classes of rings, polynomial
rings in n variables over the real or complex number and in integers of an algebraic
number field. It was David Hilbert (1862-1943) who first introduce the term Ring. Example 9.2. The set 2Z of all even number under addition and multiplication.
The theory of commutative ring was given by Emmy Noether (1882-1935) in her
monomental paper “Ideal theory in rings” in 1921. Example 9.3. The set kZ of multiples of integer k with respect to addition and
multiplication.
Note 9.1. The set Z of all integers under the operation of addition (+) is a
commutative group and under multiplication (×) is a semi group, all though it Example 9.4. Z = {0, 1, 2, 3, 4, 5} under addition and multiplication.
is not a group. Further, the two operations are connected by distributive law,
Example 9.5. R(n) of all (n × n) matrices with reals under matrix addition and
namely for all a, b, c ∈ Z, we have a.b + a.c = a.(b + c) and (b + c).a = b.a + c.a.
matrix multiplication.
System such as Z, +, · are of frequent occurence and are of general importance. In
Commutative Ring (Ring with an identity) :
generalisation of this basic notion leads to the concepts of system called a rings.
Definition 9.2. The commutative ring is a ring in which the multiplication is
Definition 9.1. Let R be a non-empty set with two binary operation + and ·.
commutative. The study of commutative rings is called commutative algebra.
Then the triple R, +, ·) is said to be a ring if it satisfies the following axioms

9. UNIT : FUNDAMENTALS OF RINGS 107 9. UNIT : FUNDAMENTALS OF RINGS 108

Note 9.2. By examples (1) to (3), we have multiplication is commutative. How- Moreover in these cases multiplication follows commutative law. Therefore,
these are called a commutative division ring.
ever, the rings of matrix is not a commutative ring. Since every ring is an abelian
group under +, it will have an identity for +. This we call zero. A ring need not Definition 9.5. A field is a commutative ring with identity (1 ̸= 0) in which
have an identity for ×. The (2Z, +, ×) is one such example. When a ring has a every non-zero element has a multiplicative inverse.
identity for multiplication, we call it as a ring with identity or a ring with unit
Example 9.12. The rings Q, R, C are fields.
element. For each of the ring in examples (1) and (3), integer 1 is an identity.
For the ring of matrices, the unit matrix of n rows happens to be the identity. Definition 9.6. A skew field is a division ring whose ring product is specifically
Some special classes of Rings: not commutative

Definition 9.3. If R is a commutative ring then a ̸= 0 ∈ R is said to be a zero Example 9.13. If a, b are elements of a field with ab = 0 then if a ̸= 0 it has
divisor if there exists a, b ∈ R and b ̸= 0 and ab = 0. A commutative ring is an an inverse a−1 and so multiplying both sides by this gives b = 0. For this reason,
integral domain if it has no zero divisors. division rings are called a skew fields.

Example 9.6. The strucute (J, +, ×) of all integers is an integral domain. For Note 9.3.
nm = 0 where m, n ∈ Z holds if and only if atleast one of m or n is zero. (i) The set of all complex numbers under + and × form a field.

Example 9.7. The strucute (2Z, +, ×) of all even integers is an integral domain (ii) The ring of quaternions is a division ring not a field.

under the operation + and ×. (iii) Division ring has no zero divisors.
√ (iv) A field is an integral domain for
Example 9.8. The real number of the form m + n 3, where m, n are integers
form an Integral domain. 1. It is a commutative ring;
2. Being a division ring, it has no zero divisors.
Definition 9.4. A ring is said to be a division ring if its non zero elements
(v) A finite integral domain is a field.
form a group under multiplication.
Definition 9.7. A subring S of a ring R is a subset of R which is a ring under
Example 9.9. The ring H of all real quaternions is a division ring. Where the
the same operations as R. A non-empty subset S of R is a subring if a, b ∈ S =⇒
set of real quaternions H = {a0 + ia1 + ja2 + ka3 : ai ∈ R}; i2 = j 2 = k 2 =
a − b, ab ∈ S. So S is closed under subtraction and multiplication.
ijk = −1 (Note that ij = −ji, ik = −ki, jk = −kj, so multiplication in H is not
commutative). Example 9.14.

Example 9.10. The ring ⟨Q, +, ·⟩ of rationals is a division ring. 1. The structure (2Z, +, ×) is a subring of (Z, +, ×).

2. The structure (Z, +, ×) is a subring of (Q, +, ×).


Example 9.11. The ring ⟨R, +, ·⟩ of real are division rings.
9. UNIT : FUNDAMENTALS OF RINGS 109 9. UNIT : FUNDAMENTALS OF RINGS 110

3. The Gaussian integers Z[i] = {a + ib : a, b ∈ Z} is a subring of (C, +, ×), (v) Fields do not have ideals other than trivial ideals, that is if F is a field, then
where the gaussian integer is a complex number whose real and imaginary {0} and F are the only ideals of F .
parts are both integers. [∵ let S be an ideal of F and S be a zero ideal (S ̸= {0}), then there exist
a ∈ S, where a ̸= 0, a ∈ F . Since F is a field, a−1 ∈ F ⇒ a−1 a ∈ S, 1 ∈ S.
Note 9.4. Hence S = F .]

(i) The Gaussian integers Z[i] is an integral domain but not a field. (vi) Multiplication of ideals of a ring are ideals but union of ideals need not be
an ideal.
(ii) Intersection of two subring is a subring but union need not be a subring.
(vii) If U is an ideal of ring R and S is a subring of R containing U , then U is an
S = 2Z, T = 3Z be a subring of Z.
ideal of S also.
But S ∪ T = {0, ±2, ±3, ±4, ±6 · · · } is not a subring of Z.
[∵ 2, −3 ∈ S ∪ T, 2 − 3 = −1 ∈
/ S ∪ T ]. Lemma 9.1. Let R be a commutative ring with unity whose ideals are {0} and R
(iii) The Subring of ring with unity need not be a ring with unity. For instance itself. Then R is a field.
(2Z, +, ×).
Proof. Let a ̸= 0, be an element of R. We verify that a−1 ∈ R. Consider Ra =
Definition 9.8. A non-empty subset U of a ring R is said to be an ideal of R if {ra : r ∈ R}. Also, we verify that Ra is an ideal of R. For x = r1 a and y = r2 a

(i) For any a, b ∈ U, a − b ∈ U in Ra, we have x − y = (r1 − r2 )a where r1 − r2 ∈ R. Therefore x − y ∈ Ra.


Simillarly, for any r ∈ R and x = r1 a ∈ Ra, we have rx = r(r1 a) = (rr1 )a
(ii) For any a ∈ U =⇒ ar ∈ U and ra ∈ U
where rr1 ∈ R. Therefore rx ∈ Ra.

If only ar ∈ U , then U is a right ideal and if only ra ∈ U then U is a left ideal. If Since R is a commutative ring, we have xr = rx ∈ Ra. Therefore Ra is an

U is both left and right ideal of a ring R, then U is called two sided ideal of R or ideal of R.

simply an ideal of R. Since {0} and R are the only ideals of Ra, it follows that Ra = {0}, Ra = R.
Now 1 ∈ R ⇒ 1.a ∈ Ra. Therefore a ∈ Ra, where a = 0. We have, Ra ̸= 0.
Note 9.5.
Hence Ra = R. Again 1 ∈ R ⇒ 1 ∈ Ra and 1 = ba for some b ∈ R. Therefore
(i) In any ring R, {0} and R are ideals of a ring R. These are called improper ab = ba = 1 ⇒ b = a−1 ∈ R.
ideals of R. [Here {0} → null ideal or zero ideal,R → unit ideal]. These
ideals are also known as trivial ideals. Thus R is a commutative ring with unity in which all non-zero elements are
units in R. Hence R is a field.
(ii) A ring R is simple if it has no proper ideals.

(iii) If U is an ideal of a ring R and 1 ∈ U then U = R. Lemma 9.2. Let R be a ring with unity (real not be commutative) in which {0}
[∵ U ⊂ R is obvious. Further, if r ∈ R, 1 ∈ U and r.1 ∈ U then r ∈ U ∀r ∈ R. and R are the only two left ideals. Then R is a division ring (Skew field).
Therefore R ⊂ U =⇒ U = R].

(iv) For a ∈ R, let Ra = {xa : x ∈ R} then Ra is a left ideal of R. In otherwords, Proof. Let a ̸= 0 be any element of R. We verify that a−1 ∈ R. Consider
if R is a ring and a ∈ R, let r(a) = {x ∈ R : ax = 0}. Then r(a) is a right Ra = {ra : r ∈ R}.
ideal of R.

9. UNIT : FUNDAMENTALS OF RINGS 111 9. UNIT : FUNDAMENTALS OF RINGS 112

Now we verify that Ra is an ideal of R. Let x = r1 a and y = r2 a ∈ Ra. Then Sx1 ∈ U ∀ S ∈ R implies r(Sx1 ) ∈ U ∀ r ∈ R ⇒ Sx1 ∈ [R : U ]. Also
r(x1 S) = (rx1 )S ∈ U ⇒ r(x1 S) ∈ U ∀ r ∈ R ⇒ x1 S ∈ [R : U ]. Therefore [R : U ]
x − y = (r1 − r2 )a ∈ Ra. Therefore x − y ∈ Ra.
is an ideal of R.
Consider r ∈ R and x ∈ Ra such that x = r1 a. Then rx = r(r1 a) = (rr1 )a ∈ Let u ∈ U , Clearly ru ∈ U ∀ r ∈ R such that U be an ideal. Therefore
u ∈ [R : U ] ∀ u ∈ U ⇒ U ⊂ [R : U ].
Ra as (r1 r2 ∈ R).
Since {0} and R are the only left ideals, we have Ra = {0} or Ra = R. Now Definition 9.9. Let R be a ring and U be an ideal of R. Then the set R
U
= {U+r :
1 ∈ R ⇒ 1 · a = a ∈ R and Ra ̸= {0}. Hence Ra = R. Again 1 ∈ R ⇒ 1 ∈ Ra r ∈ R} becomes a ring under the operation + and × given by U+r + U+s = U+r+s
R
1 = ba for some b ∈ R. We see that b ̸= 0. Hence b has left inverse b−1 = a. and U+r × U+s = U+rs for rs ∈ R. The zero of is U+0 U and the nefative U+r
U
Consider b(ab) = (ba)b = 1(b) = b. Therefore b−1 [b(ab)] = b−1 [b] ⇒ ab = 1. R
is U−1 . Therefore the ring is a quotient ring.
U
We have ab = ba = 1 [∵ b−1 b = bb−1 = 1 and a = b−1 ]. In otherwords, a quotient ring (also known as factor ring, difference ring or
Thus R is a non commutative ring with unity in which all non-zero elements residue class ring) is a ring that is the quotient of a ring and one of its ideals.
are units in R. Hence R is a division ring.
Note 9.7.
Note 9.6. For a ∈ R, let aR = {ax : x ∈ R}. Then aR is a right ideal of R. R
(i) is commutative if R is commutative [∵ U+r U+s = U+rs = U+sr = U+s U+r ].
U
Excercise 9.1. Let U be a left ideal of a ring R, and λ(U ) = {x ∈ R : xu = 0},
R R
(ii) is a ring with unity U+1 if is a ring with 1 as unity [∵ U+r U+1 =
then prove that λ(u) is an ideal of R. U U
U+r·1 = U+r ].
Solution : Let λ(U ) = {x ∈ R : xu = 0 ∀ u ∈ U } ⊂ R. Since 0µ = 0∀ µ ∈ U
we see that 0 ∈ λ(U ). Therefore λ(U ) ̸= ϕ. For x, y ∈ λ(U ), we have xu = 0 and
yu = 0 ∀ x, y ∈ R, µ ∈ U . Therefore xu − yu ∈ λ(U ) = 0 ⇒ (x − y)u = 0 ∀ u ∈ U . SUMMARY
Hence (x − y) ∈ λ(U ).
Ring theory studies the structure of rings, their representations, or, in different
Let x ∈ λ(U ) so that xu = 0 ∀ u ∈ U and r ∈ R be any element. Since
language, modules, special classes of rings as well as an array of properties that
r(xu) = 0. Hence r(xu) = r · 0 = 0 ∀ u ∈ U ⇒ rx ∈ λ(U ). Further, by virtue of
proved to be of interest both within the theory itself and for its applications such as
the fact that U is a left ideal of R, we have ru ∈ U for all u ∈ U, r ∈ R. Now for
extension of Group Theory, the study of geometric objects, topology and wide ar-
any x ∈ λ(U ), we have x(ru) = 0 ∀ u ∈ U . Therefore (xr)u = 0 ∀ u ∈ U . Hence
eas with in the mathematics, computer science, mathematical/theoretical physics
xr ∈ λ(U ). Therefore λ(U ) is an ideal of R.
and in many area of allied sciences.

Excercise 9.2. Let U is an ideal of R and a set [R : U ] = {x ∈ R : rx ∈ U ∀ r ∈ KEYWORDS


R}. Then prove that [R : U ] is an ideal of R containing U .
Ring, commutative ring, integral domain, division ring, field, subring, ideals, quo-
Solution. Clearly R = 0 ∈ U ∀ r ∈ R where r ∈ [R : U ]. It is the zero element tient ring.
of R. Therefore 0 ∈ [R : U ] and [R : U ] be non-empty [R : U ] + ϕ.
Let x1 , x2 ∈ [R : U ] such that rx1 ∈ U and rx2 ∈ U ∀ r ∈ R. Then rx1 − rx2 = ASSESSMENT / TERMINAL QUESTIONS
r(x1 − x2 ) ∈ U ∀ r ∈ R. Therefore x1 − x2 ∈ [R : U ]. Further, x1 [R : U ] and 1. Show that intersection of two sub rings is asubring. But union of two sub
S ∈ R. rings need not be a sub ring.
We need only prove that Sx1 and x1 S ∈ [R : U ].
2. Let R be a finite integral domian. Then show that R is a field.
9. UNIT : FUNDAMENTALS OF RINGS 113 114

3. If every x ∈ R satisfies x2 = x, prove that R must be commutative. 10 UNIT: Ring Homomorphism and Isomorphisms
4. If a, b are any elements of a ring R, prove that

(i) −(−a) = a,
OBJECTIVES
(ii) −(a + b) = −a − b,
(iii) −(a − b) = −a + b. After studying this unit, you should be able to:
5. Show that S is an ideal of S + T , where S is any ideal of R and T is any
• define, and give examples of, different kinds of ring homomorphisms;
subring of R.

6. Let U and V be ideals of ring R. Show that U + V is an ideal of R. • obtain the kernel and image of any ring homomorphism;

7. Show that the ring of real quaternions is a division ring. • prove, and apply, some basic properties of a ring homomorphism;
8. Give an example to show that the left ideal need not be right ideal and right
ideal need not be left ideal. • state, prove and apply the Fundamental Theorem of Homomorphism for

REFERENCES rings;

1. I. N. Herstein: Topics in Algebra, 2nd Edition, John Wiley and Sons, 2008. • define, and give examples of, ring isomorphisms.
2. Surjeet Singh and Qazi Zameeruddin: Modern Algebra, 8th Edition, Vikas
INTRODUCTION
Publishing House, 2021.

3. N. Jacobson: Basic Algebra-I, 2nd Edition, Dover Publications, 2009. Analogous to the notion of a group homomorphism and isomorphism, we have

4. M. Artin : Algebra, 2nd Edition, Prentice Hall of India, 2015. studied the ring homomorphism and isomorphism. Therefore, it is essential to
foresee the ring homomorphism and isomorphism to be a map between rings that
5. Darek F. Holt, Bettina Eick and Eamonaa A. O’brien: Handbook of com-
putational group theory, 1st Edition, Chapman & Hall/CRC Press, 2005. conserve the ring structure of its domain. In this case there are two binary oper-
6. J. B. Fraleigh : A first course in abstract algebra, 7th Edition, Addison- ations involved.
Wesley Longman, 2002. Ring Hommorphism:
7. D. S. Dummitv and R.M. Foote: Abstract Algebra, 2nd Edition, John Wiley
& Sons, 2003. Definition 10.1. Let (R, +, ×) and (R1 , +, ×) be two rings. Then ϕ : R → R1 is
said to be a homomorphism if
8. V. K. Khanna and S. K. Bhambri: A course in Abstract algebra, 4th Edition,
Vikas Publishing House Pvt Ltd., 2013.
(i) ϕ(x + y) = ϕ(x) + ϕ(y) ∀ x, y ∈ R

(ii) ϕ(xy) = ϕ(x) · ϕ(y) ∀ x, y ∈ R

Note 10.1.

(i) A homomorphism ϕ is called a/an

10. UNIT: RING HOMOMORPHISM AND ISOMORPHISMS 115 10. UNIT: RING HOMOMORPHISM AND ISOMORPHISMS 116

√ √ √ √
1. monomorphism, if ϕ is 1-1; 3. Let ϕ : Z 2 → Z 2 by ϕ(a + b 2)a − b 2. Then ϕ is a homomorphism.
√ √
2. epimorphism, if ϕ is onto; [Consider x = a + b 2, y = c + d 2.]

3. isomorphism, if ϕ is 1-1 and onto (i.e., R ≈ R1 ). √ √ √


ϕ(x + y) = ϕ(a + b 2 + c + d 2) =ϕ(a + c + 2(b + d))
1
(ii) If ϕ is homomorphism of R onto R , then √
=(a + c) − (b + d) 2

1. ϕ(x) = 0; =ϕ(x) + ϕ(y)


√ √
2. ϕ(−x) = −ϕ(x) for every x ∈ R. ϕ(xy) =ϕ((a + b 2)(c + d 2))
√ √
=ϕ(ac + ad 2 + bc 2 + bd2)
Kernel and Image of Homomorphism √
=ϕ((ac + 2db) + 2(ad + bc))
Definition 10.2. If ϕ : R → R1 is a homomorphism then the kernel of ϕ is √
=(ac + 2db) − 2(ad + bc)
K{x ∈ R : ϕ(x) = 0} or kernel of ϕ is the set of all elements a ∈ R such that √ √
=(ab 2)(c − d 2)
ϕ(a) = 0, the zero element of R1 .
=ϕ(x) · ϕ(y)].
Definition 10.3. Let ϕ : R → R1 be a ring homomorphism. If a ∈ R, then
√ √
ϕ(a) ∈ R1 , then ϕ(a) is the image of under ϕ. Therefore, ker(ϕ) = {a + b 2; ϕ(a + b 2) = 0} = {0}.

Note 10.2. 4. Let ϕ : Z → Zn , where Zn = {0, 1, 2, 3, · · · (n − 1)} be defined by ϕ(a) = r


where a ≡ r(mod n). Then ϕ is a homomorphism.
(i) If ϕ : R → R1 is a homomorphism with ker(ϕ), then ker(ϕ) is a subgroup of
R under addition. Consider, a ≡ r1 (mod n), b ≡ r2 (mod n).

Therefore a + b ≡ (r1 + r2 )(mod n).


(ii) If a ∈ kerϕ and r ∈ R, then both ar and ra are in ker(ϕ).
ϕ(a + b) = (r1 + r2 ) = ϕ(a) + ϕ(b).
(iii) kerϕ is an ideal of R, Image ϕ is a subring of R1
ker(ϕ) ={a ∈ Z ; ϕ(a) = 0}
Example 10.1.
={a ∈ Z ; no where a ≡ r(mod n)}
1. Let ϕ : R → R1 defined by ϕ(a) = 0 for all a ∈ R. Then ϕ is homomorphism. ={a ∈ Z ; a ≡ o(mod n)}
Therefore ker(ϕ) = {a ∈ R : ϕ(a) = 0}. Thus ker(ϕ) = R.
={a ∈ Z ; a = kn : k ∈ Z}

2. Let ϕ : R → R1 by ϕ(x) = x ∀ x ∈ R is a homomorphism. ={kn : k ∈ Z}

Therefore ker(ϕ) = {x ∈ R : ϕ(x) = 0} = {0}. ∴ ker(ϕ) ={nz}.]


10. UNIT: RING HOMOMORPHISM AND ISOMORPHISMS 117 10. UNIT: RING HOMOMORPHISM AND ISOMORPHISMS 118

Lemma 10.1. The homomorphism ϕ : R → R1 is an isomorphism if and only if R ∼ R1


3. = 1
W W
ker(ϕ) = {0}.
Proof. 1. Let ϕ : R → R1 be an epimorphism with U as kernel. Clearly U is
Proof. Let ϕ : R → R1 be an onto homomorphism. R
an ideal of R so that the quotient ring
is an ideal.
U
Suppose ϕ is an isomorphism, so that it is one-one also we now verify that
R
Define η : → R1 by η(U1 ) = ϕ(a)
ker(ϕ) = {0}. U
Let a ∈ ker(ϕ) be any element, then ϕ(a) = 01 . We verify that η is an isomorphism

Put 0 ∈ ker(ϕ) and ϕ(0) = 01 ⇒ ϕ(a) = ϕ(0)


(i) η is one-one
ϕ is one-one, a = 0. R
For U+a · U+b ∈
Therefore ker(ϕ) = {0}. U
Suppose η(U+a ) = η(U+b )
Conversely, Suppose ker(ϕ) = {0}.
ϕ(a) = ϕ(b)
We verify that ϕ is an isomorphism.
For this we have to verify that ϕ is one-one ϕ(a) = ϕ(b) for some a, b ∈ R. ϕ(a) − ϕ(b) = 01

Therefore ϕ(a) − ϕ(b) = 01 ⇒ ϕ(a − b) = 01 ϕ(a − b) = 01


a − b ∈ ker(ϕ) where ker(ϕ) = {0} Therefore a − b ∈ kerϕ = U
a−b=0⇒a=b a−b∈U
Therefore ϕ is one-one and hence isomorphism. U+a = U+b
R
Note 10.3. Suppose R is a ring and U is an ideal of R. Let γ : R → defined Therefore η is one-one
U
R
by γ(a) = U+a . Then γ is a homomorphism of R onto . (ii) Onto
U
Let b1 = R1 . As ϕ is onto, ∃ an element b ∈ R such that ϕ(b) = b1 .
[This is an epimorphism (γ), natural homomorphism].
Therefore b1 = ηU+
R
Fundamental theorems of Homomorphism of rings: Thus for any b1 ∈ R1 , there exists an element U+b ∈ satisfying
U
1
ηU+b = b
Theorem 10.1. Let R and R1 be rings and ϕ is a homomorphism of R onto R1
Therefore η is onto.
with kernel U . Then
R
(iii) Homomorphism For U+a · U+b ∈
R U
1. R1 ≈
U
2. Moreover there is one to one correspondence between the set of ideals W 1 of
R1 and the set of ideals W of R containing U .

10. UNIT: RING HOMOMORPHISM AND ISOMORPHISMS 119 10. UNIT: RING HOMOMORPHISM AND ISOMORPHISMS 120

Consider We have to verify this correspondence is one to one.

Let L be an ideal of R contained in (U (L ⊂ U )) and L1 = {ϕ(x) : x ∈ L} ⊂


η(U+a + U+b ) =η(U+a+b )
R1
=ϕ(a + b)
We verify that L1 is an ideal of R1 where ϕ(x) · ϕ(y) ∈ L1
=ϕ(a) + ϕ(b)
x, y ∈ L1 ⇒ x − y ∈ L1 ⇒ ϕ(x) − ϕ(y) ∈ L1
=η(U+a ) + η(U+b )
Similarlly, ϕ(r) ∈ R1 and ϕ(x) ∈ L1 , then r ∈ R and x ∈ L
η(U+a · U+b ) =η(U+a )η(U+b )
rx ∈ L ⇒ ϕ(rx) ∈ L1 ⇒ ϕ(r) · ϕ(x) ∈ L1
=ϕ(ab)

=ϕ(a)ϕ(b) Thus L1 is an ideal of R1 .

If we define T = {x ∈ R ; ϕ(x) ∈ L1 }
Therefore η is homomorphism, one-one and onto.
T becomes an ideal of a ring R contained in U as proved in case of ideals of
Therefore η is an isomorphism, W and W 1 .
R∼ 1
∴ =R
U We can prove that L = T .

2. Let W 1 ⊂ R1 and W 1 be an ideal R1 . Define W = {x ∈ R : ϕ(x) ∈ W 1 } ⊂ l ∈ L, ϕ(l) ∈ L1 ⇒ L ∈ T ∀ l ∈ L


R. We verify that w is an ideal of R. Let x, y ∈ W , so that ϕ(x)ϕ(y) ∈ W 1 .
Therefore L ⊂ T
Therefore ϕ(x) − ϕ(y) ∈ W 1 , ϕ(x − y) ∈ W 1 ⇒ x − y ∈ W
Similarly, l ∈ L, ϕ(t) ∈ L1 ⇒ t ∈ L f or any t.
Further or any r ∈ R and x ∈ W we have, ϕ(r) ∈ R1 and ϕ(x) ∈ W 1
Therefore T ⊂ L ⇒ T = L
Therefore ϕ(r) · ϕ(x) ∈ W 1
Thus there is one-one correspondence between the set of ideals W 1 of R1
ϕ(rx) ∈ W 1 ⇒ rx ∈ W and the set of all ideals W of R containing U .
Also xr ∈ W and hence W is an ideal of R. To claim that U ⊂ W
3. Suppose W is the ideal of R containing U corresponding to ideal W 1 of R1
a ∈ U (U is a kernel) ϕ(a) = 01 ∈ W 1 R R1
so that the quotient ring ≊ 1 are defined.
W W
ϕ(a) ∈ W 1 R1
Define ψ : R → 1 by ψ(a) = W 1 + ϕ(a) and verify that it is epimorphism.
W
Therefore a ∈ U ⇒ a ∈ W for any a. Therefore U ⊂ W (U contained in W )

Thus there is a correspondence between the set of ideals W 1 of R1 and set


of all ideals W of R containing U . (R1 ⊂ R)
10. UNIT: RING HOMOMORPHISM AND ISOMORPHISMS 121 10. UNIT: RING HOMOMORPHISM AND ISOMORPHISMS 122

Consider Finding ker(ψ),

R1
ψ(a + b) =W 1 + ϕ(a + b) ker(ψ) ={x ∈ R : ψ(x) ∈ W 1 , the zer of }
W1
=W 1 + [ϕ(a) + ϕ(b)] ={x ∈ R : W 1 + ϕ(x) ∈ W 1 }
=W 1 + ϕ(a) + W 1 + ϕ(b) ={x ∈ R : ϕ(x) ∈ W 1 }
ψ(a + b) =ψ(a) + ψ(b) ={x ∈ R : x ∈ W }

ker(ψ) =W.
Consider

ker(ψ) = W −→ (2).
ψ(ab) =W 1 + ϕ(ab)
R R1
From (1) and (2), we have ≈ 1.
1
=W + [ϕ(a)ϕ(b)] W W
This is called an first isomorphism theorem for rings.
=(W 1 + ϕ(a)) + (W 1 + ϕ(b))

ψ(ab) =ψ(a)ψ(b).

Therfore ψ is a homomorphism.
R1
Further, for any element W 1 + U (ϕ(b) = b1 ) of , we have b1 ∈ R1 . Since
W1
ϕ is onto for every b ∈ R , there exist an element b ∈ R such that ϕ(b) = b1 .
1 1

Then ψ(b) = W 1 + ϕ(b) = W 1 + b1


R1
Thus for any W 1 + b1 ∈ ∃ an element b ∈ R such that b1 = ϕ(b) ∈ R1
W1
1 1
and ψ(b) = W + b .

Therefore ψ is onto, which is also epimorphism.

Hence by fundamental theorem of homomorphism for rings, we have

R R1
≈ 1 −→ (1)
ker(ψ) W

10. UNIT: RING HOMOMORPHISM AND ISOMORPHISMS 123 10. UNIT: RING HOMOMORPHISM AND ISOMORPHISMS 124

7. Let R = R or Q. Show that the only isomorphism of R onto R is the identity


SUMMARY map.
A+B ∼ B
Ring theory studies the structure of rings, their representations, or, in different 8. Let A, B be two ideals of a ring R. Show that = .
A A∩B
language, modules, special classes of rings as well as an array of properties that
REFERENCES
proved to be of interest both within the theory itself and for its applications
such as extension of Group Theory, the study of geometric objects, topology and 1. I. N. Herstein: Topics in Algebra, 2nd Edition, John Wiley and Sons, 2008.
wide areas with in the mathematics, computer science, mathematical/theoretical
2. Surjeet Singh and Qazi Zameeruddin: Modern Algebra, 8th Edition, Vikas
physics and in many area of allied sciences.
Publishing House, 2021.

KEYWORDS 3. N. Jacobson: Basic Algebra-I, 2nd Edition, Dover Publications, 2009.

4. M. Artin : Algebra, 2nd Edition, Prentice Hall of India, 2015.


Ring homomorphism, ring isomorphism, kernel, image, fundamental theorem of
homomorphism for rings. 5. Darek F. Holt, Bettina Eick and Eamonaa A. O’brien: Handbook of com-
putational group theory, 1st Edition, Chapman & Hall/CRC Press, 2005.
ASSESSMENT / TERMINAL QUESTIONS
6. J. B. Fraleigh : A first course in abstract algebra, 7th Edition, Addison-
1. If R is a ring with unity and f : R → R′ is a homomorphism , where R′ is Wesley Longman, 2002.
an integral domain such that ker(f ) ̸= R then show that f (1) is unity og
7. D. S. Dummitv and R. M. Foote: Abstract Algebra, 2nd Edition, John Wiley
R′ .
& Sons, 2003.
2. Let f : R → R′ be an onto homomorphism, where R is a ring with unity.
8. V. K. Khanna and S. K. Bhambri: A course in Abstract algebra, 4th Edition,
Show that f (1) is unity of R′ .
Vikas Publishing House Pvt Ltd., 2013.
3. Show by an example that we can have a homomorphism f : R → R′ such
that f (1) is not unity of R′ , where 1 is unity of R.

4. State and prove the fundamental theorem of ring homomorphism.

5. Let Z be the ring of integers. Show that the only homomorphism from
Z → Z are the identity and zero mapping.

6. Show that the relation of isomorphism in rings is an equivalence relation.


125 11. UNIT: SPECIAL TYPES OF IDEALS 126

11 UNIT: Special Types of Ideals Proof. Clearly, S = {ra : r ∈ R} ⊂ R. Let x = r1 a and y = r2 a, where r1 , r2 ∈ R.
Then x − y = (r1 − r2 )a ∈ S as r1 , r2 ∈ S.
Therefore x − y ∈ S.
OBJECTIVES
For r ∈ R and x = r1 a forall r1 ∈ R, we have

After studying this unit, you should be able to: rx = rr1 a ⇒ rx ∈ S as rr1 ∈ R. Also xr ∈ S.
Therefore S is an ideal of R.
• define, and give examples of, different types of ideals;
Given 1 ∈ R ⇒ 1a ∈ S ⇒ a ∈ S.
• uderstanding the concepts of principle ideal and principle ideal ring; To verify that S is a principle ideal generated by a.
Choose an idel T of R containing a and show that S ⊂ T . So, ra ∈ S ⇒ r ∈ R
• uderstanding the concepts of prime ideal;
and a ∈ S ⇒ r ∈ R and a ∈ T [ ∵ T is an ideal of R].
• uderstanding the concepts of maximal ideal; Therefore ra ∈ T .
S ⊂ T for any ideal T of R containing a. Therefore S is a principle ideal of R.
INTRODUCTION
Thus S = ⟨a⟩ or S = (a).
Ideal is a subring of a mathematical ring with certain absorption properties. The
Note 11.1. In any commutative ring R with unity the trivials ideals are {0} and
concept of an ideal was first defined and developed by German mathematician
R are principle ideals.
Richard Dedekind in 1871. In particular, he used ideals to translate ordinary
i.e. {0} = (0) ⇒ R = (1)
properties of arithmetic into properties of sets. In this unit, we study the special
types of ideals such as principle ideal, prime ideal and maximal ideals of a ring. Principle Ideal Ring:

Definition 11.2. An integral domain R is said to be a principle ideal ring


Principle Ideal:
(PID) if every ideal in R is a principle ideal.
Definition 11.1. An ideal S of a ring R is said to be a principle ideal of a ring
Theorem 11.2. A ring Z of integers is a Principle ideal ring (PIR).
R if there exists an element a ∈ S such that any ideal T of R containing a ∈ S
also the element a is called a general of a and S is said to be generated by a. This Proof. Interger Z is an integral domain. Let S be any ideal of Z. Then S = {0}

concept is denoted by S = (a) or S = ⟨a⟩ or S ̸= {0}.


If S = {0}, then S is a principle ideal generated by {0}. i. e. S = (0).
Example 11.1. Every ideal of (Z, +, ·) is of the form nZ, which is generated by
Now, suppose S ̸= {0}. Then there are some nonzero elements in S
n so every ideal of Z is a principle ideal.
i. e. a ∈ S and a ̸= 0.
Theorem 11.1. If a is an element of a commutative ring with unity, then S = Since S is a subgroup of Z under addition S being an ideal.
{ra : r ∈ R} is the principle ideal of R. We see that −a ∈ S.

11. UNIT: SPECIAL TYPES OF IDEALS 127 11. UNIT: SPECIAL TYPES OF IDEALS 128

That is a, −a ∈ S, where one of a or −a is a positive integer. Therefore P = mµ0 for some m ∈ Z, where µ0 = 1 or µ0 = p.
Therefore S contains some positive integers. If µ0 = 1 then µ0 (1) ⇒ U = Z.
Let s be the smallest positive integer in S. If µ0 p then µ0 = (p) ⇒ U = P .
We now verify that S = (s). Thus P ⊂ U ⊂ Z ⇒ U = P or U = Z. Therefore P is a maximal ideal of Z.
So, choose n ∈ S. Convetrsely, suppose the ideal P = (p) is maximal in Z, we verify that p is a
For n, s ∈ S, there exist q, r ∈ Z such that n = sq + r, where r = 0 or r < s. prime integer.
Therefore r = n − sq; n ∈ n ∈ S and sq ∈ S. This implies that r = n − sq ∈ S Let p = ab for a, b ∈ Z. Then we show that a = 1 or b = 1. Let U = (a) be
(∵ S is a ideal). Therefore r ∈ S, where r = 0 or r < s if r ̸= 0 then r ∈ S and any ideal in Z.
r < S which is a contrary. Now x ∈ P ⇒ x = mp for some m ∈ Z ⇒ x = m(ab). This implies that
Therefore r = 0 and hence n = sq for any n ∈ S. x = (mb)a, where mb ∈ Z. Therefore x ∈ P ⇒ x ∈ U for any x. Hence
Therefore S is a principle idel generated by s. i.e., S = (s). P ⊂ U ⊂ Z ⇒ U = P or U = Z.
Since S is an arbitrary ideal of Z, it follows that all ideals of Z are principle If U = P , then a ∈ U ⇒ a ∈ P .
ideals. Therefore Z is a principle ideal ring Therefore a = np for n ∈ Z ⇒ a = n(ab) ⇒ a = anb. Thus nb = 1, where
n, b ∈ Z ⇒ b = 1.
Corollary 11.1. Every field is a principle ideal ring.
Similarlly, if U = Z, then 1 ∈ Z ⇒ 1 ∈ U .
A field F has two ideals of R, i. e. F and {0} which are principle ideals. Therefore U = (a) = {ka : k ∈ Z}. This implies that 1 = ka ⇒ a = 1 or
Maximal ideal: b = 1. Hence P is a prime number.

Definition 11.3. An ideal M of a ring R is said to be a maximal ideal of R if Theorem 11.4. If R is a commutative ring with unity and M is an ideal of R,
M ⊂ U ⊂ R for any ideal U of R. This implies that U = M or U = R. R
then M is a maximal ideal of R if is a field.
M
Theorem 11.3. An ideal of the ring Z of integers is maximal if and only if it is R
Proof. Suppose M is a ideal of R, is a field.
generated by some prime integer. M
R
We verify that M is a maximal ideal of R. Since is a field, then there are
M
Proof. Suppose P is an ideal generated by p that is P = (p). where P is a prime R R
two ideals, M and in .
integer, which is an ideal in Z. M M
But under the natural homomorphism(epimorphism).
We verify that P is a maximal ideal. Let U be any ideal of Z satisfying R R
γ : R −→ , there is a one-one correspondence between and R.
P ⊂ U ⊂ R. Because, Z is a principle ideal ring and all of its ideals are principle M M
R
Let W and S be the ideals of R corresponding to the ideals M and respec-
ideal. Hence, U is a principle ideal. M
Let U = (µ0 ), where µ0 is an integer. Consider p ∈ P ⇒ p ∈ U and since
U = (µ0 ). Hence U = {mµ0 : m ∈ Z}.
11. UNIT: SPECIAL TYPES OF IDEALS 129 11. UNIT: SPECIAL TYPES OF IDEALS 130

R prime ideal of a ring R if ab ∈ P ⇒ a ∈ P or b ∈ P , where a, b ∈ R.


tively of . Therefore
M
Example 11.2.
W ={x ∈ R : γ(x) ∈ M }

={x ∈ R : M + x ∈ M } 1. In an integral domain R, the zero ideal {0} is a prime ideal. [a, b ∈ P
suppose ab ∈ {0}
={x ∈ R : x ∈ M }
ab = 0 ⇒ a = 0 or b = 0
W =R ∩ M.
⇒ a ∈ {0}, b ∈ {0}].
Therefore W = M (R ∩ M = M as R ⊂ M )
R 2. In the ring (Z, +, ·) the ideal P = (3) is a prime ideal.
S = {x ∈ R : γ(x) ∈ }
M
R Let a, b ∈ 3, then ab = 3k for k ∈ Z
S = {x ∈ R : M + x ∈ }
M ab
R =k
S = {x ∈ R : x ∈ } 3
M
R ⇒ 3|ab
S=
M
Thus M is the any proper ideal of R. ⇒ 3|a or 3|b
Therefore M is a maximal ideal. ⇒ a = 3k1 , b = 3k2
Conversely, suppose M is a maximal ideal of a commutative ring R with unity.
R ⇒ a ∈ {P } or b ∈ {P }
We verify that is a field.
M
R 3. In Z, the ideal M = (4) is not a prime ideal.
Because R is a commutative ring with unity, it follows that is also commu-
M
tative ring with unity M + 1. [2, 2 ∈ (4) but 2 ∈
/ (4)].
R
Further, under the natural homomorphism ϕ : R −→ , there is a one-one
M 4. In Z, the ideal M = (6) is not a prime ideal.
R
correspondence between the ideals of and the ideals of R. Because M is a
M [2, 3 ∈ (6) but 2 ∈
/ (6) or 3 ∈
/ (6)].
maximal ideal of R. There are only two ideals M, R in R.
R
But as proved in part I, the ideals M, R are related to M and respectively. Theorem 11.5. In a commutative ring with unity, a maximal ideal is always a
M
R R prime ideal.
Hence is a commutative ring with unity with only two ideals M and .
M M
R
Therefore is a field. Proof. Let R be a commutative ring with unity and M be a maximal ideal of R.
M
Prime Ideal: To verify that M is a prime ideal of R. Let shall claim that either a ∈ M or
b ∈ M . Suppose a ̸= M , now consider the ideal such that
Definition 11.4. Let R be a commutative ring. An ideal P of R is said to be a
(a) + M = {ax + m : x ∈ R, m ∈ M } of R.

11. UNIT: SPECIAL TYPES OF IDEALS 131 11. UNIT: SPECIAL TYPES OF IDEALS 132

Satisfying M ⊂ (a) + M ⊂ R Example 11.3. R = 2Z = {0, ±2, ±4, . . .} (commutative ring with unity)
Since M is maximal ideal of R, M = (4) = {0, ±4, ±8, ±12, . . .} is an ideal of R. (4) is not prime ideal but a
(a) + M ⊂ M ⇒ (a) + M = M or (a) + M = R. maximal ideal of R.
Clearly (a) + M ̸= R [∵ (a) ∈
/ M and a ∈ (a) + M ]
Theorem 11.6. In a prime ideal ring, every non zero prime ideal is maximal.
Hence (a) + M = R
Now 1 ∈ R ⇒ 1 ∈ (a) + M Proof. Let R be a prime ideal ring and M be a prime ideal of R. We verify that
⇒ 1 = ax + M for some x ∈ R, m ∈ M M is a maximal ideal of R
⇒ b = abx + mb where abx ∈ R, mb ∈ M Let U be an ideal of R satisfying M ⊂ U ⊂ R.
⇒ b = abx + mb ∈ M for some abx ∈ M We shall claim that M = U or U = R, because R is a principle ideal ring,
Therefore M is a prime ideal. whose all ideals of R are principle ideals.
Therefore M and U are principle ideals. (∵ R is PIR)
Let F = {γ : [0, 1] −→ R ; γ is constant}
Let M = (S) and U = (t), now
 
1
Define ϕ : F −→ R by ϕ(γ) = γ which is a homomorphism.
2  S ∈ M ⇒ S ∈ M ⊂ U ⇒ S ⊂ U ⇒ S = tr for r ∈ R
1
Therefore M = kerϕ = {γ ∈ F ; γ = 0} S = tr ∈ M
2
Show that M is a prime  ideal also,
  Therefore t ∈ M or r ∈ M
1
Therefore γ1 , γ2 ∈ M ⇒ γ1 γ2 =0 Consider t ∈ M ⇒ tx ∈ M for some x ∈ R
    2   
1 1 1 1 Thus tx ∈ U ⇒ tx ∈ M , U ⊂ M where M ⊂ U
⇒ γ1 γ2 = 0 where γ1 γ2 ∈R
2 2   2 2
U = M −→ (1)
1 1
⇒ γ1 = 0 or γ2 =0
2 2 Similarlly, r ∈ M ⇒ r = y ∈ M for some y ∈ R
⇒ γ1 ∈ M or γ2 ∈ M
r = try (s = tr)
Therefore M is a prime ideal.
1 = ty ∈ U for some y
Note 11.2. Therefore 1 ∈ U
⇒ U = R −→ (2)
(i) In any a commutative ring with unity, a prime ideal need not be maximal
Thus from (1) and (2), we have
ideal.
M ⊂ U ⊂ R ⇒ U = M or U = R
[∵ In Z ( a commutative ring with unity) P = {0} is a prime ideal but not
Thus M is maximal.
a maximal ideal of Z].
Theorem 11.7. Let R be an integral domain with ideal P , then P is a prime idel
(ii) The resultof above theorem is not true if R does not contain the unity (unit R
if and only if is an integral domain.
element). P
11. UNIT: SPECIAL TYPES OF IDEALS 133 11. UNIT: SPECIAL TYPES OF IDEALS 134

R R
Proof. Let P be a prime ideal of R. We verify that is an integral domain. Since ∵ If P is a prime ideal of R. ⇒ is an integral domain.
P P
R R
R is a commutative ring, it being an integral domain it follows that is also a ⇒ is a field.
P P
commutative ring.
⇒ P is a maximal ideal.
For a, b ∈ R consider P+a P+b = P . Thus Pab = P
Therefore ab ∈ P ⇒ a ∈ P or b ∈ P 3. Every non zero prime ideal of a principle ideal ring is a maximal ideal.

P+a = P or P+b = P .
SUMMARY
R
Therefore has no zero divisor.
P
R A ring in which every ideal is principal is called principal, or a principal ideal ring.
Thus is an integral domain.
P A a prime ideal is a subset of a ring that shares many important properties of a
R
Conversely, suppose is an integral domain .
P prime number in the ring of integers. The prime ideals for the integers are the
We verify that P is a prime ideal
sets that contain all the multiples of a given prime number, together with the zero
Choose ab ∈ P for some a, b ∈ P .
ideal. A a maximal ideal is an ideal that is maximal (with respect to set inclusion)
Therefore P+ab = P .
amongst all proper ideals.
Hence (P+a )(P+b ) = P .
KEYWORDS
Therefore (P+a ) = P or (P+b ) = P . Therefore a ∈ P or b ∈ P ⇒ ab ∈ P ⇒
a ∈ P or b ∈ P . Ideals, principle ideal, principle ideal ring, prime ideal, maximal ideal.

Thus P is a prime ideal.


ASSESSMENT / TERMINAL QUESTIONS
Note 11.3. In view of the theorems, we prove that every maximal ideal is a com-
1. Show that all fields are principal ideal rings.
mutative ring with unity is a prime ideal.

2. Investigare whether the rings R = 2Z and I = 4Z are prime ideals.


1. In a commutative ring with unity, maximalideal ⇒ prime ideal
3. Investigare whether the rings R = 2Z and I = 4Z are maximal ideals.
Proof. M is a maximal ideal in a commutative ring with unity.
R R 4. Find all the prime ideals and the maximal ideals of Z16 .
is a field. ⇒ is an integral domain.
M M
R 5. Let R be the ring all real valued continuous functions defined on the closed
⇒ is a prime ideal.
M interval [0, 1]. Let M = {f (x) ∈ R : f ( 13 ) = 0}. Show that M is maximal
(Here instead of M , we can choose a P as maximal ideal to prime ideal).
ideal of R.

2. If R is a finite commutative ring with unity, we prove that every prime ideal 6. Let R = Z[i] = {a + ib : a, b ∈ Z}. Let M = ⟨2 + i⟩. Then show that M is a
of R is a maximal ideal. maximal ideal of R.

11. UNIT: SPECIAL TYPES OF IDEALS 135 136

7. Let R be the ring of n × n matrices over reals. Show that R has only two 12 UNIT: The field of Quotients
ideals {0} and R. Hence show that {0} is maximal ideal od R.

8. Give an example of a prime ideal, which is not maximal. OBJECTIVES

REFERENCES After studying this unit, you should be able to:

1. I. N. Herstein: Topics in Algebra, 2nd Edition, John Wiley and Sons, 2008. • define imbedding of rings and known its elementary properties;

2. Surjeet Singh and Qazi Zameeruddin: Modern Algebra, 8th Edition, Vikas • explain the some special classes of imbedding rings along with examples;
Publishing House, 2021.
• explain some fundamental lemma’s of imbedding rings ;
3. N. Jacobson: Basic Algebra-I, 2nd Edition, Dover Publications, 2009.
• solveing some problems of imbedding rings ;
4. M. Artin : Algebra, 2nd Edition, Prentice Hall of India, 2015.
• discuss the nature of field of quotients of an integral domain.
5. Darek F. Holt, Bettina Eick and Eamonaa A. O’brien: Handbook of com-
INTRODUCTION
putational group theory, 1st Edition, Chapman & Hall/CRC Press, 2005.
In this unit, the concept of imbedding the rings has been explained. The definition
6. J. B. Fraleigh : A first course in abstract algebra, 7th Edition, Addison-
and important theorem related to imbedding the rings are presented as follows.
Wesley Longman, 2002.

7. D. S. Dummitv and R.M. Foote: Abstract Algebra, 2nd Edition, John Wiley
Definition 12.1. A ring R is said to be imbedded into a ring R1 if there is an
& Sons, 2003.
isomorphism of R into R1 , that is R is imbedded into R1 if R ≈ S where S ⊂ R1 .
8. V. K. Khanna and S. K. Bhambri: A course in Abstract algebra, 4th Edition, Further if R and R1 are unit elements 1 and 11 respectively.
Vikas Publishing House Pvt Ltd., 2013. [Then we insist that the isomorphism should take 1 onto 11 ].

Theorem 12.1. Every integral domain can be imbedded into a field.

Proof. Let R be an integral doimain. Consider the set

S = R × (R − {0}) = {(a, b)|a, b ∈ R, b ̸= 0}.

a
We can look upon the elements of S as fractions where both the numerator a
b
and denomirator b in R, b ̸= 0.
12. UNIT: THE FIELD OF QUOTIENTS 137 12. UNIT: THE FIELD OF QUOTIENTS 138

a ac
We need to introduce a relation in S in order to consider the fractions and , Then ab′ = a′ b and cd′ = c′ d. Thus, we have,
b bc
for nay c ∈ R, c ̸= 0, identical.
Define a relation ∼ on S by (a, b) ∼ (c, d) if ad = bc, ∼ is an equaivalence (ad + bc)b′ d′ = adb′ d′ + bcb′ d′

relation: = (ab′ )dd′ + (cd′ )bb′


Clearly (a, b) ∼ (a, b). If (a, b) ∼ (c, d), then ad = bc which implies da = cb since = (a′ b)dd′ + (c′ d)bb′
R is commutative. = a′ d′ bd + b′ c′ bd
Hence (c, d) ∼ (a, d).
= (a′ d′ + b′ c′ )bd.
Suppose (a, b) ∼ (c, d) and (c, d) ∼ (e, f ).
Then ad = bc and cf = de. Thus, we have, daf = adf = bcf = bde = dbe.
  a′ d′ +b′ c′ 
This implies ad+bc

bd
= b′ d′
.
Since d ̸= 0 and R is an integral domain, we can cencel d to obtain af = be that
 a   c   a ′   c′ 
Therefore b + d = b′ + d′ and thus addition is well-defined.
is (a, b) ∼ (e, f ). The operation of multiplication can be similarly varified to be well-defined. Fur-
Hence ∼ is an equivalence relation on S. ther,
Let ab denote the equivalence class caontaining (a, b) called the fraction associated
 
1) Addition in F is commutative:
with (a, b).
Let F denote the collection of all fractions. i.e., hai hci h ad + bc i
+ =
b d db
n hai o h cb + da i
F = |a, b ∈ R, b ̸= 0 . =
b h c ibd h a i
= + .
d b
We define addition and multiplication operations in F to make it into a field:

2) Addition is associative:
hai hci   h i h i h i
ad + bc a c ac
+ = , × = .
b d bd b d bd
h a i h c i e h a i h c i h e i
+ + = + +
b d f b d f
a  a′ 
Note that b
if and only if a′ b = ab′ . Also b ̸= 0, b ̸= 0 implies bd ̸= 0 as
= b′

R is an integrel domain and so ad−bc


   ac 
bd
, bd ∈ F . 0
is the zero element, for 01 + ab = ab .
     
3) 1
Addition is well-defined: h0i h0i
We note that = for any non-zero a in R.
1 a
   ′    ′
Suppose ab = ab′ and dc = dc′ .

12. UNIT: THE FIELD OF QUOTIENTS 139 12. UNIT: THE FIELD OF QUOTIENTS 140

 −a  a h a ih c i h a ih e i h ac i h ae i
4) b
is the additive inverse of b
: + = +
b d b f bd bf
h acbf + bdae i
hai h −a i h −ab + ba i
=
+ = bdbf
b b b2 h acf + ade i
h0i
= = ,
b2 bdf
h0i
= .
1 since b ̸= 0.
h ih i h i
a b 1
a  −a 
Thus − b
= b
. 8) For any a, b ∈ R, both non-zero, we have, b a
= 1
.
h i
Now for any ab ∈ F , we have
5) Multiplicative is associative:
h a ih 1 i hai
h a ih c ih e i h ac ih e i = ,
= b 1 b
b d f bd f
h ace i
=
h i
1
bdf ans so 1
is the identity element deoted simply by 1.
h a ih ce i
= h i
b df 9) Let x = ab be a non-zero element of F . Then a ̸= 0 (and b ̸= 0). Let
h a ih c ih e i h i
= . y = ab . Then,
b d f
h a ih b i
6) Multiplication is commutative: xy =
b a
h ab i
h a ih c i h ac i =
= ba
b d bd i h1i
h ca = .
= 1
h db
c ih a i
= . Hence x is a unit in F .
d b
h i
a
Thus F is a field. Now define the map f : R → F by f (a) = 1
.
7) The distributive law holds in F :
Then
h a ih c i h e i h a ih cf + de i
+ = ha + bi
b d f b df f (a + b) =
h acf + ade i a
= , and hai h b i
bdf = +
1 1
= f (a) + f (b) and,
12. UNIT: THE FIELD OF QUOTIENTS 141 12. UNIT: THE FIELD OF QUOTIENTS 142

h ab i
f (ab) = Similarly,
1
h a ih b i
=
1 1 (ab)−1 (cd)−1 =ac(d−1 b−1 )
= f (a)f (b).
− =(ac)(bd)−1 ∈ K
h i
a
h i
b =ab−1 cd−1 ∈ K.
Further, f (a) = f (b) =⇒ 1
= 1
i.e., a = b.
Hence f is an embedding of R into the field.
Further for a ̸= 0 in D, we have aa−1 ∈ K 1 .
Field of Quotients of an integral domain: Therefore 1 ∈ K 1 , because K 1 is a subring of K containing unity.
Therefore K 1 is a subfield of K.
Definition 12.2. Let D be an integral domain and F = {a, b such that a, b ∈ D ̸=
We verify that K 1 ≈ F where F = {[a, b] : a, b ∈ D, b ̸= 0} is the quotient field
0} be the set of equivalence class under the equivalence relation given by (a, b) ∼
of D.
(c, d) if ad = bc, then F becomes a field and this field is called an quotients of
Define ϕ : F −→ K 1 by ϕ[a, b] = ab−1 .
field of an integral domain D.
For [a, b], [c, d] ∈ F , Consider
Theorem 12.2. If K is any field containg an integral domain D, then K contains
a subfield isomorphic to the quotient field F (D). ϕ([a, b] + [c, d]) =ϕ([ad + bc, bd])

=(ad + bc)(bd)−1
Proof. Let D be an integral domain and K be be a field containing D. For any
a, b ∈ D(b ̸= 0), we have a, b ∈ K. =ad + bc(d−1 b−1 )

Hence (ab)−1 ∈ K. =ab−1 + cd−1


1 −1
Let K = {(ab) ∈ K such that a, b ∈ D(b ̸= 0)} ⊂ K. =ϕ[a, b] + ϕ[c, d]
We verify that K 1 is a sub field of K.
For this, it is enough to show that K 1 is a subring of K, containing unity Similarly,

because other properties of field follows from K. Consider,


ϕ([a, b][c, d]) =ϕ([ac, bd])

(ab)−1 − (cd)−1 =add−1 b−1 − cbb−1 d−1 =(ac)(bd)−1

=(ad − cb)b−1 d−1 =(ac)(d−1 b−1 )

=ab−1 − cd−1 ∈ K. =(ab−1 )(cd−1 )

=ϕ[a, b]ϕ[c, d]

12. UNIT: THE FIELD OF QUOTIENTS 143 12. UNIT: THE FIELD OF QUOTIENTS 144

Therefore ϕ is homomorphism. Excercise 12.1. Show that every element x of F can be expresesd as x = ab−1 , a, b ∈
Let ϕ([a, b]) = ϕ([c, d]). This implies that ab−1 = cd−1 . R, b ̸= 0.
Therefore ad = bc. Hence (a, b) ∼ (c, d). That is, [a, d] = [c, d].
Solution. We identity the element of R with their image in F , The element a
Therefore ϕ is one to one.
h i
a
being identifed with f (a) =1
. We can then regand R as a subring of F . For
For any ab−1 ∈ K 1 , we have ab ∈ D and b ̸= 0
h i
x = ab ∈ F , we have,
Therefore ab ∈ F and ϕ([a, b]) = ab−1
hai h a ih 1 i
Therefore ϕ is onto.
=
1 b 1 b
In view of the above facts, we have ϕ is an isomorphism of F on to K . h a ih b i−1
=
This proves that F is the smallest field containing D. 1 1
= f (a)f (b)−1
Corollary 12.1. The quotient field of a finite integral domain consider with itself.
= ab−1 ,
Note 12.1. If D is a finite integral domain, then it is a field. Infact it is the
smallest field containing D. But quotient field F (D) is the smallest field containing undwer the identification. Thus the elements of f can be expressed as quotients
D(By above theorem). of elemets of R.

Corollary 12.2. Any two isomorphic integral domain have isomorphic quotient Excercise 12.2. Show that the field F is the smallest field containing R.
field.
Solution. Let F ′ be any field containing R. Then for any x ∈ F , we have
Proof. Let D and D1 be the two isomorphic integral domain under the isomor- x = ab−1 , a, b ∈ R, b ̸= 0. Since R ⊆ F ′ and since F ′ is a field, x = ab−1 ∈ F ′ .
phism D onto D1 [i.e. ϕ : D −→ D1 ]. Thus F ⊆ F ′ .
Suppose F and F 1 are the quotient fields of D and D1 respectively.
Therefore, F = {[a, b] : a, b ∈ D} and F = {[ϕ(a), ϕ(b)] : a, b ∈ D}. KEYWORDS
By above theorem and corollary, we have F ≈ F 1 .
Imbedding of ring; Field of Quotients of an integral domain.

SUMMARY
ASSESSMENT / TERMINAL QUESTIONS

In this unit, we discuss the conditions under which a ring R cab be embedded in a 1. If [a, b] = [a′ , b′ ] and [c, d] = [c′ , d′ ], then prove that [a, b][c, d] = [a′ , b′ ][c′ , d′ ].
field. Clearly, a necessary condition for this is ring R must be an integral domain.
2. Show that the quotient field is the smallest field containing D.
We show that this condition is sufficient also by proving integral domain R can be
embedded in a field. The construction of this field is similar to the construction 3. Show that any ring can be embedded into a ring with unity [Hint. Embed-
of the field of rationals Q from the ring of integers Z. ding of a ring into ring of endomorphisms].
12. UNIT: THE FIELD OF QUOTIENTS 145 12. UNIT: THE FIELD OF QUOTIENTS 146

4. Show that any ring R with unity can be embedded into a ring of endomor- BLOCK 4: ADVANCED RING THEORY
phisms of some additive abelian group.

5. Show that the quotient field of a finite integral domain coinsides with itself.
UNIT-13: Euclidean rings
6. What is the field of quotients of a finite integral domain?
UNIT-14: Classical Theorems on Eucliden rings
7. What is the quotient field of 2Z, where Z is the ring of integers.
UNIT-15: Polynomial rings
8. Show that an integral domain can be embedded into a field.
UNIT-16: Polynomial over the Rational field
REFERENCES

1. I. N. Herstein: Topics in Algebra, 2nd Edition, John Wiley and Sons, 2008.

2. Surjeet Singh and Qazi Zameeruddin: Modern Algebra, 8th Edition, Vikas
Publishing House, 2021.

3. N. Jacobson: Basic Algebra-I, 2nd Edition, Dover Publications, 2009.

4. M. Artin : Algebra, 2nd Edition, Prentice Hall of India, 2015.

5. Darek F. Holt, Bettina Eick and Eamonaa A. O’brien: Handbook of com-


putational group theory, 1st Edition, Chapman & Hall/CRC Press, 2005.

6. J. B. Fraleigh : A first course in abstract algebra, 7th Edition, Addison-


Wesley Longman, 2002.

7. D. S. Dummitv and R.M. Foote: Abstract Algebra, 2nd Edition, John Wiley
& Sons, 2003.

8. V. K. Khanna and S. K. Bhambri: A course in Abstract algebra, 4th Edition,


Vikas Publishing House Pvt Ltd., 2013.

147 13. UNIT : EUCLIDEAN RINGS 148

13 UNIT : Euclidean rings Note 13.1.

(i) When we say, in the definition of euclidean domain, that there exist a non
negative integer d(a) for any a ̸= 0, we mean, there exist a function d from
OBJECTIVES R − {0} to Z+ ∪ {0}, where Z+ is set of positive integers. This function d is
called euclidean valuation on R. Also, the last condition in the definition
After studying this unit you will be able to: is called euclidean algorithm.
• define euclidean rings and known its elementary properties; (ii) We can show that the t and r mentioned in euclidean algorithm condition
in the definition of euclidean domain are uniquely determined if and only if
• explain some fundamental properties of euclidean rings;
d(a + b) ≤ M ax.{d(a), d(b)}.
• explain the some special classes of euclidean rings along with examples; Let d(a + b) ≤ M ax.{d(a), d(b)}. Suppose a = tb + r = t1 b + r1 .
Let r1 − r ̸= 0, then b(t − t1 ) = r1 − r ̸= 0, and so t − t1 ̸= 0. Now
• understanding the nature of every field is an euclidean ring;

• describe the importance of every euclidean ring is a principle ideal ring ; d(b) ≤ d(b(t − t1 ))
= d(r1 − r)
• solveing some problems related to euclidean rings.
≤ M ax.{d(r1 ), d(−r)} given condition
= M ax.{d(r1 ), d(−r)}
INTRODUCTION < d(b), which is not possible.
In this unit, we study about divisibility in rings in a generalized form in terms of
euclidean domain. A ring without zero divisors in which an integer norm and an Thus r1 − r = 0 =⇒ b(t − t1 ) = 0 or t − t1 = 0 as b ̸= 0. This implies that
associated division algorithm (i.e., a Euclidean algorithm) can be defined. t = t1 and r = r1 .
Conversley, let t, r be uniquely determined and suppose d(a+b) > M ax.{d(a), d(b)}
Euclidian Ring: for some a, b(non zero) in R.
Now b = 0(a + b) + b = 1.(a + b) − a.
Definition 13.1. Let R be a commutative ring without zero divisos(Integral do- Also d(−a) = d(a) < d(a + b) and d(b) < d(a + b). Thus for b, 1 ∈ R, there
exist t = 0, r = b or t1 = 1, r1 = −a such that b = t.1+r, b = t1 .1+r1 , where
main), then R is said to be an Euclidean ring or If there exists a function r ̸= r1 (as a + b ̸= 0), t ̸= t1 , which is a contradction to the uniqueness.
D : R −→ Z + satisfying the following condition Hence d(a + b) ≤ M ax.{d(a), d(b)}.

(i) d(a · b ≥ d(a) ∀ a, b ∈ R − {0}. Theorem 13.1. Z[i] = {a + ib : a.b ∈ Z}, the set of Gaussian integers, is a
Euclidean ring.
(ii) For any a, b ∈ R − {0}, there exists t, r ∈ R such that a = bt + r where r = 0
or d(r) < d(b). Proof. Define d : Z[i] −→ {0} −→ Z + by d(x) = |x|2 ; x = a + ib. Therefore
d(a + ib) = |a + ib|2 = a2 + b2 .
Example 13.1. The ring (Z, +, ·) is an Euclidean ring.
For any x, y ∈ Z[i] − {0}, we have |xy|2 = |x|2 |y|2 ≥ |x|2 . That is |xy|2 ≥ |x|2 .
Solution. Define d : Z −→ Z + by d(a) = |a|. Then for any a, b ∈ Z − {0} we
Therefore d(x, y) ≥ d(x).
have |ab| = |a||b| ≥ |a|. That is |ab| ≥ a) ⇒ d(ab) ≥ d(a). For any a, b ∈ Z − {0},
such that t, r ∈ Z. By division algorithm, we have a = bt + r, where r = 0 or
r < |b| or d(r) < d(b). Therefore Z is an Euclidean ring. Example 13.2. Every field is an Euclidean ring.
13. UNIT : EUCLIDEAN RINGS 149 13. UNIT : EUCLIDEAN RINGS 150

Solution. Let R be a field. It follows that the ring R is a commutative ring Note 13.2. Euclidean ring possesed unity.
without zer divisors.
Therefore R is an integral domain. Definition 13.2. An element d ∈ R is said to be GCD of a, b ∈ R if
Define a function d : R−{0} −→ Z + by d(a) = |a|. Then for any a, b ∈ R−{0},
we have (i) d|a and d|b
|ab| = |a||b| ≥ |a| (∵ |b| = 1).
Therefore d(ab) ≥ d(a). That is d(ab)d(a).
(ii) c|a and c|b
For any a, b ∈ R − {0}, there exists t, r ∈ R such that a = bt + r, where r = 0
or r < b.
Therefore a = bt + r, where d(r) < d(b). This concept is denoted by d = (a, b).
Hence the function d satisfies the condition of Euclidean ring.
Therefore the field is an Euclidean ring. Lemma 13.1. Let R be an Euclidean ring. Then a, b ∈ R have GCD, d moreover
d = (a, b), d = λa + µb for λ, µ ∈ R.
Theorem 13.2. Every Euclidean ring is a principle ideal ring.

Proof. Let R be an Euclidean ring and S be any ideal of R. We verify that S is Proof. Let A = {ra + sb : r, s ∈ R} ⊂ R. To verify that A is an ideal of R.

an principle ideal ring. Let x = r1 a + s1 b and y = r2 a + s2 b.

If S = {0}, then S is an principle ideal generated by (0). That is S = (0). Therefore x − y = (r1 − r2 )a + (s1 − s2 )b

Suppose S ̸= {0}, then there exists non zero elements in S. Let a ∈ S where x − yA as r1 − r2 ∈ R, s1 − s2 ∈ R.

a ̸= 0 with least d(a) (i.e. d(a) < d(x) ∀ x ∈ S). Futher, for any u ∈ R, x = r1 a + s1 b, we have ux = u(r1 a + s1 b). This implies

Now we verify that S is a principle ideal generated by a0 . that ux = ur1 a + us1 b. Therefore ux = (ur1 )a + us1 )b.

Therefore S = a0 . Since ur1 ∈ R, us1 ∈ R.

Let a ∈ S. Then we have aa0 ∈ S ⇒ aa0 ∈ R − {0}. That is a, a0 are non zero Therefore ux ∈ A and also xuıA.

elements of an euclidean ring R. Hence A is an ideal of R.

For non zero element a, a0 ∈ R, there exists t, r ∈ R such that a = a0 t + r, But every ideal in A of an euclidean ring R is a principle ideal, we have A = d

where r = 0 or r < a0 . for d ∈ A.

Therefore a = a0 t + r where r = 0 or d(r) < d(a0 ). Therefore d = λa + µb for some λ, µ ∈ R.

Hence r = a − a0 tS. That is r ∈ S and d(r) < d(a0 ), which is contradiction, Now, we verify that d is G ⊂ D, i.e. d = (a, b)

since d(a0 ) is the least in S. Because R is a Euclidean ring, it possess unity 1.

Therefore r = 0. But then a = 1.a + 0.b ∈ A, we have b = 0.a + 1.b ∈ A.

Hence a = a0 t for any a ∈ S (All elements of S are multiples of a0 ). Therefore a, b ∈ A, where A is an ideal generated by d (A = (d)).

Therefore S is a principle ideal generated by a0 . That is S = a0 and because Suppose c ∈ R satisfies c|b and c|a then c|µb and c|λa ⇒ c|λa + µb

S is an arbitrary ideal of R, it follows that all ideals of R are principle ideals. ⇒ c|d, where d = λa + µb.

Therefore R is a principle ideal ring. Thus the result follows.

13. UNIT : EUCLIDEAN RINGS 151 13. UNIT : EUCLIDEAN RINGS 152

Lemma 13.2. Let R be an Euclidean ring and a, b ∈ R. Then Hence a = (ab)x for some x ∈ R implies that a(1 − bx) = 0.

(i) d(ab) = d(a) if b is unit in R Here a ̸= 0, since A = (a), we have 1 − bx = 0.

Hence 1 = bx for some x ∈ R.


(ii) d(ab) > d(a) if b is non unit in R
Therefore b is an unit in R, which is not true.
Proof. (i) By the definition of an euclidean ring
Thus d(ab) ̸= d(a) implies that d(ab) > d(a), when b is a non unit in R.

d(ab) ≥ d(a) −→ (1)

Note 13.3.
If b is a unit in R, then b−1 exists in R.
(i) A necessary and sufficient condition for a non zero element a in an Euclidean
Therefore d[(ab)b−1 ] ≥ d(ab), we have ring to be a unit is d(a) = d(1).
[a ∈ R(a ̸= 0), we have a is a unit in R. Then by (i) of above lemma, we
d(a) ≥ d(ab) −→ (2) have d(1a) = d(1) ⇒ d(a) = d(1)].
From equations (1) and (2), we hae (ii) Z[i] is the fourth root of unity or the units of Z[i] = ±1, ±i

d(ab) = d(a), when b is a unit in R. √ √


Excercise 13.1. Show that Z[ 2] = {a + 2b : a, b ∈ Z} is an euclidean domain.

(ii) Let A be an ideal of R. Solution:
√ It is easy to see that Z[ √2] is an integral domain. Define a mapping.
d : Z[ 2] − {0} −→ Z by d(a + 2b) = |a2 − 2b2 |. Then |a2 − 2b2 | ≥ 1 as
Since R is a principle ideal ring. √ a
a − 2b2 = 0 ⇒ 2 = , which is not possible.
2
b
Therefore Ais a principle ideal. Again,
√ √ √
Let A = (a) = {xa : x ∈ R. We note that d(xa) ≥ d(a) (By the definition d[(a + 2b)(c + 2d)] = d[(ab + 2bd) + 2(ad + bc)]
of an euclidean ring). = |(ac + 2bd)2 − 2(ad + bc)2 |
= |(a2 − 2b2 )(c2 − 2d2 )|
Therefore a is a generator of an ideal if d(a) is the least integer among all
= |a2 − 2b2 ||c2 − 2d2 |
elements of the idel. ≥ |a2 − 2b2 |

= d(a + 2b) −→ (1)
We now verify that d(ab) ̸= d(a). √ √ √
i.e., d(a + 2b) ≤ d[(a + 2b)(c + 2d)].
Let us assume that b is a non unit in R.
√ √ √ √
Let now a + 2b and c + 2d be two member of Z[ 2] and suppose c + 2d ̸= 0,
Suppose d(ab) = d(a).

Therefore abis also a generator of A. (i.e. (ab) = A)

Therefore a ∈ A ⇒ a ∈ ab.
13. UNIT : EUCLIDEAN RINGS 153 13. UNIT : EUCLIDEAN RINGS 154

then require that a, b, c, d are itegers.]


√ √ √
a + 2b (a + 2b)(c − 2d) =⇒ d(r) = |c2 − 2d2 ||(m − p)2 − 2(n − q)2 |
√ =
c + 2d c2 − 2d2 ≤ |c2 − 2d2 ||(m − p)2 + 2(n − q)2 |

ac − bd 2(bc − ad) 1 2
= 2 + ≤ |c2 − 2d2 || + |
c −√2d2 c2 − 2d2 4 4 √
= m + 2n say ≤ |c2 − 2d2 | = d(c + 2d)
√ √ √ √ √
where m and n are rationals. Hence, for a + 2b, c + 2d ∈ Z[ 2] ∃ p + 2q, r ∈ Z[ 2] such that
Now m = [m] + θ where [m] is the greatest integer not greater than m and θ is
√ √ √
fractional part of m. (a + 2b) = (c + 2d)(p + 2q) + r,
1 1
If 0 ≤ θ ≤ , take p = [m] and if < θ < 1, take p = [m] + 1.
2 2 √ √
1 where either r = 0 or d(r) < d(c + 2d) showing that Z[ 2] is an Euclidean
Thus there exist an integer p, such that |m − p| ≤ . domain.
2
Similarly, we can find an integer q, such that |n − p| ≤ 12 .
Put m − p = α, n − p = β, we have |α| ≤ 12 , |β| ≤ 12 . SUMMARY
Also we have √
a + 2b √ An Euclidean domain (also called a Euclidean ring) is an integral domain that
√ = (p + q) + 2(q + β) can be endowed with a Euclidean function which allows a suitable generalization
c + 2d
√ of the Euclidean division of the integers. This generalized Euclidean algorithm
a + 2b √ √ can be put to many of the same uses as Euclid’s original algorithm in the ring
=⇒ √ = (p + 2q) + (α + 2β)
c + 2d of integers: in any Euclidean domain, one can apply the Euclidean algorithm to
√ √ √ √ √ compute the greatest common divisor of any two elements.
=⇒ a + 2b = (c + 2d)(p + 2q) + (c + 2d)[(m − p) + 2(n − q)],
√ √ KEYWORDS
where, of course, (p + √2q) ∈ Z[ √ 2] as p, q √
are integers √
we can
√ thus write a + 2b = (c +√ 2d)(p + √2q) + r, where √ r = (c + 2d)[(m − Euclidean domain, Euclidean algorithm, Euclidean valuation, Greatest common
p) + 2(n − q)] √ and as r = (a + 2b) − (c + 2d)(p + 2q) divisor.
we notice r ∈ Z[ 2].
Now if ASSESSMENT / TERMINAL QUESTIONS
√ √
r ̸= 0, d(r) = d[(c + 2d){(m − p) + ((n − q) 2}] 1. In a euclidean domain R with valuation d, show that
√ √
= d[(c + 2d)][d((m − p) + (n − q) 2)]. (i) d(a) = d(−a) for all 0 ̸= a ∈ R.
(ii) if a, b are associates, then d(a) = d(b).
[using equation (1) one may notice here that in proving (1) we do not essentially
(iii) if a | b and d(a) = d(b), then a, b are associates.
(iv) if for 0 ̸= a ∈ R, d(a) = 0, then a is unit.
(iv) if a | b and a is not an associate of b, then d(a) < d(b)
(a, b ̸= 0).

2. Show that the ring of every integer in not an euclidean domain.

13. UNIT : EUCLIDEAN RINGS 155 156


3. Show that R = {a + b −5} : a, b ∈ Z is not a euclidean domain. 14 UNIT : Classical Theorems on Eucliden rings
4. State and prove the division algorithm.

5. Show that every ideal in a euclidean domain is a principal ideal.


OBJECTIVES
6. Show that an euclidean domain is a principal ideal domain (PID).
After studying this unit you will be able to:
7. Show that in a principal ideal domain (PID), every non-zero prime ideal is
maximal. • explain the concepts of prime element in euclidean ring;
8. Let R be an euclidean domain and let A be an ideal of R, then there exist
ao ∈ A such that A = {ao x : x ∈ R}. • define the concepts of relatively prime element ;

REFERENCES • describe the inique factorization theorem and their significance ;


1. I. N. Herstein: Topics in Algebra, 2nd Edition, John Wiley and Sons, 2008.
• explain some fundamental lemmas of fermat’s theorem;
2. Surjeet Singh and Qazi Zameeruddin: Modern Algebra, 8th Edition, Vikas
Publishing House, 2021. • state and prove the fermat’s theorem .

3. N. Jacobson: Basic Algebra-I, 2nd Edition, Dover Publications, 2009.

4. M. Artin : Algebra, 2nd Edition, Prentice Hall of India, 2015. INTRODUCTION


5. Darek F. Holt, Bettina Eick and Eamonaa A. O’brien: Handbook of com-
putational group theory, 1st Edition, Chapman & Hall/CRC Press, 2005. In this unit, we study the unique factorization theorem, which states that every
positive whole number can be expressed as a product of prime numbers in essen-
6. J. B. Fraleigh : A first course in abstract algebra, 7th Edition, Addison-
Wesley Longman, 2002. tially only one way. Also, we study the Fermat’s theorem on sums of squares.

7. D. S. Dummitv and R.M. Foote: Abstract Algebra, 2nd Edition, John Wiley
& Sons, 2003. Prime element in Euclidean ring:
8. V. K. Khanna and S. K. Bhambri: A course in Abstract algebra, 4th Edition,
Definition 14.1. In the Euclidean ring , a non unit element p is said to be a
Vikas Publishing House Pvt Ltd., 2013.
prime element if p = ab for a, b ∈ R. one of a and b is a unit in R.

Example 14.1. In any euclidean ring R, one is a prime element of R. That is


1 = 1.1 (1 is a unit).

Example 14.2. In the ring R of integers, all prime integers are prime element,
because 13 = 1 × 13 1 is a unit of Z.

Note 14.1. Let R be a euclidean ring. Then any ideal A = a0 is a maximal ideal
in R if and only if a0 is a prime element of R.
14. UNIT : CLASSICAL THEOREMS ON EUCLIDEN RINGS 157 14. UNIT : CLASSICAL THEOREMS ON EUCLIDEN RINGS 158

Definition 14.2. In an Euclidean ring R, the element a and b are said to be Proof. Since P1 |p1 , p2 , p3 , ·, pn and p1 , p2 , · · · , pn = p11 , p12 , · · · , p1m , we have
relatively prime element of R is their G ⊂ D is the unity of R. P1 |p11 , p12 , · · · , p1m .
We denote this concept by (a, b) = 1 Therefore P1 |atleast one of (p11 , p12 , · · · , p1m ).

Note 14.2. So assume P1 |p1 . That is p11 = u1 p1 .

Therefore u1 ∈ R is a unit in R(Because by exchanginging u1 with u1 p11 ).


(i) Ler R be an Euclidean ring. Suppose that for a, b, c ∈ R with a|bc and
(ab) = 1 then a|c. Hence p1 and p11 are associative. ⇒ p1 p2 , · · · , pn = u1 p1 p12 · · · p1m

p2 p3 · · · pn = u1 p12 p13 · · · p1m −→ (1)


(ii) If P is a prime element in an Euclidean ring R and P |ab, ∀ ab ∈ R then
P atleast one of a or b. Now, we can repeat the above argument on relation (1), with p2 we have
p3 · · · pn = u1 u2 p13 · · · p1m
[P † a ⇒ (p, a) = 1
Containing the same and if m > n, argument n times
[P |ab and (p, a) = 1 ⇒ P |b]

(iii) P |a1 a2 · · · an P = u1 u2 · · · un p1n+1 p1n+2 · · · p1m

P|atleast one of (a1 , a2 , · · · , an )


where u1 u2 · · · un are units and p1n+1 , p1n+2 , · · · , p1m are non units. Being a
(iv) Let R be an integral domain with unity. (R is a Euclidean ring) prime element. So the product in right hand side (RHS) cannot be 1. There-

Suppose that for a, b ∈ R, both a|b and b|a then a = µb where µ is a unit in fore m ≤ n.

R Similarly, we can prove n ≤ m [Take a = vb and same applying].

(v) Associative element : Let R be an Euclidean ring with two elements a, b ∈ R In the above process, we also see that pi and pj are as associatesand con-

are said to be associates in R. If b = µa where u is a unit or a = vb where versely.

v is a unit.
Fermat’s theorem on sums of squares:
Unique Factorisation theorem: Before proving the Fermats theorem, we first prove the following lemmas.

Theorem 14.1. Let R be an euclidean ring and a ̸= 0 be a non-element Lemma 14.1. Let p be a prime integer and suppose that for some integer p,
unit in R. Suppose that a = p1 , p2 , p3 , · · · , pn = p11 , p21 , · · · , p1m with Pi and relatively prime to p. We can find integers x and y such that x2 + y 2 = cp, then
Pj are the prime elements of R. Then n = m and each Pi (1 ≤ i ≤ n) is an P can be written as the sum of the squares of two integers. (i.e. p = a2 + b2 )
associate of Pj (1 ≤ j ≤ m).
Proof. We know that, the ring Z of integers is a subring of the Z[i], where Z[i] is
Gaussian integers (Z ⊂ Z[i] )

14. UNIT : CLASSICAL THEOREMS ON EUCLIDEN RINGS 159 14. UNIT : CLASSICAL THEOREMS ON EUCLIDEN RINGS 160

If p ∈ Z ⇒ p ∈ Z[i] If a2 + b2 = p2 then from (∗), we have f 2 + g 2 = 1, which is a contradiction as


We verify that given integer p need not be a prime element of Z[i]. d(f + ig) ̸= d(1)
Given p|cp and cp = x2 + y 2 Therefore a2 + b2 = p for a, b ∈ Z exists by given hypothesis.
⇒ p|x2 + y 2
Lemma 14.2. If p is a prime number of the form 4n + 1, we can solve x2 ≡
p|(x + iy) or p|x − iy where x + iy, x − iy ∈ Z[i].
−1(mod p).
If p is to be a prime element of Euclidean ring Z[i], then
p|(x + iy) or p|(x − iy) Proof. Let p = 4n + 1

Suppose p|x + iy and x + iy = p(u + iv) ( ∵ u + iv ∈ Z[i]) p − 1 = 4n


p−1
Therefore x = pu and y = pv Therefore is an even integer.
2  
p−1 p−1
⇒ p|x and p|y. Choose x = 1, 2, 3, · · · , where is even.
2 2
Suppose p|x − iy, where p|x + iy. (p − 1)
Therefore x = (−1), (−2), (−3), · · · ,
Therefore p2 |x2 + y 2 = p2 |cp = p|c, which is a contradiction.   2 
p−1 −(p − 1)
Therefore x2 = 1, 2, 3, · · · , (−1), (−2), (−3), · · · ,
As (c, p) = 1 2 2
But (p − k) ≡ (−k)(mod p)) or (−k) ≡ (p − k)(mod p)
Therefore p † x + iy.
Similarly, we have p † x − iy.
  
p−1 −(p − 1)
x2 = 1, 2, 3, · · · , (−1), (−2), (−3), · · · , (mod p)
Therefore p is not a prime element of Z[i] for p|(x + iy)(x − iy) but p † (x + iy) 2 2
  
p−1 p − (p − 1)
and p † (x − iy). = 1, 2, 3, · · · , (p − 1), (p − 2), (p − 3), · · · , (mod p)
2 2
In consequence of this, we have
  
p−1 2p − p + 1)
= 1, 2, 3, · · · , (p − 1), (p − 2), (p − 3), · · · , (mod p)
P = (a + ib)(f + ig) such a + ib, f + ig ∈ Z[i] and none of a + ib and f + ig is 2 2
 
(p − 1) (p + 1)
a unit in Z[i] = 1, 2, 3, · · · , , · · · (p − 2)(p − 1) (mod p)
2 2
Note that d(a + ib) ̸= d(1) and d(f + ig) ̸= d(1)
x2 =(p − 1)!(mod p)
Now from p = (a + ib)(f + ig) where p is a prime integer,
we have p = (a − ib)(f − ig) But by Wilson theorem,
⇒ p2 = (a2 + b2 )(f 2 + g 2 ) −→ (∗) (p − 1)! ≡ −1(mod p)
Therefore a2 + b2 |p2 or f 2 + g 2 |p2 x2 ≡ (−1)(mod p).
Therefore a2 + b2 = 1 or p or p2 (Since p is prime)
Fermats theorem for rings:
but a2 + b2 ̸= 1 (∵ d(a + ib) ̸= 1)
a2 + b2 = p or p2 Theorem 14.2. If p is the prime number of the form 4n + 1, then p = a2 + b2
for some integer a and b then there exists x (by above lemma) such that x2 ≡
14. UNIT : CLASSICAL THEOREMS ON EUCLIDEN RINGS 161 14. UNIT : CLASSICAL THEOREMS ON EUCLIDEN RINGS 162

(−1)(mod p).
ASSESSMENT / TERMINAL QUESTIONS
Proof. Since we only need to use the remainder of x divison by p. We can choose
x such that 0 ≤ x ≤ p − 1. That is x < p. 1. Define a prime element in euclidean ring with an example.
p p
We can also assume that |x| < or x ≤ . √
p 2 2 2. Show that Z[ −5] is an integral domain, which is not an unique factorization
Suppose x > , then choose y = p − x.
2 domain.
2 2 2
Therefore y = p + x − 2px.

2 2 2
y ≡ x (modp) where x ≡ −1(modp). 3. Show that Z[ −3] is not an unique factorization domain.
p
Therefore y 2 ≡ −1(modp) with y = p − x < .
2 4. In unique factorization domain R, show that non-zero prime ideal (̸= R)
By replacing x by y, we have the above condition
p contains a prime element.
x2 ≡ −1(modp) with x < .
2
p2 5. If p = 4n + 1, where p is a prime number, then show that x2 ≡ −1(mod p).
Therefore x2 + 1 = cp for some integer c with x2 ≤ but cp = x2 + 1
4
2
p 6. Write a short note on the Wilson’s theorem.
cp ≤ + 1 < p2 ⇒ cp < p2 ⇒ c < p.
4
Therefore (c, p) = 1.
7. Explain the significance of the Fermat’s theorem on sums of squares
Hence, p = a2 + b2 for some integer a and b, due to above lemmas.
8. State and prove the Fermat’s theorem on sums of squares.

SUMMARY REFERENCES

An unique factorization domain (UFD) is a ring in which a statement analogous 1. I. N. Herstein: Topics in Algebra, 2nd Edition, John Wiley and Sons, 2008.

to the fundamental theorem of arithmetic holds. Specifically, a UFD is an integral 2. Surjeet Singh and Qazi Zameeruddin: Modern Algebra, 8th Edition, Vikas
domain in which every non-zero non-unit element can be written as a product of Publishing House, 2021.
prime elements, uniquely up to order and units. Important examples of UFDs are
the integers and polynomial rings in one or more variables with coefficients coming 3. N. Jacobson: Basic Algebra-I, 2nd Edition, Dover Publications, 2009.

from the integers or from a field. 4. M. Artin : Algebra, 2nd Edition, Prentice Hall of India, 2015.

KEYWORDS 5. Darek F. Holt, Bettina Eick and Eamonaa A. O’brien: Handbook of com-
putational group theory, 1st Edition, Chapman & Hall/CRC Press, 2005.
Euclidean function, Euclidean domain, Unique factorization domain, Fermat’s the-
6. J. B. Fraleigh : A first course in abstract algebra, 7th Edition, Addison-
orem on sums of squares.
Wesley Longman, 2002.

14. UNIT : CLASSICAL THEOREMS ON EUCLIDEN RINGS 163 164

7. D. S. Dummitv and R.M. Foote: Abstract Algebra, 2nd Edition, John Wiley 15 UNIT : Polynomial rings
& Sons, 2003.

8. V. K. Khanna and S. K. Bhambri: A course in Abstract algebra, 4th Edition, OBJECTIVES


Vikas Publishing House Pvt Ltd., 2013.
After studying this unit you will be able to:

• explain the concepts of prime element in euclidean ring;

• define the concepts of relatively prime element ;

• describe the inique factorization theorem and their significance ;

• explain some fundamental lemmas of fermat’s theorem;

• state and prove the fermat’s theorem .

INTRODUCTION

A polynomial ring is a ring formed from the set of polynomials in one or more
variables with coefficients in another ring, often a field.

Polynomial Rings:

Definition 15.1. Let R be a commutative ring with unity and x is an indetermi-


nate then the set R[x] = {f (x) = a0 + a1 x + a2 x2 + · · · + an xn /ai ∈ R} becomes a
commutative ring with unity under + and . defined as follows.
For
f (x) = a0 + a1 x + a2 x2 + · · · + am xm

g(x) = b0 + b1 x + b2 x2 + · · · + bn xn , m > n

Then

f (x) + g(x) = (a0 + b0 ) + (a1 + b1 )x + · · · + (an + bn )xn + an+1 xn+1 + · · · + am xm .


15. UNIT : POLYNOMIAL RINGS 165 15. UNIT : POLYNOMIAL RINGS 166

or i.e,. deg(f (x)) = m and deg(g(x)) = n.


f (x).g(x) = c0 + c1 x + · · · + ck xk , (k = m + n) t
 = c0 + c1 x + · · · + ct x , (t = m + n)
But f (x).g(x)
 Pj

 i=0 ai bj−i , if j < (m + i)
with 



 Pj a b ,

if j < (m + i) where cj = am bn , ifj = (M + i)
i=0 i j−i

 

 


c j = am b n , ifj = (M + i) 0,

if j > (m + i).

Therefore cm+n = am bn ̸= 0(am ̸= 0, bn ̸= 0)



0,

if j > (m + i).
i.e,. cm+n is the non-zero coefficient with the term xm+n and all the coefficients
The Ring R[x] is called Polynomial ring and the elements of R[x] are called poly- after cm+n with power of x more than m + n are zero.
nomials. Therefore deg(f (x).g(x)) = m + n = deg(f (x)) + deg(g(x)).

Note 15.1. Therefore deg(f g) = deg(f ) + deg(g).

Lemma 15.2. If the ring R ia an integral domain then R[x] ia also an integral
(i) The Zero element of R[x] is 0 or 0(x) = 0 + 0x + 0x2 + · · · and unity of R[x]
domain.
is, 1 or 1(x) = 1 + 0x + 0x2 + · · ·
Proof. Since R is a commutative ring it follows that R[x] is also commutative ring
(ii) R can be embedded in R[x] under ϕ : R → R[x] given by ϕ(a) = a + ox +
2 , we verify that R[x] has no zero divisors.
0x + · · ·
Let f (x) and g(x) be non-zero polynomials of degree m and n, respectively.
(iii) If R is a commutative ring that R[x] is a commutative ring. i.e., f (x) ̸= 0, g(x) ̸= 0 and deg(f (x) = m ≥ 0), deg(g(x)) ≥ 0.
But deg(f g) = deg(f ) + deg(g).
Degree of polynomial:
Therefore deg(f g) = m + n ≥ 0.
Definition 15.2. The degree of a polynomial f (x) is the largest integer m the Therefore deg(f g) ≥ 0, f (x)g(x) ̸= 0.
coefficient am is not zero. Therefore R[x] has no zero divisors.
i.e,. f (x) = a0 + a1 x + · · · + am xm is a polynomial of degree ′ m′ if am ̸= 0. Therefore R[x] is a commutative ring without zero divisors and having it is an
The degree of f (x) is denoted by deg(f (x)) or deg(f ). Note that degree of 0 - integral domain.
polynomial is undefined but we choose it to be ∞.
Corollary 15.1. If F is a field, then F [x] is an integral domain.
Lemma 15.1. For f, g ∈ R[x], we have deg(f g) = deg(f ) + deg(g).
Proof. Since F is a field if follows that F ia an integral domain.
Proof. Let f (x) = a0 + a1 x + · · · + am xm , (am ̸= 0) Therefore F [x] is an integral domain [by above Lemma].
and g(x) = b0 + b1 x + b2 x2 + ... + bn xn , (bn ̸= 0)
Theorem 15.1. If F is a field, then F [x] is not a field.
be polynomials of degree m and n respectively.

15. UNIT : POLYNOMIAL RINGS 167 15. UNIT : POLYNOMIAL RINGS 168

Proof. Since F is also a field it is a commutative ring with unity 1. i.e,. we assume that the result f (x) = tg(x) + r(x) with r(x) or deg(r(x)), true
We verify that every non-zero element in f [x] need not be units. for all polynomial whose degree deg(g(x)) < deg(f (x)).
m n
Let f (x) be a non-zero element of F [x]. Consider f1 (x) = f (x) − am b−1
n g(x)x x .
Pm−1 Pm−1 i n−r
Therefore deg(f (x)) ≥ 0. Therefore f1 (x) = am xm + i=1 ar x i
− [am b−1 n n
n ][bn x x + 1=0 bi x ]x

If deg(f (x)) = 0, thenf (x) ∈ F . Therefore deg(f1 (x)) ≤ m − 1 < m = deg(f (x)) (i.e,. deg(f1 (x)) < deg(f (x))).
Therefore f (x) = a + 0x + 0x2 + · · · = q ∈ F, a ̸= 0 By induction hypothesis, the result is true for f1 .
Therefore [f (x)]−1 exists in F . Therefore, there exist ti , (x) ∈ F [x] such that ti (x)g(x) + r(x) = f (x), where
i.e,. f (x) is a unit in F . r(x) = 0 or deg(r(x)) < deg(g(x)).
Suppose deg(f (x)) > 0, we verify that [f (x)]−1 does not exist in F [x]. But from by previews equation we have,
m−n
Suppose om the contrary [f (x)]−1 = g(x) ∈ F [x], we observe that g(x) ̸= 0(x). f (x) = f1 (x) + am b−1
n g(x)x
m−n
Because in this case g(x) = 0(x); we have, f (x).g(x) = f (x).0. This implies f (x) = t1 (x)g(x) + r(x) + am b−1
n g(x)x
m−n
that 1 = 0. f (x) = [t1 (x) + am b−1
n x ]g(x) + r(x)
m−n
Therefore deg(g(x)) ≥ 0. i.e,. f (x) = t(x)g(x) + r(x), where t(x) = t1 (x) + am b−1
n x ∈ F [x].
Consider deg(f g) = deg(f ) + deg(g). Thus f (x) = t(x)g(x) + r(x), where r(x) = 0 or deg(r(x)) < deg(g(x)).
deg(f g) > 0. Therefore F [x] is an Euclidean ring.
Therefore deg(1) > 0. (∴ deg(1) = 0)
Theorem 15.3. F [x] is a Principal Ideal ring (PIR).
[f (x)]−1 does not exists in F [x].
Proof. Since F is a field is clear thatF is a commutative ring without zero divisors.
Theorem 15.2. : If F is a field then F [x] is an Euclidean ring.
Therefore F [x] is also a commutative ring without zero divisors. Let S be any
Proof. : Since F is a field we see that F [x] is an integral domain. ideal of F [x]. If S = 0 then S is a principal ideal generated by ’0’ Polynomial.
Define a function d : (F [x] = 0) → Z + by d(f (x)) = deg(f (x)) i.e,. S = 0.
We verify that deg(f g) ≥ d(f ) for f g ∈ F [x] − 0. Suppose on other hand, S ̸= 0. Then there are some non-zero polynomial S.
From deg(f g) = deg(f ) + deg(g), (deg(f ) ≥ 0, deg(g) ≥ 0) we have Let g(x) be a non-zero polynomial in F [x] of least degree.
deg(f g) ≥ deg(f ) (i.e,.d(f g) ≥ d(f )). i.e,. deg(g(x)) < deg(h(x))∀h(x) ∈ S with h(x) ̸= 0.
Now, for any f (x)g(x) ∈ F [x] − 0 we verify that there exist t(x) and r(x) such we now verify that S = g(x).
that f (x) = t(x)g(x) + r(x) with r(x) = 0 or d(r(x)) < d(g(x)). Let f (x) be any polynomial in S.
If deg(f (x)) < deg(g(x)) then we have f (x) = 0(x)g(x) + f (x). Therefore f (x)g(x) ∈ F [x], where F [x] is a Euclidean ring.
Where m ≥ n, we prove that result by induction on deg(f (x)), Where r(x) = 0 or deg(r(x)) < deg(g(x)).
Therefore r(x) = f (x) = t(x)g(x) ∈ S.
15. UNIT : POLYNOMIAL RINGS 169 15. UNIT : POLYNOMIAL RINGS 170

If r(x) ̸= 0, then r(x) ∈ S with the condition that deg(r(x)) < deg(g(x)), Therefore [2ψ(x) − xϕ(x)]g(x) = 0
which is not possible. where [2ψ(x) − xϕ(x)]g(x) ∈ Z[x] an In domain (∵ g(x) ̸= 0)
∴ r(x) = 0. [2ψ(x) − xϕ(x)] = 0
Hence f (x) − t(x)g(x). Clearly, all coefficients of ϕ(x) are even integers.
∴ All elements of S are multiples of g(x). Therefore ϕ(x)2h(x) −→ (2)
∴ S is an orbitrary ideal of F [x]. for some polynomial h(x) ∈ Z[x]
Therefore S = (g(x)). By using (2) in (1) we see that
F [x] =⇒ all ideals of F [x] are Principle ideals. 2 = 2h(x)g(x) ⇒ h(x)g(x) = 1 ∈ S
∴ F [x] is an Principle ideal ring. Therefore 1 ∈ S
But we can verify that 1 ∈
/S
Note 15.2. F [x] is a principle ideal ring, when F is a field, but R[x] is not a
Because 1 ∈ S ⇒ 1 = 2p(x) + xq(x); p, q ∈ Z[x].
principle ideal ring over any orbitrary ring.
⇒ 1 = 2{a0 + a1 x + · · · + an xn } + x{b0 + b1 x + · · · + bm xm }
Example 15.1. Consider Z[x], where Z is not a field. 1 = 2a0
Therefore a0 ∈ Z
Solution : We verify that Z[x] is not a PIR.
∴ S is not a principle ideal.
Let S = ⟨2, x⟩ = {2p(x) + xq(x)|p(x)q(x) ∈ Z[x]}
Therefore Z[x] is not a Principle ideal ring (PIR).
For a(x) = 2p1 x + xq1 x and b(x) = 2p2 x + xq2 (x) be elements of S.
Therefore a(x) − b(x) = 2[p1 (x) − p2 (x)] + x[q1 (x) − q2 (x)]
Euclidean algorithm for polynomials over a field:
where {p1 (x) − p2 (x)} ∈ Z[x] and {q1 (x) − q2 (x)} ∈ Z[x]
Therefore a(x) − b(x) ∈ S Theorem 15.4. Let F be a field and f (x), g(x) be any two polynomials in F [x]
Similarly, we can show that r(x)a(x) and a(x)r(x) ∈ S for some r(x) ∈ Z[x]. not both of which are zero then f (x) and g(x) have a greatest common divissor
Therefore S is not a principle ideal. GCD d(x) which can be expressed in the form d(x) = m(x)f (x) + n(x) + g(x) for
Suppose S is a principle ideal generated by a polynomial g(x) ∈ Z[x] some polynomials m(x) and n(x) in F [x].
i.e, S(g(x))
Proof. Let S = {s(x)f (x) + t(x)g(x)|s(x)t(x) ∈ F [x]}
Since 2 and x are in S we have
Let a(x) = {s1 (x)f (x) + t1 (x)g(x) and b(x) = {s2 (x)f (x) + t2 (x)g(x)
2 = ϕ(x)g(x) −→ (1)
Therefore a(x) − b(x) = [s1 (x) − s2 (x)]f (x) + [t1 (x) − t2 (x)]g(x).
and x = ψ(x)g(x) (becauseϕ, ψ ∈ Z[x])
where s1 (x) − s2 (x) and t1 (x) − t2 (x) ∈ F [x]
Therefore 2x = ϕ(x)g(x) and 2x = 2ψ(x)g(x)
Therefore a(x) − b(x) ∈ S
⇒ 2ψ(x)g(x) = xϕ(x)g(x)
Similarly for any r(x) ∈ F [x], we have

15. UNIT : POLYNOMIAL RINGS 171 15. UNIT : POLYNOMIAL RINGS 172

r(x)a(x) = r(x){s1 (x)f (x) + t1 (x)g(x)} Example 15.2. p(x) = x2 + 1 is a polynomial of degree 2 x2 + 1 ∈ R[x] is
r(x)a(x) = [r(x)s1 (x)]f (x) + [r(x)t1 (x)]g(x) irreducible. [∵ (x2 + 1) = (x + i)(x − i) : x + i, x − i ∈
/ R[x]] over R. But
where r(x)s1 (x) and r(x)g(x) ∈ F [x] p(x) = x2 + 1 ∈ C[x] is irreducible over C [(x2 + 1) = (x + i)(x − i)|x ± i ∈ C[x]].
Therefore r(x)a(x) ∈ S Similarly, p(x) = x2 + 1 ∈ Z2 [x], Z2 = {0, 1}.
Therefore S is an ideal of F [x], which is a euclidean ring.
Since every euclidean ring is a principle ideal ring it follows that F [x] is a p(x) =x2 + 1

principle ideal. =x2 + 2x + 1(∵ 2 = 0 in Z2 )


Therefore S is a principle ideal. =(x + 1)(x + 1)
Let S = (d(x)) for some d(x) ∈ F [x] (∵ d(x) ∈ S we have d(x) = m(x)f (x) + p(x) =x2 + 1.
n(x)g(x) for m(x)n(x) ∈ F [x].
Now we verify that d(x) is the GCD of f (x) and g(x). Therefore p(x) = x2 + 1 is irreducible in Z2 .
Since 1, 0 ∈ S we have,
Note 15.4. If p(x) is irreducible over a field F then p(x) has no roots in F .
f (x) = 1 · f (x) + 0 · g(x) and g(x) = 0 · f (x) + 1 · g(x) ∈ S
Therefore f (x) · g(x) ∈ S Note 15.5. Any polynomial in F [x] can be written as product of finite number
Since all elemente of S in d(x);
Note 15.6. If the ring R is an integral domain with unity, then the following are
d(x)|f (x) and d(x)|g(x)
equivalent:
Let c(x) be element of F [x] suchthat c(x)|f (x) and c(x)|g(x)
Now, c(x)|f (x) and c(x)|g(x) (i) R is a Unique factorization domain.
⇒ c(x)|m(x) · f (x) and c(x)|n(x) · g(x)
(ii) Every non-zero, non-unit element of R is a finite product of irreducible elel-
⇒ c(x)|[m(x) · f (x) + n(x) · g(x)]
ments and every irreducible element is prime.
c(x)|d(x)
Therefore d(x) is GCD of f (x) and g(x) or d(x) = (f (x)g(x)). (iii) Every non-zero , non-unit element of R is a finite product of prime elments.

Irreducible Polynomial: Lemma 15.3. The ideal A = (p(x)) in F [x] is a maximal ideal if and only if p(x)
is an irreducible polynomial in F [x].
Definition 15.3. A non-zero polynomial p(x) in F [x] is said to itrreducible over
F if p(x) = a(x)b(x) for a(x)b(x) ∈ F [x] implies one of x(x) and b(x) is a unit in Proof. Let A = (p(x)), where p(x) is an irreducible polynomial. We verify that A

F [x]. is a maximal ideal.


Suppose U is any ideal of F [x] satisfy A ⊂ U ⊂ F [x].
Note 15.3. Irreduciblility of a polynomial depends on the fields of reference.
Since U is an Euclidean ring F [x], it follows that U is a principal ideal.
15. UNIT : POLYNOMIAL RINGS 173 15. UNIT : POLYNOMIAL RINGS 174

Let U = (q(x)) for some q(x) ∈ F [x]. Since p(x) ∈ A ⊂ U , we have p(x) ∈ U and commutative algebra, and algebraic geometry.
therefore p(x) = h(x)q(x) for some h(x) ∈ F [x].
Therefore h(x) or q(x) is a unit. KEYWORDS
−1
Suppose h(x) is aunit. Then p(x) = h(x)q(x) =⇒ q(x) = p(x)h (x) ∈ A.
Polonomial rings, degree of polynomial, euclidean algorithm for polynomials, irre-
Therefore q(x) ∈ A and hence U ⊂ A, which leads to U = A.
−1 −1 ducible polynomial.
Similarly, if q(x) is a unit, then q (x) ∈ F [x]. Then q(x)q (x) ∈ U (i.e., 1 ∈ U ).
Therefore U = F [x]. Thus A ⊂ U ⊂ F [x] =⇒ U = A or U = F [x].
ASSESSMENT / TERMINAL QUESTIONS
Therefore A is a maximal ideal of F [x].
Converserly, suppose that A = (p(x)) is a maximal ideal of F [x]. 1. Explain the significance of polynomial rings.
We verify that p(x) is irreducible polynomial in F [x]. Suppose p(x) is reducible,
2. Define an irreducible polynomial with an example.
that is p(x) = q(x)r(x), where both q(x) and r(x) ar non-units.
Choose B = (q(x)). Clearly, p(x) = q(x)r(x) =⇒ p(x) ∈ B. Therefore A ⊂ B ⊂ 3. If the ring R ia an integral domain then show that R[x] ia also an integral
F [x]. domain.
We prove that B ̸= A and B ̸= F [x].
4. Show that x3 + 3x2 + 2 is irreducuble over Z5 .
If B = A, then q(x) ∈ B =⇒ q(x) ∈ A, and therefore
q(x) = p(x)g(x) or q(x)r(x) = p(x)g(x)r(x). So, p(x) = p(x)g(x)r(x) which 5. If F is a field then F [x] is an Euclidean ring.

reduces to [1 − q(x)r(x)]p(x) = 0. hence g(x)r(x) = 1.


6. Show that F [x] is a Principal Ideal ring.
Therefore r(x) is a unit in F [x], which is not true.
Similarly, if B = F [x], then 1 ∈ B since 1 ∈ F [x]. Therefore 1 = q(x)t(x) for 7. Prove that p(x) = x2 + x + 1 ∈ Z2 [x] is irreducible and hence verify that

some t(x) ∈ F [x]. Thus q(x) is a unit in F [x], which is not true. A = (p(x)).

Therefore A ⊂ B ⊂ F [x] =⇒ B ̸= A and B ̸= F [x]. Therefore A is not maximal 8. Verify that p(x) = x3 − 2 ∈ Q[x] is irreducible in Q[x]. Hence deduce that
ideal, which is contradiction. Therefore p(x) is an irreducible polynomial. A = (p(x)), is maximal ideal in Q[x].

REFERENCES
SUMMARY
1. I. N. Herstein: Topics in Algebra, 2nd Edition, John Wiley and Sons, 2008.
The importance of such polynomial rings relies on the high number of properties
2. Surjeet Singh and Qazi Zameeruddin: Modern Algebra, 8th Edition, Vikas
that they have in common with the ring of the integers. Polynomial rings occur
Publishing House, 2021.
and are often fundamental in many parts of mathematics such as number theory,

3. N. Jacobson: Basic Algebra-I, 2nd Edition, Dover Publications, 2009.

15. UNIT : POLYNOMIAL RINGS 175 176

4. M. Artin : Algebra, 2nd Edition, Prentice Hall of India, 2015. 16 UNIT : Polynomial over the Rational field
5. Darek F. Holt, Bettina Eick and Eamonaa A. O’brien: Handbook of com-
putational group theory, 1st Edition, Chapman & Hall/CRC Press, 2005. OBJECTIVES

6. J. B. Fraleigh : A first course in abstract algebra, 7th Edition, Addison- After studying this unit you will be able to:
Wesley Longman, 2002.
• explain the concepts of prime element in euclidean ring;
7. D. S. Dummitv and R.M. Foote: Abstract Algebra, 2nd Edition, John Wiley
• Study the properties of primitive and monic polynomials;
& Sons, 2003.

• state and prove the Gauss lemma ;


8. V. K. Khanna and S. K. Bhambri: A course in Abstract algebra, 4th Edition,
Vikas Publishing House Pvt Ltd., 2013. • explain some fundamental properties of primitive polynomials;

• solve the problem related to Eisenstein criteria for irreducibility of a poly-


nomial.

INTRODUCTION

In this unit we choose the polynomials with integer coefficients and determine a
criteria for irreducibility of a polynomial. Particularly, Eisenstein’s irreducibility
criterion is a method for proving that a polynomial with integer coefficients is
irreducible

Primitive polynomials

Definition 16.1. The Polynomial f (x) = a0 + a1 x + a2 x2 + ... + an xn ∈ Z[x] is


said to be primitive, if the gcd of the integers a0 , a1 , ...an is 1.

In other words, f (x) is primitive in Z[x], if its co-efficients are relatively primes.

Definition 16.2. The content of a polynomial f (x) = a0 + a1 x + ... + an ∈ Z[x]


is the gcd of the co-efficients a0 , a1 , ...an . It is denoted by c(f ).
16. UNIT : POLYNOMIAL OVER THE RATIONAL FIELD 177 16. UNIT : POLYNOMIAL OVER THE RATIONAL FIELD 178

Note 16.1. We now verify that f (x)g(x) is primitive.


Suppose f (x)g(x) is not primitive. Then c0 , c1 , ...cm+n are all divisible by
(i) c(f ) = 1, if f (x) is a primitive polynomial.
integer larger then one and hence by some prime integer p.
(ii) If f (x) ∈ Z[x] is non primitive polynomial then f (x) = c(f )fp (x), where i.e., p/cl for l = 0, 1, 2, ..., m + n.
fp (x) is a primitive polynomial. Therefore p/a0 b0 , p/a1 b1 , ..., p/an bm etc.
Hence p/a0 , or p/b0 , p/a1 or p/b1 , ...p/an or p/bm .
Definition 16.3. A polynomial f (x) in Z[x] is said to be integer monic poly-
Since f (x) is primitive it follows that a can not divide all the co-efficients ai
nomial, if co-efficient of its highest degree in x is 1. (i.e., leading co-efficient of
(i = 0, 1, 2, ...n).
f (x) is 1).
Let aj be the first co-efficient of f (x) which is not divisible by p.
h
Theorem 16.1. The product of two primitive polynomials is a primitive, or if c(f ) = Therefore p/a0 , a1 , ...aj−1 and p/b0 , b1 , ...bk−1 , which implies that
i
1, c(g) = 1, then c(f g) = 1. p/(a0 bj+k + a1 bj+k−1 , ..., aj−1 bk ) and p/(b0 aj+k + b1 aj+k−1 , ..., aj+1 bk−1 ). From
(1), we see that p/aj bk , and therefore p/aj or p/bk , which is not true.
Proof. Let f (x) = a0 +a1 x+a2 x2 +...+am xm and g(x) = b0 +b1 x+b2 x2 +...+bn xn
Therefore f (x)g(x) is a primitive polynomial.
be primitive polynomial in Z[x].
Then f (x)g(x) = c0 + c1 x + c2 x2 + ... + ct xt , where, t = m + n and Corollary 16.1. If f (x), g(x) ∈ Z[x], then c(f g) = c(f )c(g).

Proof. For f (x), g(x) ∈ Z[x], we have f (x) = c(f )fp (x) and g(x) = c(g)gp (x),
 j
P
aj bj−i if j < m + n



where fp (x) and gp (x) are primitive. Therefore f (x).g(x) = c(f ).c(g)fp (x)gp (x)

i=0

cj = am bT if j = m + n

 and c(f g) = c(f )c(g)c(fp (x)gp (x)).


0 if j > m + n. Hence c(f g) = c(f )c(g), since fp (x), gp (x) are primitive.

For j + k < m + n, we have Gauss lemma:


If a primitive polynomial f (x) can be factorized as a product of two polynomials
j+k
having rational co-efficients, it can be factorized as the product of two polynomials
X
cj+k = ai bj+k−i
i=0 having integer co-efficients [or If a primitive polynomial is reducible over Q, then
= a0 bj+k + a1 bj−k−1 + ... + aj−1 bk+1 + aj bk + ... + aj+k b0
it is reducible over Z also].
= aj bk + (a0 bj+k + ... + aj−1 bk+1 ) + (aj+1 bk−1 + ... + aj+k b0 )
Proof. Let f (x) be a primitive polynomial which can be factorized over the ratio-
+ (b0 aj+k + ... + aj+1 bk−1 ).

h
Therefore, we have aj bk = cj+k − a0 bj+k + ... + aj−1 bk+1 + (b0 aj+k + ... +
i
aj+1 bk−1 ) −→ (1)

16. UNIT : POLYNOMIAL OVER THE RATIONAL FIELD 179 16. UNIT : POLYNOMIAL OVER THE RATIONAL FIELD 180

nals Q. Therefore, Proof. Without loss of generality we may assume that f (x) is primitive, because,
by taking out the greatest common factor d of its co-efficients the hypothesis of
f (x) = g(x)h(x), whereg(x), h(x) ∈ Q[x] theorem is not disturbed, since p ∤ an implies p ∤ d.
  
p0 p1 p2 r0 r 1 r2
= + x + x2 + ... + x + x2 + ... Suppose f (x) = a0 + a1 x + ... + an−1 xn−1 + an xn is reducible over Q. Then by
q0 q1 q2 s0 s1 s2
α(a0 + a1 x + a2 x2 + ...) γ(b0 + b1 x + b2 x2 + ...) Gauss lemma it is reducible over Z.
= . ,
β δ Therefore (f (x) = a0 + a1 x + ... + an−1 xn−1 + an xn ) = (b0 + b1 x + ... + br xr )(c0 +
c1 x + ... + cs xs ).
where, β = LCM (q0 , q1 , q2 , ...) and δ = LCM (s0 , s1 , s2, ...).
where bi s and cj s are integers and r > 0, s > 0, n = r + s.
α = gcd(p0 , p1 , p2 , ...), γ = GCD(r0 , r1 , r2 , ...)
By comparing the co-efficients of like terms we get,
Therefore f (x) = µλ (a0 + a1 x + ...)(b0 + b1 x + ...), where λ = αγ, µ = βδ.
f (x) = µλ g ∗ (x)h∗ (x), where g ∗ (x), h∗ (x) are primitive in Z[x] −→ (1)
a0 = b0 c0
µf (x) = λg ∗ (x)h∗ (x)
a1 = (b0 c1 + b1 c0 )
c(µf (x)) = λc(g ∗ (x)h∗ (x))
...
µc(f ) = λc(g ∗ )c(h∗ ) k
X
ak = (b0 ck + b1 ck−1 + b2 ck−2 + ... + bk c0 ) = bj ck−j .
µ.1 = λ.1
j=0
µ = λ.
Therefore, bk c0 = ak − (b0 ck + b1 ck−1 + b2 ck−2 + ... + bk−1 c1 ) −→ (1)
From (1), we get f (x) = g ∗ (x)h∗ (x).
Now p/a0 =⇒ p/b0 c0 and therefore p/b0 or p/c0 .
∗ ∗
Thus f (x) = g(x)h(x) =⇒ f (x) = g (x)h (x), where f (x), g(x) ∈ Q[x] and
We verify that, p can not divide both b0 and c0 . If p/b0 and p/c0 , then p2 /b0 c0 .
g ∗ (x), h∗ (x) ∈ Z[x].
Hence p2 /a0 , which is contradiction to the given condition p2 ∤ a0 .
Corollary 16.2. Monic polynomial which is factorized as the polynomia of two We see that p can not divide all the b′ s. Because if all the b′ s were divisible
non-constant polynomials having rational co-efficients is also factorized as the by p, then all the co-efficients of f (x) would be divisible by p. In particular p/an
product of two monic polynomial with integer co-efficients. since an = br cs . This is a contradiction.
Suppose bk is the first b which is not divisible by p. that is p ∤ bk , where
Proof. Proof follows by the fact that a monic polynomial is primitive.
k ≤ r ≤ n. We have, p/ak , p/(b0 ck + ... + bk−1 c1 ).
Eisenstein criteria for irreducibility of a polynomial: Then from (1), we see that p/bk c0 .

2 n Therefore p/bk or p/c0 , which is contradiction.


Theorem 16.2. Let f (x) = a0 +a1 x+a2 x +...+an x be a polynomial with integer
Therefore f (x) is irreducible over the rationals Q.
co-efficients. Suppose that for some prime p ≥ 2, p/a0 , p/a1 , ..., p/an−1 , p ∤ an and
p2 /a0 . Then f (x) is irreducible over Q. Corollary 16.3. A polynomial f (x) is irreducible over Q if and only if f (x + α)
16. UNIT : POLYNOMIAL OVER THE RATIONAL FIELD 181 16. UNIT : POLYNOMIAL OVER THE RATIONAL FIELD 182

is irreducible over Q, where α ∈ Q. Example 16.1. f (x) = 2x2 − 4x + 6 is irreducible over Q.

Proof. Suppose f (x) is irreducible over Q so that it has no roots in Q. To verify Consider f (x) = 2x2 − 4x + 6.
that f (x + α) is irreducible over Q. We claim that f (x + α) has no roots in Q.
h i
Suppose, if possible, f (x + α) has a root β in Q. Then f (x + k) = 2 (x + k)2 − 2(x + k) + 3
h i
= 2 x2 + 2xh + k 2 − 2x − 2k + 3
h i
f (x + α) = (x + α − β) h(x + α), where h(β) ̸= 0. Therefore has
h i h i h i h i
f (β − α) + α = (β − α) + α − β h β − α + α = 2 x2 + (2k − 2)x + (k 2 − 2k + 3)
f (β) = 0.h(β) = (6 − 4k + 2k 2 ) + (4k − 4)x + 2x2 .
f (β) = 0.
Put k = 1, then f (x + 1) = 2x2 + 4 = 2(x2 + 2).
i.e., β is root f (x) in Q, which is a contradiction.
Therefore f (x + 1) is irreducible implies f (x) is irreducible.
So f (x + α) has no roots in Q and hence it is irreducible over Q.
Conversly, suppose f (x + α) is irreducible over Q, so that it has no roots in Excercise 16.2. Prove that (i) x3 − 3x − 1 (ii) x3 + x2 + 2x − 1
rational Q.
Solution. (i) f (x) = x3 − 3x − 1
To prove f (x) is irreducible over Q, we prove that it has no roots in Q. Suppose
f (x) has root say γ in Q. Then f (x + k) = (x + 3)3 − 3(x + k) − 1

f (x) = (x − γ)g(x) where g(γ) ̸= 0 = x3 + 3x2 k + 3k 2 x + k 3 − 3x − 3k − 1


h i
f (x + α) = (x + α) − γ) g(x + α) = x3 + 3kx2 + 3(k 2 − 1)x + (k 3 − 3k − 1).
h i h i
f (γ − α) + α = (γ − α) + α − γ g(γ − α + α)
h i
f (γ − α) + α = 0.g(γ) Put k = 1, then f (x + 1) = x3 + 3x2 + 0x − 3.
h i
f (γ − α) + α = 0. Therefore 3 is a prime 3/(−3), 3/0, 3/3, 3 ∤ 1 and 32 ∤ (−3).

Therefore γ − α is a root of f (x + α), which is contradiction. Therefore f (x) has


(ii) f (x) = x3 + x2 − 2x − 1
no roots in Q.
Hence it is irreducible over Q.

Excercise 16.1. If p is a prime number , prove that the polynomial xn − p is f (x + k) = (x + 3)3 + (x + k)2 − 2(x + k − 1)
irreducible over Q.
= x3 + 3x2 k + 3k 2 x + k 3 + x2 + 2xk + k 2 − 2x − 2k − 1
Solution. let f (x) = xn − p = −p + 0.x + 0.x2 + ... + 0.xn−1 + xn , where p is a = x3 + (3k + 1)x2 + (3k 2 + 2k − 2)x + (k 3 + k 2 − 2k − 1.
prime satisfying p/(−p), p/0, p/0, ..., p/0 and p ∤ 1, p2 ∤ (−p).
Therefore by Einstein criteria f (x) is irreducible over Q. Put k = 2. Then, f (x + 2) = x3 + 7x2 + 14x + 7.

16. UNIT : POLYNOMIAL OVER THE RATIONAL FIELD 183

So, 7 is a prime satisfying 7/7, 7/14, 7/7, 7 ∤ 1 and 72 ∤ 7.


By Einstein’s criteria, we see that f (x + 2) is irreducible over Q.
Hence f (x) is also irreducible over Q.

Excercise 16.3. Prove that the cyclotomic polynomials, ϕ(x) = xp−1 + xp−2 + ... +
x + 1, where p is a prime is irreducible over Q.
x−1
Solution. Consider ϕ(x) = x−1

(x − 1)p − 1
ϕ(x + 1) =
(x + 1) − 1
p(p−1) p−2
xp + pxp−1 + 2
x ... + px + 1 − 1
=
x
p(p − 1) p−1
= ϕ(x + 1) = xp−1 + pxp−2 + x ... + p.
2

Therefore p is a prime satisfying p/p, ...p/ p(p−1)


2
, p/p, p ∤ 1 and p2 ∤ p.
By Eistein criteria, we see that ϕ(x + 1) is irreducible over Q and hence ϕ(x) is
also irreducible over Q.

Example 16.2. 1 + x + x2 + ...v + x2n−1 .

Excercise 16.4. If m and n are relatively primes and if (x − m/n)/(a0 + a1 x +


... + ar xr ), where a′ s are integer, then m/a0 and n/ar .
Hence deduce that if arr integer monic polynomial has a factore (x − r), ∈ Q, then
r is an integer (i.e., r ∈ Z).

Solution. Let (a0 + a1 x + ... + ar xr ) = (x − m/n)(b0 = b1 x + ... + br−1 xr−1 ).


Therefore a0 = − m b , a1 = b0 −
n 0
m
b , ...ar−1
n 1
= br−2 − m
b ,a
n r−1 r
= br−1 , where
(m, n) = 1.
Now from a0 = − m b , we have na0 = −mb0 and therefore m/na0 . Hence m/a0 ,
n 0

since (m, n) = 1.
Therefore m/na0 . Hence m/a0 , (since (m, n) = 1).
m
Similarly from ar−1 = br−2 − b ,
n r−1
we get − m b
n r−1
= ar−1 − br−2 ∈ Z.

You might also like